Konstitucinė teisė

1. KONSTITUCINĖ TEISĖ IR KONSTITUCIJA
1.1. KONSTITUCINĖS TEISĖS SAMPRATA IR YPATUMAI
KONSTITUCINĖS TEISĖS ESMĖ Bazinė valstybės teisės normų dalis tradiciškai vadinama konstitucine teise. Ją sudaro pagrindinės valstybės ir ją įkūrusios bendruomenės (piliečių) politinės ir teisinės vertybės (pvz., žmogaus, šeimos, kalbos, religijos, aplinkos, darbo ar verslo svarbiausi aspektai) ir principai (pvz., nepriklausomybės ar demokratijos principai) bei su jais susijusios normos, t. y. viešojo ir iš dalies privataus elgesio taisyklės (pvz., parlamento nario ar teisėjo veiklos, tėvų ir vaikų tarpusavio elgesio taisyklės). Visa tai gali būti išreikšta viename dokumente, pavadinimu „Konstitucija” ar „Pagrindinis Įstatymas”, ir jį detalizuojančiuose teisės aktuose (įstatymuose, valdymo aktuose ar teismų sprendimuose) arba pateikta įvairiuose aktuose ar įstatymuose, kurių visuma rodytų esant nesusistemintą konstituciją. Bendruoju požiūriu konstitucinę teisę turi kiekviena valstybė, nesvarbu, ar ji išleido specialų dokumentą, pavadinimu „Konstitucija”. Jei valstybė tokio dokumento neturi, tai savaime nereiškia, kad jos teritorijoje nėra jokio pamatinio politinio ir teisinio sutvirtinimo, neleidžiančio įsivyrauti savivalei ir anarchijai. Antai konstitucijos, kaip vientiso vertybių, principų ir normų rinkinio, neturinčioje Didžiojoje Britanijoje iš esmės galioja seniausia pasaulyje konstitucinė teisė, kurios pagrindą sudaro įvairiais šimtmečiais vardan individo ir parlamentinės demokratijos apsaugos priimti parlamento aktai, teismų sprendimai ir susiformavę papročiai. Apibendrinant galima sakyti, kad konstitucinė teisė – tai sistema konkrečios šalies politinių ir teisinių vertybių, principų ir normų, įtvirtinančių ir reglamentuojančių esminius žmogaus, piliečio bei visuomenės junginių santykius su valstybės institucijomis ir kitais asmenimis, pagrindinių valstybės valdžios institucijų organizavimo ir veikimo tvarką.
KONSTITUCINĖS TEISĖS SĄVOKA Jei kiekviena valstybė, kaip teigėme, turi konstitucinę teisę, tai atrodytų, kad mūsų pateiktas tokios teisės apibrėžimas turėtų tikti kiekvienai valstybei, nesvarbu, ar jos pamatinė santvarka turi monarchijos, parlamentinės (prezidentinės) respublikos, autoritarinio valdymo ar vienos partijos diktatūros požymių. Tačiau XX amžiaus istorija rodo, kad vienų valstybių (JAV, VFR ar Prancūzijos) organizavimo pagrindą sudaro demokratiškai priimtos teisės normos, kitų (buvusios TSRS ir Jugoslavijos, Kubos ar kitos „socialistinės” ar „liaudies” valstybės) – politinio lyderio nubrėžti prioritetai ar vienintelės partijos suvažiavimų sprendimai, dar kitų (Irano, Saudo Arabijos) – religinės vertybės ir jų nulemtos normos. Remiantis tuo, galima
65
KONSTITUCINĖ TEISĖ
teigti, kad konstitucinę teisę geriausiai galima apibrėžti įvertinus konkrečios valstybės konkretaus laikotarpio konstitucinę teisę. Dabartinę Lietuvos Respublikos konstitucinę teisę, įsigaliojus 1992 m. Konstitucijai (toliau – Konstitucija) ir pradėjus veikti Konstituciniam Teismui, galima apibrėžti dvejopai. Pirmiausia ją galima apibrėžti pagal tai, kaip įformintas aukščiausios teisinės galios dokumentas ir kokių aktų formos jame numatytos. Šiuo platesniu požiūriu konstitucinė teisė – tai Konstitucijoje ir joje (tiesiogiai ar netiesiogiai) numatytuose įstatymuose bei juose numatytuose poįstatyminiuose teisės aktuose įtvirtintų principų ir normų sistema. Taigi tokiu atveju konstitucinės teisės šaltiniai yra Konstitucija ir jos nuostatas detalizuojantys įstatymai – konstituciniai įstatymai (Konstitucijos 47 str., 69 str. 3 d., 150 str.) ir paprastieji (t. y. nekonstituciniai) įstatymai, pavyzdžiui, Vyriausybės įstatymas (Konstitucijos 93 str.), Konstitucinio Teismo įstatymas (Konstitucijos 102 str. 2 d.), rinkimų įstatymai (Konstitucijos 34 str. 2 d.), įstatymo galią turintis Seimo statutas (Konstitucijos 76 str.) ir norminiai poįstatyminiai aktai: Vyriausybės nutarimai (Konstitucijos 95 str.), Respublikos Prezidento dekretai (Konstitucijos 85 str.). Tokią konstitucinės teisės sampratą galima vertinti teigiamai ta prasme, kad ji apibrėžta tokiais Konstitucijos žodžiais, kaip „nustato įstatymas”, „saugo įstatymas” (Konstitucijos 12 str. 3 d., 19 str.), ir yra pagrįsta detalizuojančiuose įstatymuose tradiciškai vartojamais žodžiais „Vyriausybės nustatyta tvarka”. Kita vertus, pateiktą apibrėžimą galima vertinti ir kritiškai, nes, pirma, jame pernelyg sureikšminti įstatymai ir juos leidžiantis Seimas, kuris pagal dabartinę Konstituciją nėra aukščiausioji valstybės institucija. Pažymėtina, kad Konstitucijoje nėra nuostatos, panašios į Lietuvos Respublikos 1990 m. Laikinojo Pagrindinio Įstatymo principą „Aukščiausioji Taryba – aukščiausiasis valstybinės valdžios organas”. Taigi Seimas nėra pagrindinis Konstitucijos aiškintojas ir įgyvendintojas, nes jo teisinę kūrybą Konstitucinis Teismas gali pripažinti prieštaraujančia Konstitucijai. Antra, pateiktas apibrėžimas perša mintį, kad Konstitucija tarsi galioja „įstatymų nustatyta tvarka”, nors pagal 1992 m. Konstituciją (7 str. 1 d., 105 str. 1 d., 107 str. 2 d.) įstatymai turi galioti Konstitucijos nustatyta ir Konstitucinio Teismo išaiškinta tvarka. Trečia, minėtasis konstitucinės teisės apibrėžimas sudaro įspūdį, kad ir kiti Konstitucijoje numatyti norminiai poįstatyminiai sprendimai – ministro sprendimai „ministerijos kompetencijai priklausančiais klausimais” (K 98 str. 1 d.) ar savivaldybių tarybų sprendimai (Konstitucijos 119 str. 4 d., 121 str. 2 d.) – neva yra konstitucinės teisės šaltiniai. Išeitų, kad norint visiškai pažinti konstitucinę teisę, būtina analizuoti ir visų ministrų bei savivaldybių tarybų priimtus ir galiojančius norminius sprendimus, susijusius su Konstitucija. Tokiu atveju konstitucinės teisės studijos iš esmės būtų administracinės teisės studijavimas. Ketvirta, minėtąjį konstitucinės teisės apibrėžimą kritiškai galima vertinti ir todėl, kad Konstitucijos II skirsnyje „Žmogus ir valstybė” bei kituose skirsniuose nėra žodžių „nustato Vyriausybė” ar „Vyriausybės nustatyta tvarka”. Tai rodo, kad žmogaus teisių ir laisvių bei kitų konstitucinių vertybių esminiai klausimai negali būti reglamentuojami poįstatyminiais aktais. Visa tai turint omenyje, pateikto konstitucinės teisės apibrėžimo nereikėtų vertinti rimtai. Priešingu atveju konstitucinę teisę turėtume vadinti valstybine teise. Siauresniu požiūriu dabartinę Lietuvos Respublikos konstitucinę teisę galima apibrėžti ir pagal tai, koks aukščiausiosios teisinės galios dokumentas ir jame
66
Konstitucinė teisė ir Konstitucija
įtvirtintas vertybes detalizuojantis reglamentavimas – įstatymų, poįstatyminių aktų ar teismų sprendimų pagrindu – yra svarbiausias, pastovus ir neginčijamas teismine tvarka. Šiuo požiūriu konstitucinė teisė -tai prigimtinių žmogaus teisių ir laisvių bei kitų vertybių (pirmiausia – tautos suvereniteto), jų apsaugos, gynimo ir įgyvendinimo institucines bei organizacines priemones įtvirtinančios Konstitucijos ir ją galutinai ir neskundžiamai (Konstitucijos 107 str. 2 d.) aiškinančio Konstitucinio Teismo sprendimuose išplėtotų principų bei koncepcijų visuma. Šiuo atveju konstitucinės teisės šaltiniai yra prigimtinės žmogaus teisės ir laisvės (jas sukuria žmogaus gimimas, bet ne valstybė ar konstituciją referendumu priimantys piliečiai), tautos suverenitetas (jis egzistuoja savaime prieš sukuriant valstybę ir po to), visa tai aprašanti Konstitucija ir ją aiškinančio bei taikančio Konstitucinio Teismo sprendimai. Tokią sampratą, viena vertus, galima laikyti nepakankama ir vienpusiška, nes joje pernelyg sureikšminama devynių Konstitucinio Teismo teisėjų (Konstitucijos 103 str. 1 d.) sprendimų reikšmė ir tarsi nuvertinami 141 Seimo nario priimti įstatymai. Kita vertus, pateiktą konstitucinės teisės apibrėžimą iš esmės galima vertinti kaip tinkamiausią, nes visi įstatymai ar Vyriausybės aktai gali būti Konstitucinio Teismo pripažinti antikonstituciniais ir todėl netaikomais (Konstitucijos 107 str. 1 d.). Be to, konstitucinė teisė iš principo turi būti pastovi ir negali keistis vien todėl, kad pasikeitė įstatymai ar poįstatyminiai aktai (o jie paprastai keičiasi, po rinkimų pasikeitus Seimo ir Vyriausybės sudėčiai ar šių institucijų prioritetams).
KONSTITUCINĖS TEISĖS REGULIAVIMO SRITIS (APIMTIS) Ką reguliuoja konstitucinė teisė, pasakyti trumpai ir kartu išsamiai neįmanoma. Šiuo atveju svarbu pabrėžti, kad konstitucinė teisė turi trigubą objektą, t. y. ji reguliuoja (daro norminamąjį, t. y. stabilizuojamąjį, organizuojamąjį, integruojamąjį ir apsauginį – poveikį): 1) pamatinius valstybės organizavimo principus ir procedūras, pirmiausia demokratinės, respublikinės, nepriklausomos ir teisinės valstybės principus bei juos atitinkančias procedūras – rinkimų ir referendumo tvarkos pagrindus, prigimtinių žmogaus teisių ir laisvių apsaugos ir gynimo principus, išimtiniais atvejais -jų ribojimo pagrindus, socialinių (t. y. nevalstybinių) junginių (religinių organizacijų, tautinių bendrijų, politinių partijų ar visuomenės informavimo priemonių) laisvo steigimosi ir veiklos garantijas, vietos savivaldos įgyvendinimo pagrindus. Šiuo atveju konstitucinės teisės šaltiniai yra Konstitucija, Seimo rinkimų įstatymas, Respublikos Prezidento rinkimų įstatymas, Savivaldybių tarybų rinkimų įstatymas, Referendumo įstatymas, Politinių partijų įstatymas, Visuomenės informavimo įstatymas, Vietos savivaldos įstatymas ir šių įstatymų atitiktį Konstitucijai vertinantys Konstitucinio Teismo nutarimai; 2) pagrindines valstybės valdžios funkcijas ir jas vykdančias institucijas, t. y. įstatymų leidybos, valdymo (administravimo) ir teisingumo vykdymo funkcijas bei jas pagal tam tikrus principus ir tvarką vykdančio Seimo, Respublikos Prezidento, Vyriausybės ir teismų (bei kai kurių kitų institucijų, pavyzdžiui, Valstybės kontrolės, Lietuvos banko) teisinės padėties pagrindus. Šiuo atveju konstitucinės teisės materialūs ir formalūs šaltiniai yra Konstitucija, Seimo statutas, Prezidento įstatymas, Vyriausybės įstatymas (iš dalies – ir Vyriausybės darbo reglamentas), Konsti
67
KONSTITUCINĖ TEISĖ
tucinio Teismo įstatymas, Teismų įstatymas ir šių įstatymų atitiktį Konstitucijai vertinantys Konstitucinio Teismo nutarimai; 3) valstybės įgaliojimų ribas, t. y. ne tiek Konstitucijoje tiesiogiai apibrėžtą pagrindinių valstybės institucijų kompetenciją, kiek tokios kompetencijos (pažodžiui Konstitucijos tekste neįvardytas) įgaliojimų ribas, kurios logiškai ir sistemiškai išplaukia iš prigimtinių žmogaus teisių ir laisvių, teisinės valstybės ir kitų pagrindinių principų. Šiuo atveju konstitucinės teisės šaltiniai yra trys: prigimtinės žmogaus teisės ir laisvės, Konstitucija ir įstatymų atitiktį Konstitucijai vertinantys Konstitucinio Teismo nutarimai. Šioje vietoje tikslinga aptarti konstitucinius santykius. Jie gali būti skirstomi į konstituciškai reikšmingus santykius, kurių teisės normos tiesiogiai nereglamentuoja (pvz., Respublikos Prezidento ir didžiausios Seimo narių frakcijos lyderio susitikimas bei kompromisinis susitarimas) ir konstitucinius teisinius santykius, kurie reglamentuoti konstitucinės teisės normų (pvz., Respublikos Prezidento ir Seimo santykiai, susiklostantys skiriant generalinį prokurorą ar Konstitucinio Teismo teisėjus). Konstituciniai santykiai yra: 1) esminiai žmogaus ir valstybės institucijų santykiai, susiję su prigimtinėmis vertybėmis (Konstitucijos 18 str.) ar konstituciniais principais (tokie santykiai susiklosto, pvz., valstybės pareigūnams slapta klausantis asmens pokalbių telefonu, taip pat sulaikant ar suimant žmogų; Konstitucijos 19-26 str.); 2) pagrindinių konstitucinio lygmens valstybės institucijų santykiai (pvz., Seimo ir Respublikos Prezidento santykiai, susiklostantys parenkant ir skiriant Ministrą Pirmininką; Konstitucijos 92 str. 1 d.); 3) kolegialios valstybės institucijos nario ir kitų šios institucijos narių santykiai (pvz., Ministro Pirmininko ir ministrų santykiai arba Seimo narių santykiai svarstant įstatymo projektą; Konstitucijos 69 str., 95 str.); 4) kolegialios valstybės institucijos nario ir šios institucijos (pvz., Seimo nario ir Seimo) santykiai. Šių aptartų santykių dalyvius, turinčius tam tikrą konstitucinį statusą (t. y. konstitucines teises, pareigas, įgaliojimus ar atsakomybę), galima laikyti konstitucinių teisinių santykių subjektais. Įvairiose Konstitucijos nuostatose kalbama apie: kiekvieną individą (žmogų, pilietį ar užsienietį; Konstitucijos 18-26 str., 48 str.), specialųjį individą (pvz., dirbančią motiną, darbuotoją ar „gerai besimokantį pilietį”; Konstitucijos 39 str. 2 d., 41 str. 3 d. 2 s., 51 str.), individų junginius (pvz., šeimą, religinę ar visuomeninę organizaciją, tautinę bendriją, profesinę sąjungą ar politinę partiją; Konstitucijos 35 str., 38 str., 43 str., 45 str.), autonominius darinius, t. y. vadinamąsias personalines korporacijas (valstybės pripažintas bažnyčias, universitetus, visuomenės informavimo priemones; Konstitucijos 40 str. 3 d., 43 str. 2 ir 4 d.), teritorines (vietos) savivaldybes (t. y. žemesniuosius valstybės teritorijos administracinius vienetus; Konstitucijos 11 str., 119 str., 123 str. 1 d.), pagrindines valstybės institucijas ir jų atskirus narius (pvz., Seimą ir jo narius, Vyriausybę ir ministrus; Konstitucijos 55-63 str.; 98-100 str.), aukštesniuosius administracinius vienetus (t. y. apskritis; Žr. Konstitucijos 123 str. 1 d.), valstybinę bendruomenę, svarbiausius gyvenimo klausimus sprendžiančią referendumu (Konstitucijos 9 str.), Lietuvos Respubliką ir jos konstitucinės teisės reglamentuojamus santykius su individu, jų junginiu, užsienio valstybe ar tarptautine organizacija (Konstitucijos 136 str., 138 str.).
68
Konstitucinė teisė ir Konstitucija
KONSTITUCINĖS TEISĖS PADĖTIS TEISINĖJE SISTEMOJE Konstitucinės teisės padėties valstybės teisinėje sistemoje, t. y. jos ryšio su administracine, baudžiamąja, civiline ar kita teisės normų ir principų posisteme, supratimas priklauso nuo to, kaip suvokiama Konstitucijos reikšmė (daugiau apie tai žr. kitą šio skyriaus skirsnį). Jei Konstituciją, be kita ko, suvoktume kaip žmogaus teisių ir laisvių šaltinį (jų tikrasis šaltinis – žmogaus prigimtis, gimimas, o ne Konstitucija), tuo atveju konstitucinė teisė gali atrodyti kaip svarbiausioji teisės šaka. Toks požiūris priimtinas tik su sąlyga, jei valstybės (pvz., buvusios TSRS) įstatymai ir teisės mokslas visos teisinės sistemos neskirsto į viešąją ir privatinę. Lietuvos Respublikoje toks skirstymas egzistuoja. Tai akivaizdžiai rodo naujojo Civilinio kodekso (1.1 str.) nuostatos, taip pat Lietuvos Aukščiausiojo Teismo nutartys, kur, pavyzdžiui, rašoma, kad „privatinėje teisėje turi dominuoti dispozityvus, o ne imperatyvus teisinio reguliavimo metodas. Reali ūkinė laisvė yra įmanoma tik tada, jeigu daugumą privatinės teisės normų sudaro dispozityvios, o ne imperatyvios teisės normos” (Lietuvos Aukščiausiojo Teismo 2000 10 02 nutartis (civilinė byla Nr. 3K-3-905/2000). Turint tai omenyje, galima teigti, kad Lietuvos valstybės konstitucinė teisė nėra absoliučiai svarbiausia ar pirminė teisinės sistemos šaka, nes kitos sistemos šakos nėra absoliučiai išvestinės. Vertinant Konstituciją kaip prigimtinių žmogaus teisių ir laisvių nesukuriantį (tik jas aprašantį ir reikiamą jų apsaugą numatantį) aktą, konstitucinę teisę galima suprasti kaip bazinę teisinės sistemos grandį, kuri ypatinga tuo, kad: 1) netiesiogiai (per konstitucingą įstatymų leidybą ir tiesioginį Konstitucijos taikymą teismų sprendimais) lemia atitinkamas administracinės, baudžiamosios ar civilinės teisės posistemių normų formuluotes ir jų taikymą. Šiuo požiūriu kiekvienas Konstitucinio Teismo sprendimas, kuriuo ginčijamas teisės aktas (Konstitucijos 105 str. 1 d.), reguliuojantis administracinius ar civilinius santykius, pripažįstamas antikonstituciniu, rodo konstitucionalizuotą (Konstitucijos tiesiogiai paveiktą) administracinę ar civilinę teisę; 2) pagal savo principų ir normų (žr., pvz., Konstitucijos 30 str. ar 31 str.) įgyvendinimą, apsaugą ar gynimą yra „priklausoma” nuo kitų teisinės sistemos posistemių. Todėl konstitucingą administracinė ar civilinė teisė atitinkamai rodo „konkretizuotą konstituciją” (konstituciją, pritaikytą konkrečiam atvejui). Konstitucija yra ir konstitucinės teisės, ir kitų teisinės sistemos posistemių šaltinis, nes: 1) joks įstatymas, reguliuojantis administracinius, civilinius ar baudžiamuosius santykius, negali prieštarauti Konstitucijai (Konstitucijos 7 str. 1 d.); 2) Konstitucijoje yra daug nuorodų, įtvirtintų žodžiais „nustato įstatymas” (Konstitucijos 30 str. 2 d.) ar „įstatymo nustatyta tvarka” (Konstitucijos 31 str. 1 d., 84 str. 21 p.), kurias įgyvendinant būtina laikytis principo „Konstitucija yra vientisas aktas” (Konstitucijos 6 str. 1 d.), t. y. atsižvelgti į kitas Konstitucijos nuostatas, kurios yra susijusios su konkretizuojančia nuoroda; 3) prigimtinėms žmogaus teisėms ir laisvėms bei kitoms vertybėms (pvz., nepriklausomybei, demokratijai) veiksmingai įgyvendinti, saugoti ar objektyviai ginti yra būtinos tam tikros (konstitucingos!) administracinės, baudžiamosios ar civilinės teisės normos, kurios įtvirtintos Konstitucijoje žodžiais „saugo įstatymas” (Konstitucijos 19 str., 23 str. 2 d.) ar „gina įstatymas” (Konstitucijos 21 str. 2 d.); 4) Konstitucijoje nustatyti Seimo
69
KONSTITUCINĖ TEISĖ
ir Vyriausybės įgaliojimai, kuriuos įgyvendinant (pvz., pagal minėtą vientisumo principą Seimui nustatant mokesčius, o už jų nemokėjimą ar vengimą mokėti administracines nuobaudas ar kriminalines bausmes; Konstitucijos 127 str.) leidžiamos konstitucingos administracinės, baudžiamosios ar civilinės teisės normos.
1.2. LIETUVOS RESPUBLIKOS KONSTITUCIJOS SAMPRATA IR YPATUMAI
KONSTITUCIJOS RENGIMO IR PRIĖMIMO ISTORIJA BEI PAKEITIMAI Tarptautinės teisės požiūriu Lietuvos valstybė 1940-1990 m. buvo okupuota. Vadinasi, de facto gyvavusi Lietuvos Tarybų Socialistinė Respublika (toliau – LTSR) nebuvo Lietuvos valstybingumo forma (t. y. nereiškė valstybės tęstinumo „socialistiniu pavidalu”). Pagal LTSR rinkimų įstatymą 1990 m. vasarį (ir kovo pradžioje) demokratiškai išrinkta LTSR Aukščiausioji Taryba tų pačių metų kovo 11d. pakeitė savo pavadinimą – pasivadino Lietuvos Respublikos Aukščiausiąja Taryba – ir priėmė Aktą „Dėl Lietuvos nepriklausomos valstybės atstatymo”. Tai darydama Aukščiausioji Taryba (ją galima vadinti Atkuriamuoju Seimu) siekė išvengė konstitucinio vakuumo ir įspūdžio, kad Lietuvos valstybė atsirado iš LTSR. Be to, tą pačią kovo 11 dieną po minėto Akto priėmimo buvo sąmoningai „atstatytas” ir po teisiškai svarbios pauzės („juridinės sekundės”) sustabdytas Lietuvos valstybės 1938 m. Konstitucijos galiojimas, priimtas Lietuvos Respublikos Laikinasis Pagrindinis Įstatymas, paskelbtas įsigaliojančių nuo priėmimo momento. Tokį jo pavadinimą galima suprasti, turint omeny, kad dėl skubaus priėmimo objektyviai neišvengta (be to, turint omeny TSRS neigiamą nusistatymą prieš Lietuvos nepriklausomybę sąmoningai nesistengta visiškai išvengti) daug LTSR 1978 m. Konstitucijos formuluočių ir struktūros ypatybių. Neatsitiktinai Aukščiausioji Taryba kovo 11 dieną priėmė ir protokolinį nutarimą „pavesti […] komisijai per tris savaites parengti pasiūlymus dėl priimto Lietuvos Respublikos Laikinojo Pagrindinio Įstatymo tobulinimo […]”. Visa tai leidžia suprasti, kodėl Laikinasis Pagrindinis Įstatymas, galiojęs iki dabartinės Konstitucijos įsigaliojimo 1992 m. lapkritį, keistas daugiau nei 20 kartų. Naujos (nuolatinės) Konstitucijos projekto kontūrai lėtai ėmė ryškėti laikotarpiu nuo 1991 m. pavasario iki 1992 m. pavasario, kai buvo viešai skelbiami alternatyvūs politinių partijų, darbo grupių ir atskirų ekspertų projektai. Vienas iš pagrindinių projektų skirtumų ir politinių nesutarimų objektų buvo „stipraus” Respublikos Prezidento klausimas, t. y. kolegialaus parlamento (būsimoji Seimo) ir vienasmenio valstybės vadovo galių vadovauti valstybei santykio klausimas. Kompromisiniame Aukščiausiosios Tarybos deputatų projekte, kuris Aukščiausiosios Tarybos nutarimu paskelbtas visuomenei 1992 m. gegužės 1 d. dienraščiuose, buvo įtvirtintos subalansuotos Seimo ir Respublikos Prezidento galios ir nė vienai iš šių institucijų nesuteikta išskirtinė teisė dominuoti. Galutinį projektą Aukščiausioji Taryba patvirtinto ir referendumui pateikė 1992 m. spalio 13 d. nutarimu. Tų pačių metų spalio 25 d. kartu su Seimo rinkimais vykusiame referendume Konstitucijos projektui pritarė daugiau kaip 56 procentai visų rinkėjų.
70
2. PAMATINIAI VALSTYBĖS ORGANIZAVIMO PRINCIPAI
2.1. PAMATINIŲ VALSTYBĖS ORGANIZAVIMO PRINCIPŲ BENDRA SAMPRATA
Pamatiniai valstybės organizavimo (ir funkcionavimo) principai apibendrinti Konstitucijos 1 straipsnyje: „Lietuvos valstybė yra nepriklausoma demokratinė respublika”. Ši nuostata leidžia išskirti nepriklausomos, demokratinės ir respublikinės valstybės organizavimo principus. Be to, preambulės žodžiai „siekdama […] teisinės valstybės” ir šiuos žodžius detalizuojančių nuostatų gausumas, pavyzdžiui, Konstitucijos tiesioginio taikymo ginant individualias teises principas (6 str. 1 ir 2 d.), Konstitucijos viršenybės principas (7 str. 1 d., 105 str.), žmogaus teisių ir laisvių neatimamumo principas (18 str.), teisėjų depolitizacijos ir nepriklausomybės principai (113 str. 2 d., 114 str. 1 d.), rodo, kad pamatinis valstybės organizavimo principas yra ir teisinės valstybės principas. Tokio nusistatymo laikosi ir Konstitucinis Teismas. Iš pamatinių principų kyla atitinkamos visų valstybės institucijų veiklos kryptys ir pareigos, kurios turi būti vykdomos pagal kiekvienos institucijos kompetenciją ir paskirtį. Tuo tarpu iš kitų (nepamatinių, specialiųjų) konstitucinių principų, kurie susiję su tam tikra specifine valstybės veikla (pvz., rūpinimusi žmonių sveikata ar natūralios gamtinės aplinkos apsauga, pagalbos savivaldybėms teikimu; Konstitucijos 53 str. 1 d., 54 str. 1 d., 120 str. 1 d.) kyla tik tam tikrų (ne visų) valstybės institucijų pareigos.
2.2. DEMOKRATINĖS VALSTYBĖS PRINCIPAS
Iš pirmo žvilgsnio demokratija – tai labai aiškus ir paprastas principas, reiškiantis tautos valdžią. Tai rodo Konstitucijos 2 straipsnis: „Lietuvos valstybę kuria Tauta. Suverenitetas priklauso Tautai”. Tačiau LTSR laikotarpis, kai buvo skelbiama liaudies valdžia, o realiai ją turėjo viena partija, verčia abejoti, ar demokratiją sudaro tik vienas elementas, nurodytas dabartinės Konstitucijos 2 straipsnyje. Suvokiant, kad nėra ir negali būti išankstinės ir vieningos tautos valios (ji turi būti suformuota ir išreikšta), galima teigti, kad demokratinė valstybė – tai ne tiek valdžios turėjimo, kiek jos laisvo ir atviro įgyvendinimo proceso principas. Taigi demokratinę valstybę charakterizuoja: 1) demokratiniu referendumu ir rinkimais (Seimo, Respublikos Prezidento bei savivaldybių tarybų rinkimais) ar per demokratiškai išrinktus atstovus (Seimo narius, o plačiau žiūrint – ir per Respublikos Prezidentą; 55 str. 1 d.) įgyvendinama teisinės tautos suvereni valdžia (4 str.). Pažymėtina, kad nors dėl rinkimų ir referendumų organizavimo valstybė patiria didžiulių išlaidų, žmonija iki šiol nėra sugalvojusi veiksmingesnių būdų nei rinkimai ir referendumas demokratiškai išreikšti šimtatūkstantinės (tuo labiau milijoninės) bendruomenės politinei valiai; 2) piliečių politinės teisės, kurias įgyvendinant veiksmingai daroma įtaka formuojant (pirmiausia rinkimų ir referendumo agitacinės kampanijos laikotarpiu) ir
77
KONSTITUCINĖ TEISĖ
išreiškiant (pirmiausia rinkimų ir referendumo balsavimo dieną) teisinės tautos politinę valią. Išskirtinos piliečių rinkimų teisė ir teisė laisvai vienytis į politines partijas (Konstitucijos 34 str., 35 str. 1 d.). Pažymėtinos ir tokios „politinės prigimties” teisės, kurios Konstitucijos tekste pateiktos kaip piliečių, bet įstatymais iš principo gali būti detalizuotos ir kaip Lietuvoje nuolatos gyvenančių užsieniečių teisės. Tokios yra peticijos teisė, teisė laisvai vienytis į visuomenines organizacijas, teisė „lygiomis sąlygomis stoti į […] valstybinę tarnybą” (Konstitucijos 33 str. 1 d.) ir susirinkimų laisvė (Konstitucijos 33 str. 1 ir 3 d., 35 str., 36 str.). Šiuo atveju svarbu, koks yra pilietybės įgijimo ir netekimo reglamentavimas. Ji, pasak Konstitucinio Teismo, yra „nuolatinis asmens politinis teisinis ryšys su konkrečia valstybe, grindžiamas abipusėmis teisėmis bei pareigomis ir iš jų išplaukiančiu savitarpio pasitikėjimu, ištikimybe bei gynyba”. Konstitucija, užsimindama, kad „pilietybė įgyjama gimstant ir kitais įstatymo nustatytais pagrindais” (12 str. 1 d.), įgalioja įstatymų leidėją nustatyti „pilietybės įgijimo ir netekimo tvarką” (12 str. 3 d.). Naujajame Pilietybės įstatyme, priimtame 2002 m., įtvirtinta nuostata, kad užsienietis Lietuvos pilietybę gali įgyti Respublikos Prezidento dekretu tokiais pagrindais: a) turi būti įvykdytos vadinamosios natūralizacijos sąlygos, t. y. žmogus turi nebūti kitos valstybės pilietis ar būti atsisakęs kitos valstybės pilietybės, mokėti lietuvių kalbą, turėti nuolatinę gyvenamąją vietą Lietuvos Respublikos teritorijoje pastaruosius dešimt metų (jei nėra aplinkybių, dėl kurių natūralizacija negalima, pvz., jei pilietybės siekiantis asmuo dalyvavo nusikalstamoje veikloje prieš Lietuvos valstybę); b) išimties tvarka už užsieniečio (bet ne asmens, turėjusio kada nors Lietuvos Respublikos pilietybę) nuopelnus Lietuvos valstybei, Respublikos Prezidentui priėmus dėl to dekretą. Šiuo atveju netaikoma nė viena bendrųjų natūralizacijos sąlygų (pvz., reikalavimas mokėti lietuvių kalbą ar atsisakyti kitos valstybės pilietybės). Tačiau būtina, kad „užsienietis faktiškai integruotųsi į valstybinę bendruomenę”, t. y. nusipelnytų ne tam tikrai institucijai ar įmonei, bet visai valstybei didindamas jos autoritetą (Konstitucinio Teismo 2003 m. gruodžio 30 d. nutarimas). Pažymėtina, kad pilietybės gali būti netenkama Respublikos Prezidento dekretu, jei pilietis „tarnauja kitos valstybės karo tarnyboje arba dirba kitos valstybės tarnyboje” neturėdamas Lietuvos Respublikos Vyriausybės sutikimo; 3) politinių partijų, laisvai keliančių kandidatus, agituojančių už juos ir kitaip padedančių formuoti rinkėjų politinę valią, nevaržomas steigimas ir veikla (Konstitucijos 35 str. 1 d.), lemianti daugiapartinę sistemą, kurioje nėra partijų, pagal tikslus ir veiklą prieštaraujančių Konstitucijai, taigi ir draudimui, kad niekas negali būti verčiamas į jas stoti; 35 str. 1 ir 2 d.). Konstitucijoje nedetalizuojama, kokia konkreti partinė sistema (dvipartinė, tripartinė, penkių ar 30 partijų) turėtų susiformuoti. Todėl galima teigti, kad optimalus ar idealus partijų skaičius – tai ne konstitucinis, bet politologinis klausimas. Teisiniu požiūriu svarbiausia yra tai, kad nė viena partija nebūtų privilegijuota ar „vedanti” (Žr. LTSR 1978 m. Konstitucijos 6 str.), t. y. visos partijos turi būti lygiateisės, laisvai konkuruoti keldamos kandidatus ir agituodamos už juos. Šiuo požiūriu galima suprasti ir pateisinti, kodėl įstatymų leidėjas: a) garantuoja ne vienai partijai, bet keletui partijų dotacijas iš valstybės biudžeto, t. y. įteisina valstybinį partijų finansavimą: „valstybės biudžeto dotacija paskirstoma toms politinėms partijoms ir politinėms organizacijoms, kurios yra gavusios ne mažiau kaip 3 procentus visų rinkėjų balsų, paduotų už […] keltus kandidatus ar jų sąrašus tuose Seimo ir savivaldybių tarybų rinkimuose, pagal kurių
78
Pamatiniai valstybės organizavimo principai
rezultatus skirstoma valstybės biudžeto dotacija” (Politinių partijų ir politinių organizacijų finansavimo įstatymo 13 str. 2 d.); b) nesudaro sąlygų finansiškai „suvalstybinti” partijų, t. y. nustato minėtos dotacijos ribą: „dotacija negali viršyti 0,1 procento valstybės biudžeto išlaidų” (to paties įstatymo 13 str. 2 d.). Pažymėtina ir partijų steigimo tvarka, kurią kiekviena piliečių grupė iš principo yra pajėgi įvykdyti: „Politinei partijai […] įsteigti būtina, kad ji turėtų Lietuvoje ne mažiau kaip tūkstantį steigėjų, kurie savo ar savo atstovų suvažiavime turi patvirtinti statutą (įstatus), programą ir išrinkti vadovaujančius organus” (Politinių partijų įstatymo 2004 m. redakcija). Turint tai omenyje ir atsižvelgiant į tai, kad partijas, kurių nariais gali būti tik aktyviąją rinkimų teisę (taigi 18 metų sukakę) turintys piliečiai, registruoja Teisingumo ministerija, galima pateikti tokį politinės partijos apibrėžimą: Politinė partija – tai įregistruotas, ne mažiau kaip 1000 aktyviąją rinkimų teisę turinčių piliečių jungiantis susivienijimas, siekiantis padėti formuoti ir išreikšti piliečių interesus ir politinę valią bei turintis suvažiavime patvirtintą programą, statutą ir išrinktus vadovaujančius organus; 4) visuomenės informavimo priemonių (Konstitucijoje vadinamų „masinės informacijos priemonėmis”; Konstitucijos 44 str.) laisvė, kuriai užtikrinti skirtos konstitucinės garantijos – draudimai cenzūruoti ir monopolizuoti jas. Tai detalizuota Visuomenės informavimo įstatyme, kuriame nustatyta, kad, pavyzdžiui, ne tik valstybės institucijos (išskyrus mokslo ir mokymo įstaigas), bet ir visi bankai (taigi ir privatūs bankai) negali būti dienraščių, radijo ar televizijos stočių savininkai (bet „gali leisti neperiodinio pobūdžio leidinius, skirtus visuomenei informuoti apie savo veiklą”; Visuomenės informavimo įstatymo 2004 m. redakcija). Verta atkreipti dėmesį, kad įstatymų leidėjas vardan asmens privataus gyvenimo apsaugos nustato universalius draudimus (pvz., draudimą be asmens sutikimo filmuoti ar fotografuoti privačioje valdoje, skelbti informaciją apie privatų gyvenimą be žmogaus sutikimo, išskyrus atvejus, kai informacija padeda atskleisti įstatymų pažeidimus ar nusikaltimus, o kalbant apie politikus ar garsius „visuomeninės veiklos” dalyvius – kai „informacija atskleidžia visuomeninę reikšmę turinčias privataus […] gyvenimo aplinkybes ar asmenines savybes”) ir individualias asmens savigynos galimybes, jei būtų piktnaudžiaujama informacijos laisve (pvz., „atsakymo teisę”, t. y. teisę atsakyti dienraščiui ar televizijos laidai, paneigiant tikrovės neatitinkančią informaciją, teisę reikalauti (prireikus teismine tvarka), kad dienraštis ar televizijos laida atlygintų moralinę žalą (jos maksimalus dydis – 10 tūkstančių litų, o kai teismas nustato, kad žeminanti informacija paskelbta tyčia – 50 tūkstančių, bet ne daugiau kaip 5 procentai konkrečios informavimo priemonės metinių pajamų; Visuomenės informavimo įstatymo 13 str. 3 d., 15 str., 23 str. 6 d. 54 str.); 5) daugumos principas, pagal kurį tiek sprendimai referendumu, tiek Seimo įstatymai, tiek Vyriausybės ar Konstitucinio Teismo nutarimai priimami balsų dauguma. Tą patį galima pasakyti ir apie Respublikos Prezidento rinkimus, kurie vyksta pagal daugumos sistemą: išrinktu laikomas kandidatas, surinkęs daugiausia balsų (žr. Konstitucijos 81 str. 2 d.); 6) valstybinės bendruomenės atstovo laisvas mandatas, pagal kurį kiekvienas Seimo narys, būdamas visos teisinės tautos (bet ne jos dalies, pvz., tam tiktos profesijos ar lyties) atstovas, vadovaujasi (privalėdamas laikytis Konstitucijos ir įstatymų) savo sąžine ir „negali būti varžomas jokių mandatų”, t. y. pažadų ar įsipareigojimų jį rėmusiems ar kitokiems asmenims (Konstitucijos 59 str. 4 d.). Iš šio principo
79
KONSTITUCINĖ TEISĖ
išplaukia Seimo nario galimybė laisvai jungtis (ar nesijungti) į frakcijas, o šioms pasiskelbti opozicinėmis (tai detalizuoja Seimo statutas); 7) Vyriausybės, kuri atsiranda ne per visuotinius rinkimus, atsakomybė Seimui ir šio galimybė pareikalauti, kad Vyriausybė atsiskaitytų už savo veiklą Seime, o esant reikalui – pareikšti jai nepasitikėjimą, šitaip priverčiant atsistatydinti (Konstitucijos 92 str. 5 d., 96 str. 1 d., 101 str. 1 ir 3 d.), taip pat parlamentinis apkaltos procesas, t. y. Seimo galimybė 3/5 visų narių balsų dauguma atimti kiekvieno Seimo nario mandatą ar pašalinti iš pareigų Respublikos Prezidentą, Konstitucinio Teismo, Aukščiausiojo Teismo ir Apeliacinio teismo teisėjus, „šiurkščiai pažeidusius Konstituciją arba sulaužiusius priesaiką, taip pat paaiškėjus, jog padarytas nusikaltimas” (Konstitucijos 74 str.). Seimui taip pat suteikta galimybė pareikšti nepasitikėjimą pareigūnams, kuriuos skiria ar renka jis pats (išskyrus tuos, kurie atleidžiami iš pareigų apkaltos proceso tvarka; Konstitucijos 75 str.); 8) vietos savivalda, kuri sudaro prielaidas decentralizuoti valstybės valdymą ir laisvai plėtotis tam tikros vietovės bendruomenės (vietos gyventojų, kurie iš esmės yra savivaldos subjektas) kultūriniams ir ūkiniams interesams, tradicijoms. Turint omenyje, kad LTSR laikotarpiu vadinamosios vietinės tarybos buvo „vieningos valstybinės valdžios organų sistemos” grandis (LTSR 1978 m. Konstitucijos 78 str.), pažymėtina, kad dabar pagal Konstituciją (119 str.) vietos savivalda ir ją pirmiausia išreiškiančios institucijos (savivaldybių tarybos) yra susijusios ne su „valstybės valdžios įgyvendinimu” (tai įgyvendina Seimas, Respublikos Prezidentas, Vyriausybė ir teismai; Konstitucijos 5 str. 1 d.), bet su specifinių savivaldos, kylančios iš vietos bendruomenės, galių įgyvendinimu. Todėl galima daryti dvi svarbias išvadas: a) savivaldybės institucijos iš principo nėra valstybės institucijos ir todėl nėra tiesiogiai pavaldžios Vyriausybei ar Seimui. Šiuo požiūriu nuostata „valstybė remia savivaldybes” (Konstitucijos 120 str. 1 d.) pasakoma, kad valstybė remia tai, kas nėra jos institucijos. Aišku, savivaldybės – tai valstybės teritorijos dalys, sukuriamos įstatymu (11 str.). Todėl Konstitucinis Teismas pagrįstai aiškina, kad Konstitucijoje yra įtvirtintas savivaldybių ir valstybės interesų derinimo principas; b) vietos savivalda, kaip valdymo forma, iš principo yra savarankiška viešojo valdymo grandis (žr. Konstitucijos 120 str. 2 d.) ir todėl nesutampa su vietiniu valstybės valdymu, kuris pasireiškia aukštesniuose administraciniuose vienetuose, įstatyme vadinamuose apskritimis (o jei valstybės valdymo institucijų teritoriniai padaliniai veikia savivaldybėse – ir savivaldybių lygmeniu). Taigi visa tai turint omenyje suprantama, kodėl Konstitucinis Teismas išaiškino, kad jei įstatymais savivaldybėms perduodamos valstybės funkcijos arba kitais teisės aktais (pvz., Vyriausybės nutarimais) sukuriamos naujos savivaldybių pareigos, turi būti numatomos ir šioms funkcijoms ar pareigoms reikalingos lėšos (Konstitucinio Teismo 2002 m. gruodžio 24 d. nutarimas). Šiame kontekste pažymėtina, kad Konstitucijoje įvardyta tik viena – pagrindinė ir atstovaujamoji (sprendžiamoji) – savivaldybės institucija – taryba. Tuo tarpu kitas vykdomąsias – institucijas, jų santykius gali savo nuožiūra nustatyti įstatymų leidėjas (119 str. 3 d.). Jis, pavyzdžiui, gali numatyti merą, burmistrą ar kitokį savivaldybės pareigūną ar kolegialų organą (valdybą ar magistratą) kaip „vykdomuosius organus”, kuriuos turi sudaryti taryba (119 str. 4 d.). Konstitucinis Teismas 2002 m. gruodžio 24 d. nutarime išaiškino, kad taryba pagal įstatymą negali turėti teisės sudaryti vykdomuosius organus (pvz., merą ir valdybą) iš savo narių, nes vykdomieji organai yra atskaitingi tarybai (119 str. 4 d.). Be to, Konstitucinio
80
Pamatiniai valstybės organizavimo principai
Teismo minėtame nutarime išaiškinta, kad tas pats asmuo vienu metu negali būti savivaldybės tarybos ir Seimo narys. Jei Seimo narys dalyvauja ir gauna mandatą savivaldybės tarybos rinkimuose, jis iki pirmojo naujai išrinktos savivaldybės tarybos posėdžio turi apsispręsti, kurioje institucijoje (tarybos ar Seimo) vykdys atstovo veiklą. Atsižvelgdamas į tokius išaiškinimus, įstatymų leidėjas 2003 m. nustatė naują savivaldybės institucijų sistemą, atsisakydamas mero kaip vienasmenės vykdomosios institucijos, valdybos kaip kolegialios vykdomosios institucijos, ir įtvirtino naujas nuostatas: a) meras vykdo tarybos pirmininko funkcijas pirmininkaudamas tarybos posėdžiams ir atstovaudamas jai (nebūdamas atskira, savarankiška vykdomąja institucija) ir yra renkamas tarybos iš jos narių; b) savivaldybės administracijos direktorius yra savivaldybės vykdomoji institucija ir „į pareigas skiriamas mero siūlymu savivaldybės tarybos sprendimu tarybos įgaliojimų laikui politinio (asmeninio) pasitikėjimo pagrindu” (Vietos savivaldos įstatymo 29 str.). Kalbant apie vietos savivaldą, verta pažymėti, kad Konstitucijoje (123 str. 4 d.) numatyta, jog Seimas savivaldybės teritorijoje gali laikinai įvesti tiesioginį valdymą (logiškai mąstant – valstybinį, vyriausybinį administravimą, kuris pagal savo esmę ir pobūdį nesiskiria nuo apskrities valdymo). Tokia nuostata pagal Konstitucijos vientisumo principą (6 str. 1 d.) neprieštarauja vietos savivaldos ir kartu demokratinės valstybės principams, nes: 1) tiesioginis savivaldybės valdymas gali būti tik laikinas, nepaneigiantis savivaldybės tarybos kadencijos reglamentavimo Konstitucijoje prasmės; 2) tik Seimas (bet ne Vyriausybė ar kita valdymo funkciją vykdanti institucija) gali įvesti minėtą valdymą. Be to, įstatymų leidėjas nustatė tiesioginio valdymo įvedimo pagrindus (pvz., jei savivaldybės taryba kėsinasi į valstybės teritorijos vientisumą ir konstitucinę santvarką, jei du metų ketvirčius iš eilės nevyksta tarybos posėdžiai, jei naujai išrinkta taryba per du mėnesius nuo pirmojo posėdžio sušaukimo dienos neišrenka mero; 2003 m. Laikino tiesioginio valdymo savivaldybės teritorijoje įstatymo 2 str.), kurių pagrįstumas iš principo nekelia abejonių, tuo labiau, kad pagal įstatymą Seimas gali, bet neprivalo įvesti minėtą valdymą iki 6 mėnesių Vyriausybės siūlymu, o jei nusprendžia tai daryti – privalo nustatyti naujų rinkimų į savivaldybės tarybą rinkimų datą (Laikino tiesioginio valdymo savivaldybės teritorijoje įstatymo 2 str. 4-7 d.). Detalizuojant demokratiškai tiesioginiu ar atstovaujamuoju būdu įgyvendinamą valstybinės bendruomenės suverenitetą – vienintelį valstybinės valdžios pateisinimo (legitimacijos) šaltinį (Konstitucijos 4 str.), pažymėtina, kad pagal Konstituciją tiesioginis tautos suvereniteto įgyvendinimas pirmiausia pasireiškia referendumu (9 str., 69 str. 4 d., 148 str.) ir rinkimais (55 str., 78 str., 119 str. 2 d.). Kalbant apie referendumą, pažymėtina, kad jis bus demokratinis, jei jo metu galios tiesioginių, visuotinių ir lygių rinkimų bei slapto balsavimo principai. Pagal Konstituciją (9 str.) referendumą skelbia Seimas savo iniciatyva arba kai to reikalauja ne mažiau kaip 300 tūkstančių rinkimų teisę turinčių piliečių. Referendumo įstatyme (11 str. 5 d.) nustatyta, kad piliečių parašus, kuriais patvirtinamas pritarimas konkretaus referendumo paskelbimo idėjai, ne mažiau kaip 15 piliečių iniciatyvinė grupė, įregistruota Vyriausiosios rinkimų komisijos, turi surinkti per tris mėnesius. Tuo atveju, jei referendumo idėją siūlo Seimo nariai, jų iniciatyvinė grupė turi būti ne mažesnė kaip 1/4 visų Seimo narių (Referendumo įstatymo 9 str. 3 d.). Pažymėtina, kad pagal Referendumo įstatymą skiriami privalomieji ir konsultaciniai referendumai. Privalomieji referendumai turi būti besąlygiškai rengiami, kai
81
KONSTITUCINĖ TEISĖ
sprendžiami klausimai dėl Konstitucijos 1 straipsnio (šiuo atveju sprendimas priimamas, jei tam pritaria ne mažiau kaip 3/4 piliečių, įrašytų į rinkėjų sąrašus) ir kitų I skirsnio bei XIV skirsnio nuostatų (šiais atvejais sprendimas priimamas, jei tam pritaria ne mažiau kaip pusė piliečių, įrašytų į rinkėjų sąrašus), Konstitucijos sudedamosios dalies – konstitucinio akto „Dėl Lietuvos Respublikos nesijungimo į postsovietines Rytų sąjungas” (šiuo atveju sprendimas priimamas, jei tam pritaria ne mažiau kaip 3/4 piliečių, įrašytų į rinkėjų sąrašus) keitimo ir Lietuvos valstybės dalyvavimo tarptautinėse organizacijose, jeigu „šis dalyvavimas yra susijęs su Lietuvos valstybės organų kompetencijos daliniu perdavimu tarptautinių organizacijų institucijoms ar jų jurisdikcijai” (šiuo atveju sprendimas priimamas, jei tam pritaria daugiau kaip pusė referendume dalyvavusių piliečių). Be to, privalomasis referendumas gali būti rengiamas, jei Seimas ar 300 tūkstančių rinkimų teisę turinčių piliečių siūlo spręsti kitą klausimą, susijusį su įstatymų ar jų nuostatų priėmimu (Referendumo įstatymo 4 str. 1 ir 2 d., Žr. Konstitucijos 69 str. 4 d.). Galima priminti, kad 2003 m. gegužės 10-11 dienomis vykusiame privalomajame referendume dėl Lietuvos Respublikos narystės Europos Sąjungoje dalyvavo daugiau kaip pusė (63,3 procento) piliečių, turinčių rinkimų teisę ir įrašytų į rinkėjų sąrašus, o už klausimo sprendimo tekstą „Pritariu Lietuvos Respublikos narystei Europos Sąjungoje” balsavo daugiau kaip pusė (91,07 procento) referendume dalyvavusių piliečių. Konsultaciniai referendumai, kurių daugiau kaip pusės piliečių (įrašytų į rinkėjų sąrašus) balsais priimtas sprendimas įpareigoja Seimą nagrinėti jo įgyvendinimą, gali būti rengiami kitais valstybei ir visuomenei svarbiais klausimais, jei tai inicijuoja 300 tūkstančių rinkimų teisę turinčių piliečių ar Seimas, ir dėl to nebūtina rengti privalomojo referendumo (Referendumo įstatymo 5, 8 str.). Detalizuojant demokratinius ir reguliarius (kas ketveri metai vykstančius) Seimo ir (kas penkeri metai rengiamus) Respublikos Prezidento bei (kas ketveri metai organizuojamus) savivaldybių tarybų rinkimus, pažymėtina, kad jų konstituciniai principai yra vienodi: visuotinė, tiesioginė ir lygi rinkimų teisė bei slaptas balsavimas (Konstitucijos 55 str. 1 d., 78 str. 2 d., 119 str. 2 d.). Visuotinės rinkimų teisės principas apima visų veiksnių piliečių (o savivaldybių tarybų rinkimuose pagal Konstitucijos 119 straipsnio 2 dalies 2002 m. pakeitimą -ir kitų nuolatinių savivaldybės gyventojų), kurie rinkimų dieną yra sukakę 18 metų, galimybę rinkti tam tikra tvarka registruotus kandidatus (t. y. įgyvendinti aktyviąją rinkimų teisę) ir visų tų veiksnių piliečių, kurie yra sukakę Konstitucijos ar įstatymų nustatyto amžiaus (Seimo rinkimuose – 25 metų, Respublikos Prezidento – 40, savivaldybių tarybų – 20) bei atitinka kitus reikalavimus (pvz., sėslumo), galimybę būti renkamais kandidatais (t. y. įgyvendinti pasyviąją rinkimų teisę). Tiesioginės rinkimų teisės principas rodo, kad atstovus rinkėjai renka be tarpininkų. Lygios rinkimų teisės principas rodo tai, kad kiekvienas rinkėjo balsas (ir vienas „sąrašo balsas”, t. y. balsas už partijos pateiktą kandidatų sąrašą Seimo ir savivaldybių tarybų rinkimuose, ir vadinamieji trys „pirmumo balsai”, t. y. balsai už bet kurių trijų kandidatų norimą eiliškumą tame sąraše, už kurį atiduodamas „sąrašo balsas”) turi vienodą vertę skaičiuojant balsus ir nustatant galutinius rinkimų rezultatus (tokius pirmumo balsus, atiduodamus balsuojant už kandidatų sąrašą, numato Seimo rinkimų įstatymas ir Savivaldybių tarybų rinkimų įstatymas).
82
Pamatiniai valstybės organizavimo principai
Pažymėtina, kad Konstitucijoje nustatyta konkreti Respublikos Prezidento rinkimų sistema (t. y. daugumos balsų sistema – išrinktu laikomas arba daugiau kaip pusę visų balsų pirmajame rate surinkęs, arba daugiausia balsų antrajame rate surinkęs kandidatas; žr. Konstitucijos 81 str.), tuo tarpu Seimo ir savivaldybių tarybų rinkimų sistemos nenustatytos. Įstatymų leidėjas šiuo atveju pasirinko: 1) mišrią (iš dalies proporcinę barjerinę, iš dalies daugumos balsų sistemą) Seimo rinkimų sistemą, pagal kurią iš 141 Seimo nario 71 Seimo narys renkamas 71 vienmandatėje rinkimų apygardoje pagal daugumos sistemą (tai reiškia, kad išrinktu laikomas kandidatas, gavęs daugiausia rinkėjų balsų; pažymėtina, kad šiose apygardose kandidatus gali kelti partijos, o kiekvienas pilietis, turintis pasyviąją rinkimų teisę, gali iškelti ir įregistruoti savo kandidatūrą, jei tai parašais paremia ne mažiau kaip vienas tūkstantis tos apygardos rinkėjų), o 70 Seimo narių – vienoje daugiamandatėje rinkimų apygardoje (ji apima visą valstybės teritoriją) pagal proporcinę barjerinę sistemą, pagal kurią skirstant mandatus (proporcingai gautiems rinkėjų balsams) dalyvauja tik tie sąrašai, kurie įveikė vadinamąjį rinkimų barjerą, t. y. gavo ne mažiau nei 5 procentus visų rinkimuose dalyvavusių rinkėjų balsų (jei dvi ar daugiau partijų sudarė bendrą – koalicinį – kandidatų sąrašą, – ne mažiau nei 7 procentus; 2) proporcinę barjerinę savivaldybių tarybų rinkimų sistemą, pagal kurią kiekvienoje savivaldybėje sudaroma daugiamandatė rinkimų apygarda, kurioje kandidatai renkami tik pagal partijų pateiktus kandidatų sąrašus (t. y. atskiras pilietis, turintis pasyviąją rinkimų teisę, neturi galimybės iškelti savo kandidatūros), o skirstant mandatus (proporcingai gautiems rinkėjų balsams) dalyvauja tik tie sąrašai, kurie įveikė rinkimų barjerą, t. y. gavo ne mažiau nei 4 procentus visų rinkimuose dalyvavusių rinkėjų balsų (jei dvi ar daugiau partijų sudarė bendrą – koalicinį kandidatų sąrašą, -ne mažiau nei 6 procentus. Kalbant apie rinkimus, svarbu aptarti Vyriausiąją rinkimų komisiją, kurią pagal Konstituciją (67 str. 13 p.) sudaro Seimas. Kadangi ši komisija tiesiogiai paminėta Konstitucijoje, galima teigti, kad tik ji gali centralizuotai organizuoti tiek Seimo, tiek Respublikos Prezidento ar savivaldybių tarybų rinkimus, taip pat referendumus. Tai išaiškino Konstitucinis Teismas, vertindamas specialiai Respublikos Prezidento rinkimams 1993 m. pagal tam tikrą įstatymą sudarytą „Respublikos Prezidento rinkimų komisiją”, kuri pavadinta „aukščiausia Respublikos Prezidento rinkimų organizavimo institucija” (Konstitucinio Teismo 1994 m. liepos 11d. nutarimas). Vyriausiosios rinkimų komisijos teisinė padėtis detalizuota 2002 m. priimtame Vyriausiosios rinkimų komisijos įstatyme. Pagal šį įstatymą minėtoji komisija yra nuolatinė ir nepriklausoma institucija, kurią ketveriems metams sudaro Seimas iš: 1) pirmininko; 2) trijų aukštąjį universitetinį teisinį išsilavinimą turinčių asmenų, kurie burtais nustatomi iš teisingumo ministro pasiūlytų šešių kandidatų; 3) trijų aukštąjį universitetinį teisinį išsilavinimą turinčių asmenų, kurie burtais nustatomi iš Lietuvos teisininkų draugijos pasiūlytų šešių kandidatų; 4) partijų, gavusių Seimo narių mandatų daugiamandatėje rinkimų apygardoje, pasiūlytų asmenų (minėtos partijos turi teisę pasiūlyti po vieną asmenį). Pažymėtina, kad jei kandidatai atitinka reikalavimus (yra piliečiai, pagal Seimo rinkimų įstatymą gali tapti kandidatais į Seimo narius, per paskutinius trejus Seimo, Respublikos Prezidento, savivaldybių tarybų rinkimus ar referendumus nebuvo atleisti iš rinkimų ar referendumo komisijos už įstatymų pažeidimą), tai „Seimas negali pasiūlytų kan
83
KONSTITUCINĖ TEISĖ
didatų atmesti”. Svarbu ir tai, kad asmenys, paskirti į komisiją iš teisingumo ministro ir Lietuvos teisininkų draugijos pasiūlytų kandidatų, „turi sudaryti daugiau kaip 1/2 Vyriausiosios rinkimų komisijos narių” (minėto įstatymo 6 str., 7 str.).
2.3. RESPUBLIKINĖS VALSTYBĖS PRINCIPAS
Savaime aišku, kad respublika – tai ne monarchija. Tačiau kas ji iš esmės yra? Respublika, kaip valstybės valdymo forma, dar nebuvo numatyta pirmųjų laikinųjų (1918 ir 1919 m.) konstitucijų preambulėse. Ten rašoma: „Iki Steigiamasis (kuriamasis) bus nusprendęs Lietuvos valdymo formą […]”. Pirmą kartą jį įvardyta 1920 m. Konstitucijos nuostatoje „Lietuvos valstybė yra […] respublika” (1 str.). Sisteminiu požiūriu respublika – tai kelių valstybės valdžią vykdančių institucijų bendro valstybės valdymo forma. Ją charakterizuoja: 1) demokratiniai valstybės vadovo (Respublikos Prezidento) rinkimai, kurių nevaržo reikalavimai, kad kandidatai turėtų aristokratinę kilmę, turtą ar išsilavinimą (žr. Konstitucijos 78 str. 1 d.); 2) valstybės vadovo reguliarus perrinkimas kas penkeri metai (Konstitucijos 78 str.); 3) kompetencija, ribojama konstitucinių įgaliojimų sąrašo, ir pagal tokį sąrašą priimamų kai kurių dekretų kontrasignavimo (Ministro Pirmininko ar atitinkamos srities ministro parašas lemia keturių rūšių Respublikos Prezidento pasirašytų dekretų galiojimą, t. y. dekretų dėl diplomatinių atstovų užsienio valstybėse skyrimo ir atšaukimo, aukščiausių karinių laipsnių suteikimo, nepaprastosios padėties paskelbimo ir pilietybės suteikimo; Konstitucijos 85 str.); 4) konstitucinė valstybės vadovo pareiga daryti viską, kas pavesta konstitucingų įstatymų (Konstitucijos 77 str. 2 d.). Šiuo požiūriu Seimas, kaip kolegiali teisinės tautos atstovybė, gali įstatymu pavesti Respublikos Prezidentui papildomai daryti tai, kas atitinka jo konstitucinių įgaliojimų pobūdį; 5) valstybės vadovo atsakomybė, kuri gali pasireikšti arba pašalinimu iš pareigų parlamentinio apkaltos proceso tvarka (Konstitucijos 74 str.) arba patraukimu atitinkamon (pvz., administracinėn ar baudžiamojon) teisinėn atsakomybėn bendrąja tvarka, kai baigs eiti savo pareigas (Konstitucijos 86 str. 1 d.).
2.4. NEPRIKLAUSOMOS VALSTYBĖS PRINCIPAS
Kitaip nei 1990-1991 m., kai atkurtos nepriklausomos Lietuvos valstybės nesiryžo (dėl gerų santykių su tuometine TSRS ir Rusijos Federacija) de jure pripažinti dauguma Vakarų Europos valstybių, o žodis „nepriklausomybė” Lietuvoje akivaizdžiai reiškė nepriklausomybę nuo TSRS, dabar sunku trumpai ir aiškiai pasakyti, ką reiškia įvairias tarptautines sutartis ratifikavusios ir Europos žmogaus teisių teismo jurisdikciją pripažįstančios Lietuvos nepriklausomybė, tuo labiau, kad jau kalbama apie nepriklausomą Lietuvą kaip Europos Sąjungos, NATO ar kitos tarptautinės organizacijos narę. Logiškai ir sistemiškai mąstant galima teigti, kad nepriklausomybė, arba suverenitetas, reiškia, kad valstybė turi išimtinę teisę savo teritorijoje, tiksliau teisinį ir politinį savarankiškumą vykdyti pagrindines valstybės valdžios funkcijas (įstatymų leidybą, valdymą ir teisingumo vykdymą), jei tuo nėra pažeidžiamos prigimtinės žmogaus teisės ir laisvės (Konstitucijos 18 str.) ir vadovaujamasi „visuotinai pripa
84
Pamatiniai valstybės organizavimo principai
žintais tarptautinės teisės principais ir normomis” (Konstitucijos 135 str. 1 d.). Be to, nepriklausomybę išreiškia ir valstybinės bendruomenės (teisinės tautos) suverenitetas (Konstitucijos 2 str.), demokratiškai įgyvendinamas tiesiogiai ar per Seimo narius bei Respublikos Prezidentą (Konstitucijos 4 str.), o platesniu požiūriu – ir per Vyriausybę, taip pat teismus (Konstitucijos 5 str. 1 d.). Nepriklausoma valstybė neišnyksta, jei jos bendruomenė demokratiškai – tiek tiesiogiai referendumu, tiek netiesiogiai per Seimą, Respublikos Prezidentą ir Vyriausybę – pritaria, kad valstybė įsijungtų į Europos Sąjungą deleguodama jai dalį savo kompetencijos, kuri bus vykdoma kartu su kitomis šios Sąjungos narėmis. Aišku, toks delegavimas, kad būtų aiškiau, turėtų būti tiesiogiai numatytas Konstitucijoje, todėl būtina atitinkamai pakeisti (ar plečiamai aiškinti) Konstituciją, pavyzdžiui, jos 136 straipsnį. Siame straipsnyje pernelyg abstrakčiai numatytas Lietuvos valstybės dalyvavimas tarptautinėse organizacijose, nustatytas reikalavimas „jeigu tai neprieštarauja valstybės interesams ir jos nepriklausomybei”. Minėta, kad Seimas 2004 m. apsisprendė papildyti Konstitucijos 150 straipsnį, nustatantį, kad sudedamąja Konstitucijos dalimi laikomas ir konstitucinis aktas „Dėl Lietuvos Respublikos narystės Europos Sąjungoje”. Šioje vietoje svarbu prisiminti Konstitucijos sudedamąją dalį – konstitucinį aktą „Dėl Lietuvos Respublikos nesijungimo į postsovietines Rytų sąjungas” (Konstitucijos 150 str.). Jame, be kita ko, numatyta, kad „Lietuvos Respublikos teritorijoje negali būti jokių Rusijos […] karinių bazių ir kariuomenės dalinių”. Nesileidžiant į neteisines diskusijas, kiek kareivių, amunicijos ar ginkluotės turėtų apimti žodžiai „kariuomenės dalinys”, galima teigti, kad šiame akte nedraudžiama įstatymų leidėjui nustatyti, kad į Lietuvą laikinai tarptautinėms pratyboms gali atvykti tam tikro gausumo Rusijos ar kitos valstybės, esančios „postsovietinėse Rytų Sąjungose”, kareivių grupė (tai numato Tarptautinių operacijų, karinių pratybų ir kitų renginių įstatymas, pakeistas 2002 m. sausio 15 d.).
2.5. TEISINĖS VALSTYBĖS PRINCIPAS
Abstraktūs Konstitucijos preambulės žodžiai „siekdama teisinės valstybės” įvairiuose Konstitucijos straipsniuose yra sukonkretinami (5 str., 7 str., 18 str., 30 str., 105 str.). Jų vientisas (logiškai ir sistemiškai išplaukiantis) ryšys leidžia teigti, kad teisinė valstybė – tai pagal Konstituciją, konstitucingus įstatymus ir valdymo aktus bei teismų sprendimus gyvuojanti valstybė. Šiuo požiūriu teisinę valstybę charakterizuoja: 1) prigimtinių žmogaus teisių ir laisvių apsauga ir gynyba (Konstitucijos 18 str.). Esant neišvengiamai būtinybei, tos teisės ir laisvės ribojamos įstatymo nustatyta tvarka, t. y. remiantis proporcingo ribojimo principu. Tai reiškia, kad, viena vertus, ribojimu siekiamas tikslas turi atitikti ribojimo priemones (pvz., suėmimą, priverstinį gydymą, visuomeninės organizacijos ar partijos veiklos uždraudimą ar veiklos sustabdymą), kita vertus, teisės pažeidimų pobūdis ir pavojingumas turi atitikti už tokius pažeidimus įstatymu nustatomas sankcijas (pvz., baudas ar bausmes; daugiau apie visa tai – kitame skirsnyje); 2) asmenų teisinės lygybės (lygiateisiškumo) įstatymui, teismui ir kitoms valstybės institucijoms principas (Konstitucijos 29 str. 1 d.). Jį Konstitucinis Teismas yra praplėtęs įvairiais teiginiais, pavyzdžiui: „lygybės įstatymui principas reiškia „lygų
85
KONSTITUCINĖ TEISĖ
matą”, kai reikia tą pačią normą taikyti skirtingiems asmenims”; principas įpareigoja „vienodus faktus teisiškai vertinti vienodai, ir draudžia iš esmės tokius pat faktus savavališkai vertinti skirtingai”; principas reikalauja, kad „teisėje pagrindinės teisės ir pareigos būtų įtvirtintos vienodai” (Konstitucinio Teismo 1996 m. sausio 24 d., 2000 m. birželio 30 d. ir 2002 m. spalio 23 d. nutarimai). Iš viso to kyla draudimas varžyti žmogaus teises ar „teikti jam privilegijas dėl jo lyties, rasės, tautybės, kalbos, kilmės, socialinės padėties, tikėjimo, įsitikinimų […]” (Konstitucijos 29 str. 2 d.). Toks draudimas iš esmės yra asmenų nediskriminavimo ir neprivilegijavimo pagal minėtus kriterijus principas, kuris gali būti plečiamas, teisiškai įvardijant ir kitus diskriminacinio (savivališko ir neobjektyvaus) vertinimo kriterijus. Šiuo požiūriu galima pateisinti normą, kad „visi nuteistieji lygūs nepaisant jų […] genetinių savybių, neįgalumo, seksualinės orientacijos, veiklos rūšies ir pobūdžio, gyvenamosios vietos ir kitų nenumatytų Lietuvos Respublikos įstatymuose ypatybių” (Teisingumo ministro 2003 m. birželio 18 d. įsakymu patvirtintų Areštinių vidaus tvarkos taisyklių 2 p.). Šiame kontekste pažymėtinas asmenų teisinio saugumo principas, pagal kurį, viena vertus, galioja tik paskelbti įstatymai (Konstitucijos 7 str. 2 d.), kita vertus, įstatymai galioja į ateitį (t. y. negalioja atgaline data). Konstitucinis Teismas išaiškino, kad „vienas iš teisinės valstybės reikalavimų yra tas, jog teisės aktų galia turi būti nukreipiama į ateitį. Teisės aktuose įtvirtintos normos taikomos faktams ir pasekmėms, atsirandantiems po jų įsigaliojimo” (Konstitucinio Teismo 2001 m. lapkričio 29 d. nutarimas). Tiesa, teisinėje valstybėje galioja išimtis, kad „priimti įstatymai, panaikinantys veikos baudžiamumą ar sušvelninantys atsakomybę, turi grįžtamąją galią” (Konstitucinio Teismo 2001 m. sausio 11d. nutarimas). Be to, pažymėtina, kad asmenų teisinio saugumo principas apima asmens teisėtų lūkesčių principą. Jis pagal Konstitucinio Teismo išaiškinimus reiškia, viena vertus, visų valstybės institucijų pareigą laikytis prisiimtų įsipareigojimų (pvz., mokėti tam tikriems asmenims įstatymu ar valdymo aktu nustatytą pensiją ar darbo užmokestį), kita vertus, asmens teisėtai įgytų teisių (pvz., teisių gauti tam tikrą pensiją ar darbo užmokestį) apsaugą („asmenys turi teisę pagrįstai tikėtis, kad jų pagal galiojančius teisės aktus įgytos teisės bus išlaikytos nustatytą laiką ir galės būti realiai įgyvendinamos” (Konstitucinio Teismo 2001 m. gruodžio 18 d. nutarimas). Pavyzdžiui, jei kiekvienas naujos kadencijos Seimas keičia prokurorų skyrimo ir atleidimo tvarką ir todėl nė vienas paskirtas generalinis prokuroras neišbūna savo pareigų visą įstatymo nustatytą įgaliojimų laiką, tuo ne tik sudaromos antikonstitucinės prielaidos „atsirasti teisiniam netikrumui prokuratūros sistemoje”, bet ir pažeidžiamas asmens (šiuo atveju generalinio prokuroro) teisėtų lūkesčių (eiti pareigas visą įstatymo nustatytą laiką, jei nepadaromas nusikalstamas teisės pažeidimas) principas […]. Įstatymų leidėjas, nustatęs generalinio prokuroro įgaliojimų trukmę, neturi teisės nustatyti bet kokių generalinio prokuroro atleidimo iš pareigų nepasibaigus įgaliojimų terminui pagrindų” (Konstitucinio Teismo 2003 m. sausio 24 d. nutarimas). Toks išaiškinimas jau ir konstitucinės teisės ir teisės istorijos dalis, nes Seimas 2003 m. pakeitė Konstituciją, tiesiogiai nustatydamas generalinio prokuroro skyrimo ir atleidimo tvarką (84 str. 11 p., 118 str. 5 d.);
3) valdžių – įstatymų leidžiamosios, vykdomosios ir teisminės – padalijimo principas (kitaip tariant, pagrindinių valstybės valdžios funkcijų ir jas vykdančių pagrindinių valstybės institucijų – Seimo, Respublikos Prezidento, Vyriausybės ir teismų – kompetencijos atribojimo principas; Konstitucijos 67 str., 84 str., 94 str.,
86
Pamatiniai valstybės organizavimo principai
109 str.), pagal kurį, viena vertus, nė viena valstybės institucija nėra „aukščiausiasis valstybinės valdžios organas” (LTSR 1978 m. Konstitucijos 97 str. 1 d.), kita vertus, nė viena valstybės valdžią (pagal Konstitucijos 5 str. 1 d.) įgyvendinanti institucija negali atsisakyti savo įgaliojimų, perimti kitos institucijos įgaliojimus ar perleisti savo įgaliojimus kitai institucijai. Visa tai daug kartų yra akcentavęs Konstitucinis Teismas; 4) mažumų – tiek opozicinių Seimo narių grupių ar frakcijų, tiek piliečių tautinių bendrijų (Konstitucijos 45 str.) ar asmenų religinių organizacijų (43 str.) – apsaugos principas, kurio negali paneigti minėtas demokratijos elementas – daugumos principas. Šiuo požiūriu vienas iš demokratinės ir teisinės valstybės principų bendrumų – daugumos ir mažumos interesų derinimas; 5) asmens galimybė kreiptis į teismą (Konstitucijos 30 str. 1 d.), t. y. į nepriklausomą ir nešališką teismą” (Konstitucijos 31 str. 2 d.). Ši galimybė yra absoliuti, t. y. įstatymų leidėjas neturi įgaliojimo nustatyti, kad sprendimas yra neskundžiamas teismui. Akcentuojama ir asmens teisė reikalauti, kad jam teismine tvarka būtų atlyginta padaryta materialinė ir moralinė žala. Anot Konstitucinio Teismo, „net ir nesant atitinkamo įstatyminio reguliavimo, asmuo gali reikalauti jam atlyginti neteisėtais valstybės institucijų ar pareigūnų veiksmais padarytą žalą tiesiogiai remdamasis Konstitucijos 30 straipsnio 2 dalimi bei kitais Konstitucijos straipsniais, kuriuose nustatytos atitinkamų žmogaus teisių ir laisvių apsaugos ir gynimo garantijos” (Konstitucinio Teismo 2000 m. birželio 30 d. nutarimas). Be to, svarbi ir Teismų įstatymo (5 str.) nuostata, kad asmuo turi teisę į „operatyvų teismą”, t. y. „teismas visa savo veikla turi užtikrinti, kad bylos būtų išnagrinėtos […] per įmanomai trumpiausią laiką”; 6) teisėjų ir teismų vykdančių teisingumą, nepriklausomumo principas (Konstitucijos 109 str. 2 d.). Konstitucijoje numatytas dvejopas nepriklausomumas: a) individualus teisėjų nepriklausomumas. Jį, anot Konstitucinio Teismo, lemia tai, kad: pirma, teisėjų įgaliojimų trukmė neliečiama. Ją riboja bendra taisyklė: „Lietuvos Respublikos teismų teisėjai atleidžiami iš pareigų […] sulaukę įstatyme nustatyto pensinio amžiaus” (Konstitucijos 115 str. 2 p.); antra, draudžiama teisėją suimti ar patraukti baudžiamojon atsakomybėn be Seimo ar Respublikos Prezidento sutikimo (žr. Konstitucijos 114 str. 2 d.). Tai šiek tiek detalizuoja Teismų įstatymo (47 str.) nuostata: „Draudžiama įeiti į teisėjo gyvenamąsias ar tarnybines patalpas, daryti ten arba teisėjo asmeniniame ar tarnybiniame automobilyje […] apžiūrą, kratą ar poėmį”; trečia, teisėjui, gaunančiam teisėjo atlyginimą (Konstitucijos 113 str. 1 d.), negalima įstatymais ar kitais teisės aktais mažinti darbo užmokesčio (Konstitucinio Teismo 1999 m. gruodžio 21 d. nutarimas). Šiame kontekste pažymėtinos Teismų įstatymo (1 str. 3 d., 3 str. 4 d., 47 str.) nuostatos, pagal kurias „niekas neturi teisės reikalauti, kad teisėjas atsiskaitytų dėl konkrečioje byloje priimto sprendimo”, „bylose teismų priimtus sprendimus gali peržiūrėti tik teismas ir tik įstatymų nustatyta tvarka”, „teisėjas ar teismas neatsako už žalą, atsiradusią proceso šaliai dėl to, kad byloje priimtas neteisėtas ar nepagrįstas sprendimas. Šią žalą […] atlygina valstybė. Dėl teisėjo nusikalstamos veikos vykdant teisingumą atsiradusią ir asmeniui valstybės atlygintą turtinę ir neturtinę žalą valstybė regreso tvarka išieško iš teisėjo”; b) kolektyvinis teismų, kaip teisminės valdžios institucijų sistemos, nepriklausomumas ir savivalda. Konstitucinis Teismas išaiškino, kad „teismų veikla nėra ir negali būti laikoma valdymo sritimi, priskirta kuriai
87
tam tikras valdymo institucijos
KONSTITUCINĖ TEISĖ
nors vykdomosios valdžios institucijai” (Konstitucinio Teismo 1999 m. gruodžio 21 d. nutarimas). Todėl įstatymų leidėjas negali, pavyzdžiui, nustatyti teisingumo ministro įgaliojimus kontroliuoti teismų administracinę veiklą. Šioje vietoje pažymėtinos esminės Teismų įstatymo 2002 m. redakcijos naujovės, susijusios su teismų savivalda: pirma, sukurta nauja teismų savivaldos institucijų sistema: Visuotinis teisėjų susirinkimas (aukščiausia savivaldos institucija), Teismų taryba (iš 24 jos narių didžiausią narių dalį renka Visuotinis teisėjų susirinkimas, o pagal pareigas į tarybos sudėtį įeina Aukščiausiojo Teismo ir Apeliacinio teismo bei Vyriausiojo administracinio teismo pirmininkai, taip pat Respublikos Prezidento ir Seimo Pirmininko atstovai, teisingumo ir finansų ministrai ar jų įgalioti viceministrai, Seimo Teisės ir teisėtvarkos bei Biudžeto ir finansų komitetų pirmininkai ar jų pavaduotojai; pažymėtina, kad Teismų taryba – tai Konstitucijos 112 straipsnyje numatyta „speciali institucija”, kuri pataria Respublikos Prezidentui dėl teisėjų paskyrimo, paaukštinimo, perkėlimo ar atleidimo iš pareigų), Teisėjų garbės teismas (jo septynis narius skiria Teismų taryba ketveriems metams teisėjų drausmės byloms ir teisėjų prašymams dėl teisėjo garbės gynimo nagrinėti); antra, sukurtas naujas teismų savivaldos institucijų organizacinis metodinis ir techninis aptarnavimas, kurį turi atlikti Nacionalinė teismų administracija (ji kaupia ir analizuoja teismų statistiką, tvarko pretendentų į laisvas apylinkės teismo teisėjų vietas sąrašą bei teisėjų karjeros siekiančių asmenų registrą). Pažymėtina, kad Teismų įstatymas (11 str.), viena vertus, draudžia „bloginti įstatymų numatytas finansines ir materialines technines teismų veiklos sąlygas”, kita vertus, numato abstrakčią išlygą: „Kai iš esmės pablogėja valstybės ekonominė ir finansinė būklė, Seimas gali peržiūrėti teismų finansines ir materialines veiklos sąlygas”. Kaip ir pagal ką galima (reikėtų) spręsti apie „valstybės ekonominės ir finansinės būklės esminį pablogėjimą” – sudėtingas klausimas, kuris turi daug neteisinių (ūkinių) aspektų. Dar papildomai galima atkreipti dėmesį į ypatingą teisėjų ir teismų nepriklausomumo aspektą – depolitizavimą. Tai reiškia, kad teisėjams draudžiama dalyvauti politinių partijų veikloje (Konstitucijos 113 str. 2 d.), o politinėms partijoms – kištis į teisėjo ar teismo veiklą (Konstitucijos 114 str. 1 d.). Taigi teisėjas ir teismas, viena vertus, turi klausyti įstatymo, o ne politinių nurodymų ar instrukcijų (neatsitiktinai Konstitucinis Teismas išaiškino, kad teisėjų dalyvavimas „vykdant kokią nors politikos kryptį” yra imanentinis totalitarinio ir autoritarinio režimo bruožas; Konstitucinio Teismo 1995 m. gruodžio 6 d. nutarimas), kita vertus, Respublikos Prezidentas ir jam teisėjų paskyrimo klausimais patarianti teisėjų institucija (Konstitucijos 112 str. 5 d.) negali teisėjų skyrimo sieti su kandidatų (ar profesinės karjeros laiptais kylančių teisėjų) dalyvavimu politinėje veikloje (Konstitucinis Teismas 1999 m. gruodžio 21 d. nutarime nurodė, kad teisminė valdžia „vienintelė formuojama ne politiniu, bet profesiniu pagrindu”). Pažymėtina, kad depolitizavimo principas nedraudžia teisėjams jungtis į nepolitines organizacijas. Šiuo požiūriu Teisėjų asociacija ar panaši savanoriška organizacija (Teismų įstatymo 115 str.) atitinka konstitucinį teisėjo statusą.
88
3. KONSTITUCINĖS ŽMOGAUS TEISĖS IR LAISVĖS
3.1. BENDRA SAMPRATA
Konstitucijoje tiesiogiai nepasakoma, ką reiškia žodžiai „Žmogaus teisės ir laisvės” ir kodėl jos „yra prigimtinės” (18 str.). Logiškai ir lingvistiškai aiškinant Konstituciją, darytina išvada, kad prigimtinės teisės ir laisvės – tai bet kurioje valstybėje ar niekieno žemėje gimstant (ne gaunant pilietybę) įgyjamos pagrindinės teisės ir laisvės. Tai yra kiekvieno žmogaus buvimo, veikimo ar neveikimo ir neliečiamumo galimybės (ir tam tikros nepriklausomybės nuo kolektyvo – visuomenės ar valstybės – garantijos) bei kartu visos žmonijos vertybės, kurios istoriškai yra ankstesnės, negu susikūrė valstybė, o teisiškai yra aukščiau suverenios valstybės valdžios. Visa tai supaprastinus galima apibendrinti: 1. Konstitucija Lietuvos valstybės nesukuria, bet konstatuoja, kad yra gamtos sukurtos (gimstant įgyjamos) žmogaus teisės ir laisvės, kurioms apsaugoti ir ginti (žr. 6 str. 2 d., 30 str.) yra skirtos specialios Konstitucijos nuostatos (18-26 str.). Šiuo požiūriu valstybinė bendruomenė, priėmusi Konstituciją referendumu, sukuria tik teisines formuluotes, kuriomis tiesiogiai ar netiesiogiai išreiškiama gamtos sukurtų vertybių esmė. Tuo tarpu piliečių teisės (rinkti, stoti į partijas ar valstybės tarnybą; 3337 str.) įgyjamos tam tikram žmogui gaunant valstybės (šiuo atveju -Lietuvos Respublikos) pilietybę ir yra valstybės teisinės kūrybos produktas, išreikštas Konstitucijos nuostatose ir jų apsaugotas. 2. Nėra ir negali būti baigtinio prigimtinių teisių ir laisvių sąrašo, t. y. Konstitucijoje tik iš dalies aprašomos prigimtinės teisės (laisvės), kurias valstybės bendruomenė suvokė 1992 m. Tai patvirtina ir tai, kad Konstitucinis Teismas 1995 m. piliečio teisę laisvai kilnotis ir pasirinkti gyvenamąją vietą (Konstitucijos 32 str. 1 d.) įvertino kaip žmogaus (kiekvieno teisėtai esančio Lietuvoje žmogaus) teisę. 3. Niekas – nei žmogus, siekiantis žmogžudystės ar savižudybės, nei valstybės institucija ar jos pareigūnas, siekiantis apriboti ar uždrausti tam tikrą žmogaus veiklą, skirti jam baudą ar bausmę, – negali atimti ar savo nuožiūra valdyti tai, kas yra gamtos sukurta. Todėl negali valstybės institucijos nesikišti, jei žmogus laisva valia nusprendžia nusižudyti (atimti sau gyvybę) arba prašo, kad jam kiti padėtų tai padaryti (pastarasis atvejis – dabar diskutuojamas eutanazijos, kaip teisėtos pagalbos atimti gyvybę nepagydoma liga sergančiam ir dėl to kančias patiriančiam žmogui, įteisinimo klausimas). 4. Valstybė, pripažindama, kad gamtos sukurtos teisės ir laisvės yra vertybės, įsipareigoja jas besąlygiškai ginti, o esant neišvengiamai būtinybei -jas riboti. Tai ji gali atlikti konstitucingo įstatymo nustatyta tvarka pagal minėtą proporcingumo principą ir nepažeidžiant ribojamos teisės ar laisvės esmės – tai, be ko teisė ar laisvė yra beprasmė. Anot Konstitucinio Teismo, „jeigu teisė taip apribojama, kad jos įgyvendinti pasidaro neįmanoma, jeigu teisė suvaržoma peržengiant protingai suvokiamas ribas arba neužtikrinamas jos teisinis gynimas, tai tokiu atveju būtų pagrindo teigti, jog pažeidžiama pati teisės esmė” (Konstitucinio Teismo 2000 m. vasario 23 d. nutarimas).
89
KONSTITUCINĖ TEISĖ
Atkreiptinas dėmesys, kad Konstitucijoje tam tikrų teisių ir laisvių turėtojams (subjektams) įvardyti vartojami skirtingi žodžiai: „žmogus” (19-26 str.), „kiekvienas žmogus” (48 str. 1 d.), „kiekvienas dirbantis žmogus” (49 str. 1 d.), „pilietis” (32-37 str.), „asmuo” (29 str. 1 d., 30 str.), „kiekvienas lietuvis” (32 str. 4 d.). Pažymėtina, kad žodžiai „žmogus”, „asmuo”, „pilietis” nėra sinonimai. Jie skiriasi pagal turinio apimtį. Plačiausios sąvokos – „žmogus” ir „asmuo”: pirmoji apima ir piliečius, ir užsieniečius ar asmenis be pilietybės, antroji – visus fizinius ir (kalbant apie nuosavybės neliečiamumą, asmenų lygybę ar teisę kreiptis į teismą; Konstitucijos 23 str., 29 str., 30 str.) juridinius asmenis. Siauriausias yra žodis „pilietis”. Jis įvardija tik fizinius asmenis, turinčius Lietuvos Respublikos pilietybę (Konstitucinio Teismo 1995 m. sausio 24 d. nutarimas). Konstitucines asmens teises ir laisves, kad būtų aiškiau, galima grupuoti įvairiai. Pagal konstitucinių asmens teisių ir laisvių esmę ir pobūdį (t. y. pagal tai, kam apsaugoti ir ginti – gamtos sukurtoms vertybėms ar visuomenės ir valstybės sukurtoms vertybėms – jos gali būti taikomos) ir turėtojus (subjektus), jas galima skirstyti į dvi grupes: prigimtines universaliąsias ir specialiąsias. Prigimtinės universaliosios teisės ir laisvės – tai kiekvieno žmogaus teisės ir laisvės (nesvarbu, kokia jo tautybė, religija, gimtoji kalba, lytis ar pilietybė), kurios pagal savo esmę ir pobūdį įgyjamos gimstant, be to, leidžia žmogui apsisaugoti ir apginti savo autonomiją nuo valstybės institucijų ar pareigūnų. Tokios yra klasikinės gynybinės („apsauginės”) teisės ir laisvės, įtvirtintos Konstitucijos 19-26 straipsniuose. Specialiosios teisės, t. y. konkretaus žmogaus, turinčio tam tikrą pilietybę, tautybę, lytį ar amžių, teisės (pvz., 18 metų sulaukusio piliečio rinkimų teisė; žr. Konstitucijos 33-37 str.), arba kiekvieno žmogaus teisės (pvz., kiekvieno žmogaus teisė laisvai pasirinkti darbą ar kiekvieno dirbančio žmogaus teisė kasmet gauti atostogas; Konstitucijos 48 str. 1 d., 49 str.), yra išvestinės iš gimstant įgyjamų teisių kaip vertybių (pvz., žmogaus orumo neliečiamumo), bet jų tikslas nėra tiesiogiai apsaugoti žmogų nuo valstybės savivalės ar piktnaudžiavimo.
3.2. PRIGIMTINĖS UNIVERSALIOSIOS TEISĖS IR LAISVĖS
Klasikinės prigimtinės teisės ir laisvės įtvirtintos Konstitucijos 19-26 straipsniuose. Šios universaliosios žmogaus teisės į gyvybę ir į orumą yra visų kitų teisių ir laisvių pagrindas (sąlyga) ir todėl yra „aukščiausios vertybės” (Konstitucinio Teismo 1998 m. gruodžio 9 d. nutarimas). Teisės į gyvybę (19 str.) esmė — žmogaus biologinio fizinio egzistavimo neliečiamumas ir apsauga. Ši teisė yra absoliuti, nes jos neįmanoma konstitucingai riboti nepažeidžiant esmės (neatimant gyvybės). Konstitucinis Teismas 1998 m. gruodžio 9 d. nutarime pripažindamas, kad Baudžiamajame kodekse nustatyta mirties bausmė prieštarauja Konstitucijos nuostatoms „Žmogaus teisės ir laisvės yra prigimtinės” (18 str.) ir „Žmogaus teisę į gyvybę saugo įstatymas” (19 str.), rėmėsi įvairiais Konstitucijos aiškinimo būdais, iš kurių mažiausiai įtikinamais galima laikyti keturis: 1) istorinį požiūrį, – kai atsižvelgiama į Lietuvos Didžiosios Kunigaikštystės statutuose nustatytus mirties bausmės taikymo apribojimus ir Lietuvos Respublikos 1920 m. Konstitucijos nuostatą „Mirties bausmė naikinama” (16 str.),
90
Konstitucinės žmogaus teisės ir laisvės
kurios neliko vėlesnėse konstitucijose; 2) statistinį požiūrį – pastebėta, kad pastaraisiais metais Lietuvoje „daugiau negu 40 proc. nuteistųjų buvo skiriama reali laisvės atėmimo bausmė, tačiau tai nusikalstamumo didėjimo nesustabdė”; 3) lyginamąjį požiūrį – pažymėta, kad Vakarų ir Vidurio Europos valstybėse mirties bausmė panaikinta ar praktiškai nevykdoma; 4) lingvistinį požiūrį – Konstitucijos 19 straipsnyje įtvirtinta tik viena norma, kurios formuluotėje nenumatyta, kad įstatymu ar teismo sprendimu galima riboti žmogaus gyvybę. Labiausiai įtikinamais galima laikyti du Konstitucijos aiškinimo būdus: 1) loginį – išaiškinta, kad „mirties bausmė yra fizinis nuteisto žmogaus sunaikinimas, gyvybės atėmimas nepriklausomai nuo to, kokiu būdu tai daroma: sušaudant, pakariant, suleidžiant mirtiną vaistų dozę ar kitokiu būdu”; 2) sisteminį – nurodyta, kad nors Konstitucijos preambulės teiginiai apie darnią pilietinę visuomenę ir teisinę valstybę suponuoja mintį apie kiekvieno žmogaus ir visos visuomenės apsaugą nuo nusikalstamų kėsinimųsi, tačiau „bendrųjų interesų apsauga demokratinėje teisinėje valstybėje negali paneigti konkrečios žmogaus teisės apskritai”. Žmogaus teisės į gyvybę, kaip absoliučią vertybę, savaime nepažeidžia įstatymas, leidžiantis policijos, prokuratūros, Specialiųjų tyrimų tarnybos ar kitam valstybės pareigūnui panaudoti prieš žmogų šaunamąjį ginklą tam tikrais atvejais (pagrindais) ir tvarka, pavyzdžiui, kai: 1) pareigūnas užpuolamas jo gyvybei akivaizdžiai pavojingu būdu ir nėra kitų galimybių tokį užpuolimą atremti; 2) asmuo, atliekantis teismo paskirtą laisvės atėmimo bausmę, bėga iš bausmės atlikimo vietos ir akivaizdžiai atsisako paklūsti jį sustabdyti siekiančio pareigūno reikalavimui sustoti (žr., pvz., Prokuratūros įstatymo 56 straipsnį „Šaunamojo ginklo naudojimo tvarka”). Šiuo atveju vertinant įstatymo atitiktį Konstitucijai, svarbu atsižvelgti, ar įstatymas įpareigoja pareigūnus panaudoti, esant galimybei, pirmiausia kitas savigynos ar sulaikymo priemones, iššauti įspėjamąjį šūvį į orą, stengtis taikyti į kojas ir taip išvengti sunkių padarinių, tarp jų – netyčinio gyvybės atėmimo. Kalbant apie žmogaus gyvybę, problemiškas yra eutanazijos (gr. euthanasia -gera lengva mirtis) klausimas, t. y. aktyvios pagalbos (pvz., suleidžiant mirtiną vaistų dozę, atjungiant gyvybę palaikančius medicinos aparatus) nusižudyti nepagydoma liga sergančiam (ir dėl to skausmus kenčiančiam) asmeniui įteisinimas. Konstitucijos 19 straipsnio formuluotė lyg ir neleidžia teigti, kad Baudžiamajame kodekse gali nebūti numatytos baudžiamosios atsakomybės už tai, jog žmogus A, prašomas žmogaus B, padėjo pastarajam atimti gyvybę (tuo iš esmės jam ne padėdamas nusižudyti, bet jį nužudydamas). Taigi neatsitiktinai naujojo Baudžiamojo kodekso 134 straipsnyje rašoma: „tas, kas beviltiškai sergančio žmogaus prašymu padėjo jam nusižudyti, baudžiamas […]”. Tačiau akivaizdu, kad būtina atsižvelgti į tai, kad sergančio asmens gydymas beviltiškas ir skausmai nepakeliami, ir siekti atitinkamai keisti iki šiol vyravusią baudžiamosios (ir konstitucinės) teisės nuostatą, kad viešosios ar privačios medicinos įstaigų darbuotojai, viena vertus, gali ir turi švelninti mirštančio paciento skausmus, kita vertus, neturi teisės jam padėti įgyvendinti laisva valia priimtą sprendimą nusižudyti, siekiant išvengti beprasmių skausmų. Teisė į laisvės neliečiamumą (20 str.) apima žmogaus judėjimo ir bendrąją veikimo ar neveikimo (veiksmų atlikimo ar neatlikimo) laisvę, kuri gali būti ribojama (o jei tai nenumatyta Konstitucijos ir įstatymų arba taikoma ne įstatymo nustatyta tvarka – ir pažeidžiama) įvairiai: sulaikant ar suimant žmogų, skiriant jam areštą ar kitą laisvės suvaržymo (atėmimo) bausmę, jį priverstinai gydant ar taikant
91
KONSTITUCINĖ TEISĖ
organizuoto nusikalstamumo užkardymo priemones (žr. Organizuoto nusikalstamumo užkardymo įstatymu nustatytus teismo įpareigojimus). Šiuo atveju, vertinant teisės apribojimo konstitucingumą, svarbu atsižvelgti ne tik į reglamentavimo įstatymais buvimą ir jo taikymo būtinumą (pagrįstumą), bet ir konkretaus asmens, kuriam apribota judėjimo ar veikimo laisvė, savigynos galimybes, pavyzdžiui, psichikos ligonio teisę „būti išklausytam psichiatrijos įstaigos administracijos, […] teismo, kai sprendžiami jo priverstinio hospitalizavimo ir priverstinio gydymo […] klausimai” (Psichikos sveikatos priežiūros įstatymo 22 str.). Konstitucija tiesiogiai detalizuoja tik du (pavojingiausius) ribojimo atvejus: sulaikymą ir suėmimą (žr. 20 str. 2 ir 3 d.). Pažymėtina, kad kartu su Konstitucija 1992 m. spalio 25 d. referendumu priimto įstatymo „Dėl Lietuvos Respublikos Konstitucijos įsigaliojimo tvarkos” 8 straipsnyje nustatyta, kad Konstitucijos 20 straipsnio 3 dalies nuostatos (dėl nusikaltimo vietoje sulaikyto asmens pristatymo į teismą per 48 valandas tam, kad teismas, dalyvaujant sulaikytajam, nuspręstų, ar sulaikymas yra pagrįstas ir arba priimtų nutarimą asmenį suimti, arba jį tuoj pat paleistų) pradedamos taikyti „nuo tada, kai bus suderinti baudžiamojo proceso įstatymai su šia Konstitucija”. Tokia sunkiai suvokiama ir kritikuotina norma pradėta veiksmingai įgyvendinti, kai Lietuva 1995 m. ratifikavo Europos žmogaus teisių ir pagrindinių laisvių apsaugos konvenciją su laikina išlyga, kad prokuroro įgaliojimas leisti suimti asmenį, įtariamą padarius nusikaltimą, galios dar vienerius metus. Minėta Konvencija Lietuvoje įsigaliojo 1995 m. birželio 20 d., todėl ši išlyga neteko galios 1996 m. birželio 20 d. Be to, pažymėtina, kad Konstitucinis Teismas išaiškino, jog Europos žmogaus teisių ir pagrindinių laisvių apsaugos konvencijos žodis „skubiai”, vartojamas kalbant apie kiekvieno sulaikytojo ar suimtojo pristatymą teisėjui, atitinka Lietuvos Konstitucijos žodžius „per 48 valandas” ir „nusikaltimo vietoje sulaikytas asmuo” (Konstitucinio Teismo 1995 m. sausio 24 d. išvada). Tokį išaiškinimą galima ir pateisinti, ir laikyti nevisiškai įtikinamu. Loginiu požiūriu 48 valandos – pakankamai ilgas laiko tarpas sulaikyto žmogaus orumui pažeminti, ypač jei teismas 15 ar 20 valandų prievarta laikytą asmenį nuspręstų „tuojau pat paleisti”. Tiesa, analogiškas pristatymo teismui terminas nustatytas Italijos ir Portugalijos konstitucijose, o Ispanijos Konstitucijoje įtvirtinta net ilgesnė (72 valandų) trukmė. Šitai priminė ir mūsų Konstitucinis Teismas, pagrįsdamas savo išaiškinimą. Teisė į orumą (jo neliečiamumą; Konstitucijos 21 str. 2 d.) yra tokia pat reikšminga kaip ir teisė į gyvybę, nes tiek nuosavybės, tiek privataus gyvenimo ar būsto neliečiamumas yra tiesiogiai susijęs su žmogaus orumu. Taigi valstybės institucijoms ir pareigūnams draudžiama elgtis su žmogumi (pvz., keliant teisinę baimę, psichologinę prievartą ar fizinį skausmą sulaikant, suimant, atliekant asmens apžiūrą, nustatant ir skiriant bausmes, skiriant prievartinį gydymą ir pan.) kaip su daiktu. Žmogaus orumas -viena sudėtingiausių teisinių (ir filosofinių) kategorijų. Tai, kaip jis turi būti saugojamas, matyti, pavyzdžiui, iš tokių normų: „Areštinės kameros gyvenamojo (miegamojo) […] ploto norma vienam nuteistajam yra ne mažiau kaip 5 kvadratiniai metrai”; „Areštinės personalas į nuteistąjį raštu kreipiasi „Jūs” ir pavarde (į nuteistąjį areštu nepilnametį galima kreiptis žodžiu „Tu” ir vardu”) (teisingumo ministro 2003 m. birželio 18 d. įsakymu patvirtintų Areštinių vidaus tvarkos taisyklių 20 p., 36 p.). Teisė į privataus gyvenimo neliečiamumą (22 str. 1 d.) lingvistiškai ir sistemiškai aiškinant apima: 1) asmens susirašinėjimo, pokalbių telefonu ar kito
92
Konstitucinės žmogaus teisės ir laisvės
kio susižinojimo neliečiamumą (22 str. 2 d.); 2) asmeninio (taigi ir seksualinio) ir šeimyninio gyvenimo neliečiamumą (22 str. 4 d.). Konstitucinis Teismas išaiškino, kad asmens teisė į privatumą apima daugiau elementų: „privatų, šeimos ir namų gyvenimą, asmens fizinę ir psichinę neliečiamybę, garbę ir reputaciją, asmeninių faktų slaptumą, draudimą skelbti gautą ar surinktą konfidencialią informaciją ir kt.” (Konstitucinio Teismo 1999 m. spalio 21 d. nutarimas). Įstatymų leidėjo požiūriu privatus gyvenimas pažeidžiamas, kai: 1) neteisėtai įeinama į asmens gyvenamąsias ir kitokias patalpas ar „ a p t v e r t ą (išryškinta -aut.) privačią teritoriją”; 2) neteisėtai stebimas asmuo; 3) neteisėtai apieškomas asmuo ar jo turtas; 4) pažeidžiamas „korespondencijos bei asmeninių užrašų ir informacijos konfidencialumas”; 5) neteisėtai paskelbiami duomenys apie asmens sveikatos būklę; 6) atliekami kiti neteisėti veiksmai (Lietuvos Respublikos civilinio kodekso (toliau -CK) 2.23 str. 2 d.). Kalbant apie privataus gyvenimo neliečiamumą, įvairiai – ir teigiamai, ir neigiamai galima vertinti Visuomenės informavimo įstatymo (14 str. 3d.) nuostatą, kad informacija apie privatų gyvenimą gali būti skelbiama be žmogaus sutikimo tais atvejais, kai: 1) informacijos paskelbimas nedaro žalos asmeniui; 2) „informacija padeda atskleisti įstatymų pažeidimus ar nusikaltimus”; 3) „informacija yra pateikiama nagrinėjant bylą atvirame teismo procese”; 4) informacija apie viešojo asmens – valstybės politikų, tarnautojų, politinių partijų ar visuomeninių organizacijų vadovų bei „kitų visuomeninėje ir politinėje veikloje dalyvaujančių asmenų” – privatų gyvenimą atskleidžia „visuomeninę reikšmę turinčias privataus šio asmens gyvenimo aplinkybes ar asmenines savybes”. Paskutinė išlyga -sudėtingiausia ir labiausiai diskutuotina. Nuosavybės neliečiamumo garantija (Konstitucijos 23 str. 1 d.) apima ir privačios nuosavybės, kaip konstitucinio instituto, garantiją, ir įvairias subjektines (individualias) „nuosavybės teises” (Konstitucijos 23 str. 2 d.), kurias privalo saugoti įstatymų leidėjas. Konstitucinis Teismas laikosi tradicinio aiškinimo, kad „subjektines nuosavybės teisės – tai savininko teisė valdyti jam priklausantį turtą, juo naudotis bei disponuoti neperžengiant įstatymų nustatytų ribų, nepažeidžiant kitų asmenų teisių ar teisėtų interesų” (Konstitucinio Teismo 1999 m. spalio 6 d. nutarimas). Pažymėtina, kad pirmaisiais veiklos metais Konstitucinis Teismas aiškino, kad „nuosavybė yra teisė į turtą. Tai reiškia savininko teisę nekliudomai daryti turtui bet kokį poveikį, jei tuo nepažeidžiamas įstatymas, sutartis arba trečiųjų teisės, ir draudimą kitiems asmenims jį daryti” (Konstitucinio Teismo 1996 m. sausio 24 d. nutarimas). Vėliau Konstitucinis Teismas 2002 m. kovo 14 d. nutarime praplėtė nuosavybės sampratą, pažymėdamas, kad bendrasis nuosavybės neliečiamumas ir nuosavybės teisių apsauga reiškia ne tik „visų savininkų – fizinių asmenų, juridinių asmenų, savivaldybių ir valstybės” – nuosavybės apsaugą, bet ir kiekvieno asmens teisę į nuosavybę (t. y. galimybę tapti savininku). Be to, svarbu pažymėti, kad Konstitucinis Teismas sistemiškai sujungė nuosavybės ir „asmens ūkinės veiklos laisvės ir iniciatyvos” (46 str. 1 d.) turinį ir jo galimo pažeidimo vertinimą: „asmens konstitucinė teisė įgyti nuosavybę yra esminė (būtina) asmens ūkinės veiklos laisvės įgyvendinimo sąlyga”. Pavyzdžiui, jei Vyriausybės nutarimas draudžia įmonėms prekiauti „seno pavyzdžio banderolėmis” paženklintais Lietuvoje pagamintais alkoholiniais gėrimais, juos gabenti bei laikyti ir (svarbiausia!) nenustato, kaip įmonės po nutarime nurodytos datos gali realizuoti dar iki draudimo nustatymo teisėtai įgytus (joms nuosavybės teise priklausančius) alkoholinius gėrimus, akivaizdu, kad tokiu
93
KONSTITUCINĖ TEISĖ
nutarimu yra pažeidžiamos ir nuosavybės teisės (teisės valdyti nuosavybės teise priklausančius alkoholinius gėrimus, jais naudotis ir disponuoti), ir asmens ūkinės veiklos laisvė ir iniciatyva (Konstitucinio Teismo 2000 m. vasario 23 d. nutarimas). Teisė turėti savo įsitikinimus ir juos laisvai reikšti (Konstitucijos 25 str. 1 d.), taip pat informacijos laisvė (Konstitucijos 25 str. 2 d.) apima įvairias individualias galimybes (pvz., žodžio, nuomonės ar spaudos laisvę) ir kartu neliečiamas vertybes, kurios susijusios su vadinamuoju komunikacijos procesu. Anot Konstitucinio Teismo, „tokiame procese atsiranda visuomeninis kontaktas, formuojasi viešoji nuomonė” (Konstitucinio Teismo 1995 m. balandžio 20 d. nutarimas). Kalbant apie informacijos laisvę, svarbu pažymėti, kad Konstitucija (44 str.) draudžia visuomenės informavimo („masinės informacijos”) cenzūrą ir tokio informavimo priemonių (t. y. dienraščių ar kitų leidinių, leidyklų, radijo ar televizijos stočių, informacijos agentūrų ir pan.) monopolizavimą. Cenzūra, anot Konstitucinio Teismo 1995 m. balandžio 20 d. nutarimo, – tai „spaudos, kino filmų, radijo ir televizijos laidų, teatro spektaklių ir kitų viešų renginių turinio kontrolė, kad nebūtų platinamos tam tikros žinios ir idėjos”. Pažymėtina, kad aptariamos teisės nėra absoliučios. Konstitucijoje (25 str. 4 d.) vienareikšmiškai atmetamos („laisvė […] nesuderinama su”) tokios komunikacijos galimybės, kurios laikytinos „nusikalstamais veiksmais – tautinės, rasinės, religinės ar socialinės neapykantos, prievartos bei diskriminacijos kurstymu, šmeižtu ar dezinformacija”. Konstitucija (25 str. 3 d.), be to, leidžia įstatymais riboti teises (laisves) skleisti informaciją, jei tai būtina apsaugoti žmogaus garbę ir orumą, sveikatą, privatų gyvenimą, dorovę ar ginti konstitucinę santvarką. Šiuo požiūriu galima suprasti minėto Konstitucinio Teismo nutarimo išaiškinimus, kad: 1) teisės aktuose gali būti nustatytos tiek visuomeninio, tiek privataus radijo ar televizijos funkcijos (kultūrinė, jaunimo auklėjimo ir pan.), reklamos kiekis; 2) įstatymų leidėjas, nustatydamas žurnalisto teisę išsaugoti informacijos šaltinio paslaptį (neatskleisti informacijos šaltinio), negali sudaryti prielaidų neatskleisti – net teismui informacijos šaltinio net ir tada, kai „demokratinėje valstybėje informacijos šaltinį atskleisti yra būtina dėl gyvybiškai svarbių ar kitų ypač reikšmingų visuomenės interesų, […] kad būtų vykdomas teisingumas, nes informacijos šaltinio neatskleidimas galėtų sukelti daug sunkesnes pasekmes negu jo atskleidimas. […] Taigi įstatymų leidėjas, nustatydamas žurnalisto teisę išsaugoti informacijos šaltinio paslaptį […], (išryškinta – aut.) nustatyti ir tai, kad kiekvienu atveju spręsti, ar žurnalistas turi atskleisti informacijos šaltinį, gali tik teismas” (Konstitucinio Teismo 2002 m. spalio 23 d. nutarimas).
3.3. SPECIALIOSIOS PILIETINĖS (POLITINĖS) TEISĖS
Iš pirmo žvilgsnio gali atrodyti, kad visos piliečio teisės, įtvirtintos Konstitucijos 3237 straipsniuose, yra politinės teisės, nes Konstitucinis Teismas, kaip minėta, išaiškino, kad „pilietybė yra nuolatinis asmens p o l i t i n i s (išryškinta – aut.) teisinis ryšys su konkrečia valstybe” (Konstitucinio Teismo 1994 m. balandžio 13 d. nutarimas). Pagal tokį požiūrį išeitų, kad norint suteikti užsieniečiui peticijos teisę (33 str. 3 d.) ar teisę vienytis į visuomenines organizacijas (35 str. 1 d.), pirmiausia būtina keisti Konstituciją, įtvirtinant, pavyzdžiui, „žmogaus teisę teikti peticijas” ir „žmogaus teisę jungtis į visuomenines organizacijas”. Toks požiūris iš esmės yra
94
t u r i p a r e i g ą
Konstitucinės žmogaus teisės ir laisvės
k l a i d i n g a s . Akivaizdu, kad ne kiekviena piliečio teisė, pavyzdžiui, teisė laisvai kilnotis ar išvykti iš Lietuvos (32 str. 1 d.), yra politinė, nors istoriškai gal ir gali būti vertinama kaip „politinės prigimties” teisė. Klasikinėmis politinėmis galimybėmis šiandien galima laikyti tik rinkimų teisę ir teisę vienytis į politines partijas, nors ir jos gali būti iš dalies suteiktos užsieniečiams. Antai 2002 m. priimtoje Konstitucijos (119 str. 2 d.) pataisoje numatyta ne tik piliečių, bet ir kitų savivaldybės gyventojų teisė rinkti (ir būti renkamais) savivaldybių tarybų rinkimuose. Vertinant teisę vienytis į bendrijas, asociacijas ir visuomenines organizacijas ar politines partijas (35 str. 1 d.), galima teigti, kad: 1) ši teisė turi būti įgyvendinta laisvai, t. y. asmens laisva valia. Šiuo požiūriu Konstitucijos draudimas, kad „niekas negali būti verčiamas priklausyti kokiai nors bendrijai, politinei partijai ar asociacijai” (35 str. 2 d.), turi būti taikomas ir, pavyzdžiui, asmeniui A nusipirkus, paveldėjus ar kitaip įsigijus nuosavybės teise butą X daugiabučiame name. Jis negali iš karto būti verčiamas tapti to namo butų savininkų bendrijos nariu, nors tokiai bendrijai priklausė ankstesnis buto X savininkas. Konstitucinis Teismas išaiškino, kad reglamentavimas įstatymu, kai „narystės daugiabučių namų savininkų bendrijoje atsiradimas nesiejamas su savininko laisva valia”, prieštarauja Konstitucijos 35 straipsnio 1 ir 2 dalims (Konstitucinio Teismo 2000 m. gruodžio 21 d. nutarimas); 2) ši teisė yra politinė (tik piliečių) tik tada, jei ji susijusi su politinių partijų steigimu ir veikla. Tačiau kalbant apie bendrijas, asociacijas ar visuomenines organizacijas, šią teisę klaidinga laikyti politine, nes, pavyzdžiui, užsienietis, priklausydamas Lietuvoje registruotam golfo klubui, šachmatų mėgėjų draugijai, bitininkų asociacijai ar daugiabučio namo savininkų bendrijai, neįgyja išskirtinių galimybių kelti save kandidatu į Seimo narius ar dalyvauti rinkimų agitacijoje. Kita vertus, galima teigti, kad, pavyzdžiui, Asociacijų įstatymas, nustatantis, kad asociacijos nariais gali būti ir „kitų valstybių juridiniai ir fiziniai asmenys”, atitinka Konstituciją. Piliečio teisės laisvai kilnotis, pasirinkti gyvenamąją vietą Lietuvoje bei laisvai išvykti iš Lietuvos yra suformuluotos vienoje Konstitucijos nuostatoje (32 str. 1 d.). Pagal savo esmę ir tikslą jos nėra politinės, t. y. įstatymu gali ir turi būti garantuojamos ir užsieniečiams, teisėtai esantiems Lietuvoje. Šioje vietoje svarbu priminti, kad: 1) Konstitucinis Teismas, atsižvelgdamas į Lietuvoje esančių užsieniečių statuso reglamentavimą įstatymais, konstatavo Europos žmogaus teisių ir pagrindinių laisvių apsaugos konvencijoje įtvirtintą žmogaus ir Lietuvos Respublikos Konstitucijoje garantuojamą piliečio kilnojimosi teisių atitiktį pagal jų subjektus (Konstitucinio Teismo 1995 m. sausio 24 d. nutarimas); 2) įstatyme „Dėl užsieniečių teisinės padėties” (15 str.) nustatyta, jog leidimą gyventi Lietuvoje gavęs užsienietis įgyja teisę pasirinkti gyvenamąją vietą Lietuvoje, ją keisti, išvykti iš Lietuvos ir į ją sugrįžti (!) šio leidimo galiojimo laiku. Plačiau kalbant apie piliečio teisę laisvai išvykti iš Lietuvos, yra svarbus Konstitucinio Teismo išaiškinimas, kad „negali būti nustatoma leidiminė (t. y. leidimais pagrįsta – aut.) išvykimo tvarka, t. y. negali būti nustatoma tokia tvarka, kad pilietis turėtų prašyti kurios nors valstybinės institucijos leidimo išvykti iš Lietuvos” (Konstitucinio Teismo 2001 m. spalio 2 d. nutarimas). Šiuo atveju Konstitucinis Teismas vertino vairuotojo pažymėjimo paėmimo tvarką. Administracinį teisės pažeidimą vairuojant transporto priemonę padariusiam asmeniui, kuriam paskirta bauda, išduodamas laikinas leidimas vairuoti transporto priemonę, o vairuotojo pažymėjimas negrąžinamas, iki bus sumokėta paskirta bauda. Toks reglamentavimas pagal Konstitucinį Teismą „iš esmės
95
KONSTITUCINĖ TEISĖ
reiškia, kad asmuo užsienyje negali pasinaudoti minėtu dokumentu, liudijančiu jo teisę vairuoti transporto priemonę. Tačiau tai […] nereiškia, jog yra suvaržyta ar paneigta piliečio teisė laisvai išvykti iš Lietuvos”. Tačiau ginčytą reglamentavimą Konstitucinis Teismas pripažino prieštaraujančiu Konstitucijai kitu požiūriu – pagal teisinės valstybės principą, reikalaujantį proporcingai riboti žmogaus teises. Taigi „teisė vairuoti transporto priemonę yra įgyta teisė. Ją patvirtina vairuotojo pažymėjimas. Vairuotojo pažymėjimo negrąžinimas, iki bus sumokėta paskirta bauda, ir išdavimas laikino leidimo vairuoti transporto priemonę reiškia įgytos teisės suvaržymą, nes duodamas tik laikinas leidimas ja naudotis, tuo tarpu teisės vairuoti transporto priemonę atėmimas kaip administracinė nuobauda nėra paskirta”. Piliečių teisė rinktis be ginklo į taikius susirinkimus įtvirtinta Konstitucijos 36 straipsnyje. Ji yra detalizuota Konstitucinio Teismo išaiškinimuose. Ten parašyta: 1) Susirinkimų įstatymas numato pareikštinę (bet ne leidiminę) susirinkimų laisvės įgyvendinimo tvarką; 2) savivaldybės institucijos, pagal įstatymą nustatydamos nuolatinę susirinkimų vietą (kur organizuojant susirinkimus nereikia iš anksto įvertinti, ar nebus pažeistas visuomenės saugumas ar žmonių sveikata), neturi teisės reikalauti, kad visi susirinkimai būtų organizuojami tik tokioje vietoje (Konstitucinio Teismo 2000 m. sausio 7 d. nutarimas). Be to, svarbus Susirinkimų įstatymas, kuriame, be kita ko, numatytos įvairios susirinkimų formos (mitingai, demonstracijos, piketai, eitynės, procesijos), kurių vieta ir laikas suderinamas su savivaldybės vykdomąja institucija. Tik organizuojant piketą, kuriame dalyvauja „ne daugiau kaip 10 žmonių”, nebūtina organizatoriui raštu pranešti atitinkamos savivaldybės vykdomajai institucijai apie piketo vietą ir laiką (šiaip apie tai raštu pranešti būtina prieš penkias darbo dienas iki numatomo susirinkimo dienos). Susirinkimų įstatyme parašyta, kad susirinkimo organizatoriais gali būti ne tik 18 metų turintys piliečiai (tokią išvadą galima daryti iš Konstitucijos 36 str.), bet ir įregistruotos politinės partijos, visuomeninės organizacijos, profesinės sąjungos, religinės organizacijos ir kiti juridiniai asmenys (Susirinkimų įstatymo 4 str.). Rinkimasis be ginklo į taikius susirinkimus (Konstitucijos 36 str.) suprantamas ne tik pagal tai, ar susirinkimo dalyviai turi šaunamųjų ginklų. Įstatymas draudžia ir tokius susirinkimus, kurių dalyviai turi: 1) nešaunamuosius ginklus (pvz., medžioklinius peilius); 2) daiktus, kurie „specialiai pritaikyti kūno sužalojimams padaryti” (pvz., vadinamuosius kastetus, metalinius strypus, kuokas); 3) daiktus, kurie, nors ir nėra specialiai pritaikyti kūno sužalojimams (ar materialinei žalai) padaryti, bet kuriuos „realiai galima tokiems tikslams panaudoti” (pvz., namų ūkiui skirtus daiktus: kirvius, šakes ar peilius). Netaikus susirinkimas (arba juo tapti galintis susirinkimas) įstatyme suprantamas ir pagal kitus dalyvių požymius: tam tikrą apsirengimą, neapsirengimą ar sakomas kalbas. Nuostatose draudžiami susirinkimai, kurių dalyviai yra: 1) apsirengę karine uniforma, neperšaunamomis liemenėmis, slepia veidus kaukėmis ar yra kitaip užsimaskavę; 2) nuogi ar kitaip „ciniškai pažeidžia dorovę”; 3) kalbomis, plakatais, šūkiais ir kitokiais veiksmais „akivaizdžiai kursto pažeisti arba pažeidžia Lietuvos Respublikos konstitucinę santvarką ar įstatymus” (Susirinkimų įstatymo 8 str. 1p.,3p.,4.). Piliečių, priklausančių tautinėms bendrijoms, teisė puoselėti savo kalbą, kultūrą ir papročius įtvirtinta Konstitucijos 37 straipsnyje. Tai rodo, kad „tau
96
Konstitucinės žmogaus teisės ir laisvės
tinėms bendrijoms yra garantuojamas tautinio identiteto išsaugojimas, kultūros tęstinumas ir tautinė saviraiška” (Konstitucinio Teismo 1999 m. spalio 21 d. nutarimas). Pateikta teisė iš esmės nėra politinė, todėl ir užsienietis, nuolatos gyvenantis Lietuvoje, remdamasis Konstitucijos 37 straipsniu, gali kelti valstybei tam tikrus reikalavimus (pvz., gintis nuo prievartinės asimiliacijos, t. y. valstybės veiksmais keliamos grėsmės prarasti tautinį kultūrinį identitetą). Svarbūs šiuo atveju tokie Konstitucinio Teismo išaiškinimai: 1. „Kokiai tautybei priklausyti – tai asmens apsisprendimo reikalas, t. y. niekas, išskyrus patį asmenį, negali spręsti jo priskyrimo kokiai nors tautybei klausimo” (Konstitucinio Teismo 1999 m. spalio 21 d. nutarimas). Remiantis tokiu išaiškinimu, galima aiškiau suprasti, kodėl, pavyzdžiui, dvišalėse Lietuvos ir Ukrainos ar Lenkijos sutartyse dėl abipusio supratimo ir kaimyninio bendradarbiavimo nustatyta, kad Lietuvos Respublikos pilietis pats sprendžia, ar jis priklauso ukrainiečių ar lenkų tautinei bendrijai Lietuvoje (antai lenkų tautinei mažumai Lietuvoje priklauso asmenys, kurie turi Lietuvos pilietybę, yra lenkų kilmės arba priskiria save lenkų tautybei, kultūrai, laikosi lenkų tradicijų bei laiko lenkų kalbą savo gimtąja kalba (Lietuvos Respublikos ir Lenkijos Respublikos 1994 m. Draugiškų santykių ir gero kaimyninio bendradarbiavimo sutarties 13 str.). 2. Konstitucijos 37 straipsniui neprieštarauja ginčytas reglamentavimas, pagal kurį nelietuvių tautybės asmenų vardai ir pavardės išduodamame Lietuvos Respublikos piliečio pase rašomi lietuviškais rašmenimis. Toks Konstitucinio Teismo išaiškinimas pagrįstas tuo, kad, viena vertus, lietuvių kalba yra valstybinė kalba (Konstitucijos 14 str.), kuri yra „svarbi piliečių lygiateisiškumo garantija, nes leidžia visiems piliečiams vienodomis sąlygomis bendrauti su valstybės ir savivaldybių įstaigomis”, antra vertus, „Lietuvoje gyvenantys asmenys save priskiria daugiau nei šimtui tautybių […]. Jeigu teisės normomis būtų nustatyta, kad šių piliečių vardai ir pavardės Lietuvos Respublikos piliečio pase rašomi kitokiais, ne lietuviškais, rašmenimis, būtų […] sutrikdyta valstybės ir savivaldybių įstaigų, kitų įmonių, įstaigų bei organizacijų veikla”. Be to, Konstitucinis Teismas pažymėjo, kad lietuvių kalbos, kaip valstybinės kalbos, statusas reiškia, kad ją privalu vartoti tik viešajame gyvenime, o „kitose gyvenimo srityse asmenys nevaržomai gali vartoti bet kurią jiems priimtiną kalbą” (Konstitucinio Teismo 1999 m. spalio 21 d. nutarimas).
3.4. SPECIALIOSIOS ŪKINĖS IR KULTŪRINĖS (SOCIALINĖS) TEISĖS
Kiekvienas vos tik gimęs žmogus ne iš karto įgauna teisę laisvai rinktis, pavyzdžiui, darbą ar verslą (Konstitucijos 48 str. 1 d.), turėti kasmetines mokamas atostogas (Konstitucijos 49 str.) ar studijuoti aukštąjį mokslą (K 41 str. 3 d.). II Konstitucijos skirsnyje „Žmogus ir valstybė” (kuriame kalbama apie žmogaus gyvybę, laisvę, orumą) iš esmės kalbama apie prigimtines žmogaus teises. III ir IV Konstitucijos skirsniuose „Visuomenė ir valstybė” bei „Tautos ūkis ir darbas” jau kalbama apie asmens teises, kurios tarsi atskirtos nuo klasikinių prigimtinių vertybių (II skirsnio) ir pagal savo pobūdį bei paskirtį vadintinos specialiosiomis ūkinėmis ir kultūrinėmis (socialinėmis) teisėmis. Jos iš esmės nėra prigimtinės, todėl įstatymų leidėjas gali jas labiau modeliuoti, t. y. apibrėžti turinį, išimtis, nepaneigiančias
97
KONSTITUCINĖ TEISĖ
konkrečios teisės konstitucinio įtvirtinimo prasmės. Pavyzdžiui, pagal Konstituciją įstatymu negalima nustatyti žmogaus teisės gyventi ar būti oriu, jo amžiaus ar išsilavinimo, bet, kita vertus, galima nustatyti dirbti norinčio žmogaus minimalų amžių, privataus stomatologo aukštojo išsilavinimo cenzą ir pan. Visa tai nereiškia, kad specialiosios ūkinės ar kultūrinės teisės turi mažesnę konstitucinę galią nei klasikinės prigimtinės vertybės. Žmogaus teisė laisvai pasirinkti darbą ar verslą (Konstitucijos 48 str. 1 d.) reiškia, kad žmogus gali nevaržomai ar ribotai (kai būtina saugoti sveikatą, viešąją tvarką ar kitas konstitucines vertybes) pasirinkti konstitucingą (aiškiai ir įsakmiai įstatymais nedraudžiamą) darbą ar verslą ir tokį pasirinkimą įgyvendinti pagal aiškius, objektyviai būtinus ir realiai įvykdomus reikalavimus ar apribojimus. Antai žmogaus teisės laisvai pasirinkti advokato darbą esmė nepažeidžiama įstatymo, kuriuo neleidžiama tam tikrai valstybės institucijai įrašyti į advokatų sąrašą kiekvieno pageidaujančio asmens, turinčio specializuotą aukštąjį teisinį išsilavinimą (pvz., įgytą Rusijos ar buvusios TSRS prokuratūros, kitos teisėsaugos institucijos specializuotoje aukštojoje mokykloje), nes nustatyta, jog advokatas turi turėti aukštąjį universitetinį teisinį išsilavinimą. Tokio ribojimo pagrindą Konstitucinis Teismas, atsižvelgdamas į galimą advokato darbo įvairovę ir reikšmę padedant ginti įvairiai pažeidžiamas žmogaus teises ir laisves, įvertino kaip objektyviai būtiną ir konstitucingą (Konstitucinio Teismo 1996 m. liepos 10 d. nutarimas). Pažymėtina, kad pagal vientisą Konstitucijos sistemą teisės laisvai pasirinkti darbą ar verslą paskirtis yra užtikrinti būtinų žmogaus gyvybinių poreikių tenkinimą ir jo deramą padėtį visuomenėje (Konstitucinio Teismo 2003 m. liepos 4 d. nutarimas). Įstatymų leidėjo pareiga yra sudaryti reikiamas teisines prielaidas, atsižvelgiant į konkretaus darbo ar verslo pobūdį. Pasak Konstitucinio Teismo, vykdydamas tokią pareigą įstatymų leidėjas turi laikytis konstitucinių principų, tarp jų ir minėto asmenų teisėtų lūkesčių principo, ir taisyklės, kad negalima nustatyti tokio teisinio reglamentavimo, pagal kurį „asmuo, įgyvendindamas vieną konstitucinę teisę, netektų galimybės įgyvendinti kitą konstitucinę teisę”. Pavyzdžiui, Konstitucijos 48 straipsnio 1 daliai prieštarauja nuostatos, kuriose asmenims dėl gaunamos pensijos suvaržomos galimybės pasirinkti darbą ar verslą, „nors tokie asmenys ir atitiktų norintiems dirbti darbą ar verstis verslu nustatytas sąlygas”. Dar vienas pavyzdys: Konstitucija neleidžia nustatyti įstatymu, kad išėjusiems iš tarnybos valstybės pareigūnams ir kariams, turintiems teisę gauti tam tikrą pensiją, ši pensija jiems mokama visa, jei jie nėra laisvai pasirinkę tokio naujo darbo ar verslo, už kurį gautų „pajamų, nuo kurių skaičiuojamos ir mokamos valstybinio socialinio pensijų draudimo įmokos”, o jei yra laisvai pasirinkę tokį naują darbą ar verslą, už kurį gauna „pajamų, nuo kurių skaičiuojamos ir mokamos valstybinio socialinio pensijų draudimo įmokos”, tai jiems „mokama 30 procentų dydžio pareigūno ar kario valstybinės pensijos dalis”. Be to, išėjusiems iš tarnybos valstybės pareigūnams ir kariams, turintiems teisę gauti tam tikrą pensiją, ši pensija mokama, o jei jie laisvai pasirinktų vėl grįžti į tarnybą (jei tai pagal įstatymą leidžiama), anksčiau paskirta ir mokėta pensija nėra mokama. Piliečio teisė gauti pensiją -tai iš esmės kiekvieno teisėtai dirbusio ir mokesčius mokėjusio žmogaus reikalavimo teisė, netiesiogiai numatyta nuostatoje „valstybė laiduoja piliečių teisę gauti senatvės ir invalidumo pensijas, socialinę paramą nedarbo, ligos, našlystės, maitintojo netekimo ir kitais įstatymų numatytais atvejais” (Konstitucijos 52 str.). Kalbant apie pensijas, svarbi Konstitucinio Teismo
98
Konstitucinės žmogaus teisės ir laisvės
2003 m. liepos 4 d. nutarime pateikta doktrina (t. y. argumentuotų teiginių ir koncepcijų sistema). Pagal ją, įstatymu nustačius tam tikrą pensinį aprūpinimą, pagal kurį tam tikri asmenys įgyja teisę į konkrečią pensiją, valstybė „prisiima įsipareigojimą tokią pensiją paskirti ir mokėti”, o asmenys, atitinkantys įstatyme nustatytas sąlygas, „turi teisę reikalauti, kad valstybė jiems skirtų […] pensiją ir ją mokėtų” (t. y. „vykdytų įstatymu prisiimtą įsipareigojimą ir mokėtų nustatyto dydžio išmokas”). Nustatytas pensinis aprūpinimas įstatymu gali būti bloginamas tik tada, kai akivaizdžiai susidaro ypatinga situacija ir dėl to būtina apsaugoti kitas konstitucines vertybes. Kitaip tariant, esant ekonominei krizei ar kitai ypatingai situacijai, kai objektyviai trūksta lėšų, būtinų pensijoms mokėti, paskirtos ir mokamos pensijos gali būti laikinai (kol yra ypatinga situacija) mažinamos proporcingai, lyginant su „gyvybiškai svarbiais visuomenės interesais”. Be to, pensijų sistema gali būti pertvarkoma įstatymu, laikantis valstybės prisiimtų konstitucingų įsipareigojimų mokėti tam tikras pinigines išmokas. Jeigu pertvarkant pensijų sistemą iš esmės pakeičiamas pensijų teisinis reglamentavimas (arba nelieka įstatymais pirmiau nustatytų pensijų), įstatymų leidėjas privalo, viena vertus, „numatyti pakankamą pereinamąjį laikotarpį, per kurį asmenys, dirbantys atitinkamą darbą ar atliekantys atitinkamą tarnybą, suteikiančią teisę į atitinkamą pensiją pagal ankstesnį reguliavimą, galėtų pasirengti tokiems pakeitimams”, antra vertus, „nustatyti teisingą susidariusių praradimų kompensavimo (išryškinta – aut.) mechanizmą asmenims, kuriems […] pensija buvo paskirta ir mokama”. Asmens ūkinės veiklos laisvė ir iniciatyva įtvirtinta Konstitucijos 46 straipsnio 1 dalyje. Pagal Konstitucinio Teismo išaiškinimą yra ne viena konkreti, bet įvairių teisinių galimybių visuma, kurios turi sudaryti prielaidas tiek fiziniam, tiek juridiniam asmeniui savarankiškai priimti jo ūkinei veiklai reikalingus sprendimus, o jos turinį sudaro teisė laisvai sudarinėti sutartis, sąžiningos konkurencijos laisvė, ūkinės veiklos subjektų lygiateisiškumas ir kt. (Konstitucinio Teismo 2002 m. kovo 14 d. nutarimas). Tai lengviau suprasime pasitelkę konkretų Konstitucijos taikymo pavyzdį. Įstatymų leidėjas, saugodamas žmonių sveikatą, apribojo farmacinę veiklą kaip ypatingą asmens ūkinę veiklos sritį (nes ji susijusi su vaistų laikymu, gaminimu ir pardavimu), nustatydamas, kad privati vaistinė nuosavybės teise gali priklausyti arba aukštąjį farmacinį išsilavinimą turinčiam fiziniam asmeniui, arba fizinių asmenų grupei, kurioje daugiau kaip pusė vaistinės įstatinio kapitalo priklauso asmenims, turintiems farmacinį išsilavinimą. Toks ribojimas pagal Konstitucinį Teismą yra nepakankamai pagrįstas, neproporcingas ir todėl antikonstitucinis. Toks Konstitucinio Teismo sprendimas pagrįstas tuo, kad: 1) vien buvimas vaistinės savininku nereiškia, jog savininkas turi teisę vykdyti farmacinę veiklą, kuriai yra būtinas farmacinis išsilavinimas; 2) įstatymu galima nustatyti tokius vaistinių savininkų nuosavybės teisių apribojimus, kad jų nuosavose vaistinėse tik asmenys, turintys farmacinį išsilavinimą, galėtų vadovauti vaistinei ir kontroliuoti joje dirbančius asmenis, vykdančius farmacinę veiklą; 3) „neleidžiama riboti asmens teisės į nuosavybę priklausomai nuo asmens išsilavinimo”, t. y. tam tikro išsilavinimo reikalavimas negali būti nustatomas asmenims, kurie siekia „turėti nuosavybės teise vaistinių”. Kalbant apie asmens ūkinės veiklos laisvę ir iniciatyvą, pažymėtina, kad jos yra ne tik kiekvieno esamo ar galimo (būsimo) verslo subjekto individuali galimybė, bet ir valstybės ūkio pagrindas. Pagal Konstituciją (46 str. 1 d.) pagrindinės verty
99
KONSTITUCINĖ TEISĖ
bės, kuriomis grindžiamas valstybės (jos bendruomenės) ūkis – tai privati nuosavybė, asmens ūkinės veiklos laisvė ir iniciatyva. Taigi asmens ūkinės veiklos laisvė nėra absoliuti. Tai užfiksuota Konstitucijoje: „valstybė reguliuoja ūkinę veiklą taip, kad ji tarnautų bendrai tautos gerovei” (46 str. 3 d.). Ką reiškia tokia gerovė – sudėtingas politinis ir ekonominis socialinis (ne tik teisinis) klausimas. Konstitucinis Teismas šiuo klausimu pateikė abstraktų ir sudėtingą aiškinimą: tautos gerovę atspindi materialinių gėrybių vartojimas, tačiau tam įvertinti neturėtų būti sureikšmintas „individo materialinių poreikių tenkinimas”, t. y. sąvokos „bendra tautos gerovė” turinys „kiekvienu konkrečiu atveju atskleidžiamas atsižvelgiant į ekonominius, socialinius bei kitus svarbius veiksnius”. Kitaip tariant, „tautos gerovė neturėtų būti suvokiama vien materialine (ar finansine) prasme” (Konstitucinio Teismo 1997 m. vasario 13 d. nutarimas). Ką reiškia „valstybė reguliuoja” -paprastesnis klausimas. Į jį Konstitucinis Teismas atsakė taip: „ūkinės veiklos reguliavimui būdingas bendrojo leidimo metodas: leidžiama viskas, kas nėra draudžiama. Taigi draudimai yra vienas iš ūkinės veiklos reguliavimo būdų. […] Įstatyme nustatyti draudimai turi būti pagrįsti, adekvatūs siekiamam tikslui, nediskriminaciniai, aiškiai suformuluoti” (Konstitucinio Teismo 1999 m. spalio 6 d. nutarimas). Visa tai susiję su konstituciniu draudimu (jį turi detalizuoti įstatyminis draudimas) monopolizuoti gamybą ir rinką (46 str. 4 d.) ir principu „valstybė gina vartotojo interesus” (46 str. 5 d.). Antai Konstitucinis Teismas, vertindamas Telekomunikacijų įstatymo nustatytą vieno ūkio subjekto – akcinės bendrovės „Lietuvos telekomas”teisę verstis fiksuoto telefono ryšio paslaugų teikimu iki 2002 m. gruodžio 31 d., išaiškino, kad, viena vertus, pagal Konstituciją „negalima įvesti monopolio, t. y. negalima įstatymu suteikti ūkio subjektui išimtinių teisių veikti kurioje nors ūkio srityje, dėl ko ši sritis būtų monopolizuota”, o antra vertus, draudimas monopolizuoti gamybą ir rinką „nereiškia, kad yra draudžiama tam tikromis aplinkybėmis įstatyme konstatuoti monopolijos konkrečioje ūkinės veiklos srityje buvimą. Toks konstatavimas sudaro teisines prielaidas taikyti monopolistui atitinkamus reikalavimus ginant vartotojų interesus” (Konstitucinio Teismo 1999 m. spalio 6 d. nutarimas). Konstitucinis Teismas nurodė, kad specialios vartotojų interesų gynimo priemonės yra diskriminuojančių kainų nustatymo ribojimas, monopolinės rinkos prekių kainų dydžių, tarifų valstybinis reguliavimas, prekių kokybės ar kitų reikalavimų monopoliniam ūkio subjektui nustatymas ir pan. Tai išaiškinęs ir pažymėjęs, kad ginčytame įstatyme pereinamajam laikotarpiui nustatytas „konkurencijos telekomunikacijų srityje draudimas nėra visuotinis ir absoliutus – mobiliojo ryšio, „Interneto” tinklo paslaugų rinka yra laisva”, Konstitucinis Teismas nusprendė, kad ginčytos Telekomunikacijų įstatymo nuostatos neprieštarauja Konstitucijos 46 straipsniui.
3.5. KONSTITUCINĖS ASMENS PAREIGOS
Dabartinėje Lietuvos Konstitucijoje nėra vieno bendro skirsnio, kuriame būtų įtvirtintos piliečių ar asmenų pareigos. Keletą jų galima rasti 28, 38, 53 ir 139 straipsniuose. Taigi padaryta priešingai negu LTSR 1940 ir 1978 m. konstitucijose, kurios piliečių pareigas prilygino teisėms ar net iškėlė virš jų. Klasikiniai pavyzdžiai yra tokios nuostatos: 1) kiekvieno darbingo LTSR piliečio pareiga – „sąžiningai dirbti
100
Konstitucinės žmogaus teisės ir laisvės
jo pasirinktoje visuomenei naudingos veiklos srityje […]. Vengimas visuomenei naudingo darbo (o kas tai yra, galėjo spręsti tik vienintelė komunistų partija -past. aut.) nesuderinamas su socialistinės visuomenės principais (1978 m. Konstitucijos 58 str.); 2) „kiekvienas LTSR pilietis „turi saugoti ir stiprinti visuomeninę, socialistinę nuosavybę, kaip šventą ir neliečiamą tarybų santvarkos pagrindą […]. Asmenys, kurie kėsinasi į visuomeninę socialistinę nuosavybę, yra liaudies priešai” (1940 m. Konstitucijos 103 str.). Toks įsakmus ir gąsdinantis juridinis tonas bei formuluotės užgožė visas politines ar klasikines asmens teises, kurias LTSR konstitucijos pateikė su gausybe išlygų. Turint visa tai omenyje, svarbu pažymėti, kad klasikinė ir pirminė demokratinės valstybės Konstitucijos paskirtis yra įtvirtinti prigimtines žmogaus teises gintis nuo galimo valstybės institucijų ar pareigūnų kėsinimosi (o ne jų pareigas valstybėje). Įvairiuose galiojančios Lietuvos Konstitucijos skirsniuose įterptos asmens pareigos nesudaro vientisos pareigų sistemos, kurią reikėtų gretinti su žmogaus ir piliečio teisių bei laisvių sistema. Šiuo požiūriu galima kritiškai vertinti vadovėlius ar kitokius teisės leidinius, kurių struktūroje į vieną skirsnį sudedamos „pagrindinės žmogaus teisės, laisvės ir p a r e i g o s (išryškinta – aut.)”. Kiekvieno žmogaus pareiga laikytis Konstitucijos ir įstatymų bei nevaržyti kitų žmonių teisių ir laisvių (Konstitucijos 28 str.) yra teisinės valstybės principo dalis. Ši pareiga rodo, kad nė viena prigimtinė žmogaus teisė ar laisvė nėra visiškai absoliuti, jei ja piktnaudžiaujama siekiant varžyti analogišką ar panašią kito žmogaus teisę (laisvę). Žmogaus teisės ir laisvės gali ir turi būti ribojamos proporcingai kitų žmonių teisių ir laisvių varžymui. Anot Konstitucinio Teismo, „tarp asmenų teisių ir laisvių iš vienos pusės ir visuomenės interesų – iš kitos neretai kyla konfliktų, o kartais atsiranda ir prieštaravimų. Demokratinėje visuomenėje tokie prieštaravimai sprendžiami derinant skirtingus interesus ir siekiant nepažeisti jų pusiausvyros. Vienas iš interesų derinimo būdų yra asmens teisių ir laisvių ribojimas” (Konstitucinio Teismo 1997 m. vasario 13 d. nutarimas). Visa tai turi principinę reikšmę vertinant tai, kas yra teisinga ir kas yra teisingumas: „teisingumo negalima pasiekti pripažįstant tik vieno asmens interesus ir neigiant kito asmens teisėtus interesus” (Konstitucinio Teismo 1995 m. gruodžio 22 d. nutarimas). Pažymėtina, kad pareiga nevaržyti kitų žmonių teisių ir laisvių liečia ir privačius juridinius asmenis, kurie iš esmės yra tam tikrų žmonių teisinio susivienijimo forma. Tėvų pareiga (ir teisė!) auklėti savo vaikus ir juos iki pilnametystės išlaikyti (Konstitucijos 38 str. 6 d.), taip pat vaikų pareiga gerbti tėvus, globoti juos senatvėje ir tausoti jų palikimą (Konstitucijos 38 str. 7 d.) rodo, kad konstitucinė teisė iš dalies reglamentuoja ir privačius žmonių santykius, t. y. kad tam tikri tokių santykių aspektai nėra visiškai privatūs ir turi viešąją reikšmę. Toks reglamentavimas yra principinis ir abstraktus, suteikiantis įstatymų leidėjui tam tikrą teisėkūros erdvę. Antai įstatyme nustatytas „tėvų valdžios apribojimas” (CK 3.180 str.), kurio kraštutinė priemonė -neterminuotas tėvų valdžios apribojimas, kuriam esant „be atskiro tėvų sutikimo vaikas gali būti įvaikintas” (CK 3.180 str. 3 d.). Tai vaiko prevencinės apsaugos ir kardomojo gynimo priemonė, kuri gali būti taikoma tik teismo (ne valdymo institucijos) sprendimu, esant akivaizdžiai (tai gali rodyti, pvz., tėvų žiaurus elgesys su vaikais ar nesirūpinimas jais, amoralumas) būtinybei (CK 3.183 str.). Šitai suinteresuotas asmuo (tėvai ar vienas iš jų) gali ginčyti teisme (Konstitucijos 30 str. 1 d., 110 str. 2 d.). Nereikėtų pernelyg sureikšminti Konstitucijos žodžius „tėvai […] n e v a r ž o m i (išryškinta -aut.) rūpinasi vaikų […] doro
101
KONSTITUCINĖ TEISĖ
viniu auklėjimu” (Konstitucijos 26 str. 5 d.) ir neatsižvelgti į nuostatą „nepilnamečius vaikus gina įstatymas” (Konstitucijos 39 str. 3 d.). Ši nuostata (kartu su žmogaus teises įtvirtinančiais straipsniais) reikalauja, kad valstybės institucijos nedelsiant įsikištų į tėvų ir vaikų santykius, jei, pavyzdžiui, tėvai smurtauja prieš vaikus ar taiko tokią „auklėjimo” priemonę kaip „plakimas rykštėmis”. Toks „auklėjimo” būdas neatitinka žmogaus orumo neliečiamumo (Konstitucijos 21 str. 2 d.) ir su juo susijusios nuostatos, kad „draudžiama žmogų kankinti, žaloti, žeminti jo orumą, žiauriai su juo elgtis, taip pat nustatyti tokias bausmes” (Konstitucijos 21 str. 3 d.). Nuostata, kad „asmenims iki 16 metų mokslas yra privalomas” (Konstitucijos 41 str. 1 d.), rodo, kad vaikų mokymasis nėra tik tėvų privatus auklėjimo reikalas. Įstatymų leidėjas gali nustatyti tėvams tam tikras baudas, jei jie sąmoningai neužtikrina, kad būtų laikomasi minėtos nuostatos. Šiuo požiūriu tikslinga kalbėti ne tiek apie vaikų iki 16 metų pareigą mokytis, kiek apie tėvų ir valstybės institucijų pareigą užtikrinti, kad vaikai mokytųsi. Tai yra susiję su atviros, darnios ir pilietinės visuomenės siekiu, apie kurį kalbama Konstitucijos preambulėje. Galima būtų tik suabejoti, ar tikslinga Konstitucijoje įvardyti konkretų (mokytis turinčio asmens) amžių. Tai iš principo galėtų nustatyti įstatymų leidėjas – galėtų būti tiek 16, tiek 17 ar 18 metų riba. Įtvirtinus Konstitucijoje konkretų aptariamos pareigos subjekto maksimalų amžių, jo įstatymų leidėjas didinti negali (antraip netektų prasmės konstitucinis reglamentavimas). Šiuo požiūriu galima suprasti, kodėl po diskusijų Seime naujojoje (2003 m.) Švietimo įstatymo redakcijoje atsisakyta padidinti pareigos mokytis amžių iki 18 metų. Švietimo įstatyme nustatyta, kad: 1) privalomasis švietimas – tai piliečių ir užsieniečių, turinčių teisę nuolat gyventi ar laikinai apsigyventi Lietuvos Respublikoje, privalomas ir valstybės garantuojamas ugdymas iki 16 metų pagal pradinio (ketverių metų) ir pagrindinio (šešerių metų) ugdymo programas; 2) „vaikas iki 16 metų negali nutraukti mokymosi pagal privalomojo švietimo programas” (Švietimo įstatymo 2 str. 21 d., 9 str., 10 str., 29 str. 8 d.). Klasikinė piliečio pareiga (ir teisė) – ginti Lietuvos valstybę nuo užsienio ginkluoto užpuolimo, taip pat pareiga įstatymo nustatyta tvarka atlikti karo ar alternatyviąją krašto apsaugos tarnybą (139 str. 1 d. ir 2 d.). Detalizuoti šių pareigų esminius aspektus Konstitucija įgalioja įstatymų leidėją, kuris ir nustatė, kad: 1) 19 metų – minimalus, o 26 metai – maksimalus karo šauktinio amžius (Karo prievolės įstatymo 4 str. 6 d.); 2) vyras – karo prievolininko lytis (Karo prievolės įstatymo 4 str. 6 d.). Šiuo atveju minėtinas Moterų ir vyrų lygių galimybių įstatymas (2 str. 3 d.), pagal kurį lygių lyčių galimybių pažeidimu („tiesiogine diskriminacija dėl lyties”) nelaikoma „tik vyrams taikoma karinė prievolė”. Konstitucijoje netiesiogiai įtvirtinta (t. y. neįvardyta, bet logiškai ir sistemiškai išplaukia) asmens pareiga mokėti mokesčius. Joje įtvirtinta: 1) žmogaus pareiga laikytis įstatymų (Konstitucijos 28 str.); 2) išimtinis įstatymų leidėjo įgaliojimas nustatyti mokesčius (Konstitucijos 127 str. 3 d.). Pagal šias nuostatas žmogus privalo laikytis visų įstatymų, taigi ir tų, kuriuose nustatomi mokesčiai. Paprasčiau tariant, žmogus negali įvykdyti pareigos laikytis įstatymų nemokėdamas įstatymuose nustatytų mokesčių. Todėl galima pagrįstai kalbėti apie konstitucinę (o ne įstatymo sukurtą) asmens pareigą mokėti mokesčius. Pažymėtina, kad mokesčių mokėjimas yra susijęs su nuosavybe (Konstitucijos 23 str.) ir asmens teisinio saugumo
102
Pagrindinės valstybės valdžios institucijos
principu (t. y. teisine valstybe). Todėl įstatymų leidėjas neturi visiškos laisvės nustatyti bet kokius mokesčius ar baudas už jų nemokėjimą. Antai Konstitucinis Teismas išaiškino, kad Konstitucijai prieštaravo įstatymas, pagal kurį tiek vieną kartą, tiek keletą kartų pažeidus pareigą mokėti mokesčius skiriama vienoda ir pagal protingumo kriterijų neproporcingai didelė piniginė nuobauda: „ne mažiau kaip 50 000 litų” (tai išaiškinta Konstitucinio Teismo 2000 m. gruodžio 6 d. nutarime).
4. PAGRINDINĖS VALSTYBĖS VALDŽIOS INSTITUCIJOS
4.1. SEIMAS
SEIMO SAMPRATA Konstitucijos 55 straipsnio 1 dalyje pasakyta, kad Seimą sudaro 141 tautos atstovas, tačiau tiesiogiai Seimas neapibūdinamas. Konstitucinis Teismas yra detalizavęs, kad Seimas – valstybinės bendruomenės atstovybė, parlamentas, kad „kiekvienas parlamento narys atstovauja tautai, visi parlamento nariai yra tautos atstovybė” (Konstitucinio Teismo 1993 m. lapkričio 26 d. nutarimas). Konkretesnę Seimo sampratą galima pateikti remiantis valdžių padalijimo principu, kuris jau buvo aptartas. Minėta, kad pagal valdžių padalijimo principą valstybės valdžią vykdo keletas lygiaverčių institucijų, iš kurių kiekviena turi savo esminę kompetenciją (leisti įstatymus, vykdyti juos bei atlikti administravimą, vykdyti teisingumą), kurios negalima nei perduoti, nei atsisakyti ar leisti perimti kitoms institucijoms. Taigi teisiniu požiūriu Seimas: 1) nėra pagrindinė ar svarbiausioji valstybės institucija, t. y. yra ne aukščiausioji, bet viena iš aukščiausiųjų valstybinės valdžios institucijų, nurodytų Konstitucijos nuostatoje „Valstybės valdžią Lietuvoje vykdo Seimas, Respublikos Prezidentas ir Vyriausybė, teismas” (5 str. 1 d.). Pažymėtina, kad dabartinėje Konstitucijoje atsisakyta perimti Lietuvos Respublikos 1990 m. Laikinojo Pagrindinio Įstatymo (78 str.) teiginį, kad parlamentas – anuomet Lietuvos Respublikos Aukščiausioji Taryba – yra „aukščiausiasis valstybinės valdžios organas” (šis teiginys 1990 m. perimtas iš LTSR 1978 m. Konstitucijos 97 str. 1 d.); 2) yra vienintelė institucija, turinti įgaliojimą leisti įstatymus (Konstitucijos 67 str. 2 p.) – norminius teisės aktus, kurie, nors ir būdami, pagal teisinę galią, palyginti su Konstitucija, žemesnio lygio (Konstitucijos 7 str. 1 d.), yra tiesiogiai ar netiesiogiai numatyti Konstitucijoje kaip aktai, kuriais leidžiama detalizuoti esminius jos nuostatų klausimus ir riboti tam tikras žmogaus teises ir laisves; 3) yra vienintelė institucija, turinti įgaliojimą nustatyti (įstatymu) mokesčius (Konstitucijos 127 str. 3 d.) ir tvirtinti (taip pat įstatymu) valstybės biudžetą (131 str.); 4) yra tiesiogiai renkama ir atstovaujamoji valstybės valdžios institucija, pagal legitimaciją (t. y. demokratinį įgaliojimų atsiradimą iš valstybinės bendruomenės suvereniteto) galinti būti gretinama tik su Respublikos Prezidentu. Apie Seimo rinkimų sistemą kalbėta anksčiau aptariant demokratinės valstybės principą.
103
KONSTITUCINĖ TEISĖ
SEIMO KOMPETENCIJA Konstitucinių Seimo įgaliojimų sąrašas (67 str.) susideda iš 20 punktų. Juose ar kituose straipsniuose (pvz., 106 str.) įtvirtintus įgaliojimus galima sugrupuoti: 1. Įgaliojimai, susiję su įstatymų leidyba. Į šią grupę įeina mokesčių nustatymas, valstybės biudžeto tvirtinimas, ministerijų steigimas ar panaikinimas, amnestijos paskelbimas, tarptautinių sutarčių ratifikavimas ar valstybės teritorijos administracinių vienetų (savivaldybių ir „aukštesniųjų administracinių vienetų”, dabar vadinamų apskritimis) nustatymas. 2. Įgaliojimai, susiję su valdymu ir atitinkamų Seimo nutarimų priėmimu. Į šią grupę įeina valstybės kontrolieriaus ir Lietuvos banko valdybos pirmininko skyrimas, Vyriausiosios rinkimų komisijos sudarymas, Respublikos Prezidento ir savivaldybių tarybų rinkimų ir referendumų datos nustatymas, parlamentinės kontrolės vykdymas nutarimais, kuriais, pavyzdžiui, pritariama ar nepritariama Vyriausybės narių atsakymams į interpeliaciją arba pareiškiamas nepasitikėjimas Seimo skirtu ar rinktu pareigūnu. 3. Įgaliojimai, rodantys ypatingus konstitucinius santykius su Respublikos Prezidentu, Vyriausybe, teismais. Seimo kompetencijai priklauso kreipimasis (nutarimu) į Konstitucinį Teismą dėl visų jo jurisdikcijai priskirtų aktų konstitucingumo įvertinimo, trijų teismų teisėjų – Konstitucinio Teismo, Aukščiausiojo Teismo ir Apeliacinio teismo teisėjų – skyrimas (nutarimais), užsienio politikos klausimų svarstymas, pritarimas ar nepritarimas (nutarimu) Respublikos Prezidento teikiamai Ministro Pirmininko kandidatūrai, Vyriausybės programos svarstymas ir pritarimas ar nepritarimas jai (nutarimu), nepasitikėjimo Ministru Pirmininku ar Vyriausybe pareiškimas (nutarimu). 4. Įgaliojimai, susiję su valstybės apsauga ir gynyba. Seimo kompetencijai priklauso karo ir nepaprastosios padėties įvedimas, mobilizacijos skelbimas, sprendimas panaudoti ginkluotąsias pajėgas. Visa tai detalizuota Karo padėties įstatyme, Nepaprastosios padėties įstatyme ir Nacionalinio saugumo pagrindų įstatyme. Pažymėtina, kad „Seimas yra savarankiškas tiek, kiek jo galių neriboja Konstitucija” (Konstitucinio Teismo 1993 m. spalio 1 d. nutarimas).
SEIMO STRUKTŪRA Konstitucija suteikia Seimui įgaliojimą nustatyti savo struktūrą specialiu aktu -Seimo statutu (Konstitucijos 76 str.). Pasak Konstitucinio Teismo, „Seimo struktūra – tokia Seimo statute nustatytų jo vidinių padalinių sistema, kuri turi užtikrinti parlamento darbingumą, efektyvų jo funkcionavimą, t. y. turi sukurti reikiamas sąlygas bei prielaidas Seimui nepertraukiamai įgyvendinti Konstitucijoje apibrėžtas jo funkcijas” (Konstitucinio Teismo 1994 m. vasario 24 d. nutarimas). Seimo statuto nustatytoje Seimo struktūroje išskirti: Seimo komitetai, frakcijos ir Seimo vadovai (t. y. Seimo Pirmininkas ir jo pavaduotojai). Šias struktūros grandis galima detalizuoti. Seimo komitetai sudaromi pirmiausia pagal dalykinę ar profesinę Seimo narių specializaciją. Jų paskirtis – „užtikrinti tinkamą dalykinį klausimų parengimą” ir jų svarstymą Seimo posėdyje (Konstitucinio Teismo 1993 m. lapkričio 26 d. nutarimas). Jie sudaromi iš dalies laisva Seimo narių valia (kiekvienas Seimo narys gali
104
Pagrindinės valstybės valdžios institucijos
pageidauti, kad frakcija rekomenduotų Seimui jį skirti tam tikro komiteto nariu) ir iš dalies pagal imperatyvų reikalavimą, kad kiekvienas Seimo narys, išskyrus Seimo Pirmininką ir Ministrą Pirmininką (jei šis yra Seimo narys), būtų kokio nors komiteto narys (Seimo statuto 10 str.). Kiekvienas komitetas susideda iš 7-17 narių (išskyrus Europos reikalų komitetą). Komitetuose laikomasi proporcingo frakcijų atstovavimo: frakcijos rekomenduoja skirti komitetų nariais tiek savo atstovų, kiek joms yra skirta vietų (atsižvelgiant į frakcijos didumą, t. y. narių skaičių). Komitetų sudėtį tvirtina Seimas. Seimo politinė dauguma turi galimybę netvirtinti komitetų sudėties, kuri jiems atrodo nepriimtina (t. y. daryti įtaką, kad opozicinės frakcijos rekomenduotų skirti į tam tikrus komiteto narius kitus atstovus). Seimo narių frakcijos pirmiausia išreiškia politinę Seimo narių orientaciją. Jų paskirtis pagal minėtą Konstitucinio Teismo nutarimą – padėti įgyvendinti Seimo nariams pagal politines orientacijas savo tikslus (nebūtinai vien pagal jų partinę priklausomybę) bei užtikrinti „organizuotus parlamento narių grupių ryšius su parlamente atstovaujamomis politinėmis partijomis bei organizacijomis”. Frakcijos sudaromos tik pagal laisvą Seimo narių apsisprendimą. Tai reiškia, kad Seimo statutas (38 str.) neįpareigoja Seimo narius priklausyti kuriai nors frakcijai. Jame sakoma: „Seimo nariai į frakcijas jungiasi laisva valia, nevaržomi jokių mandatų”. Tiesa, galioja ribojimas, kad frakcijų kūrimas negali būti grindžiamas profesiniais ar vietiniais interesais. Tokį ribojimą galima pateisinti, nes Seimo narys privalo būti ne tam tikros profesijos, gyvenamosios vietovės ar regiono, bet visos valstybinės bendruomenės atstovas (tai įtvirtinta Konstitucijos 59 str. 4 d.). Seimo statute (26 str.) nustatyta, kad Seimo nariai, nesijungiantys į frakcijas, laikomi „vienos mišrios Seimo narių grupės nariais”. Tokia grupė turi visas frakcijai suteiktas teises. Tokį reglamentavimą galima suprasti ir pateisinti, nes: 1) frakcijos turi kai kurias specifines teises, kurių neturi atskiri Seimo nariai; 2) skiriant tam tikrus Seimo narius mišrios grupės nariais (net jei jie to nenori), siekiama ne juos diskriminuoti ar priversti jungtis į frakcijas, bet suteikti jiems vienodai veiksmingas galimybes dalyvauti Seimo darbe kaip ir kitiems frakcijose esantiems Seimo nariams. Kalbant apie frakcijų sudarymo tvarką, pažymėtina, kad laisvo mandato principas leidžia Seimui nustatyti tik pareikštinę frakcijų sudarymo tvarką. Frakciją steigiančių Seimo narių bendras pareiškimas apie frakcijos sudarymą iš esmės reiškia frakcijos veiklos pradžią. Seimo statuto (38 str., 39 str.) nuostatose įtvirtinta, kad ne mažiau kaip septyni Seimo nariai, įkūrę frakciją, sesijos metu pateikia Seimo Pirmininkui jų pasirašytą pareiškimą, kuriame nurodytas frakcijos pavadinimas ir jos seniūno bei jo pavaduotojų pavardės, o Seimo Pirmininkas, nustatęs, kad frakcija atitinka reikalavimus (susijusius su minimaliu narių skaičiumi ir draudimu kurti frakcijas pagal profesinius ar vietinius interesus), privalo ne vėliau kaip per savaitę Seimo posėdyje paskelbti apie frakcijos įkūrimą. Pažymėtina, kad, pasak Konstitucinio Teismo, tiesioginiais ar netiesioginiais leidimais pagrįsta („leidiminė”) frakcijų sudarymo tvarka, kai Seimas ar Seimo Pirmininkas turi laisvę registruoti (neregistruoti) frakciją atsižvelgdamas į tai, ar ją steigiantys Seimo nariai pasirašo deklaraciją laikytis tam tikrų įsipareigojimų, prieštarauja konstituciniam laisvo mandato principui (59 str. 4 d.). Be to, svarbus Konstitucinio Teismo išaiškinimas, kad Konstitucijai prieštarauja Seimo statuto nuostata, kad politinės partijos ir jų koalicijos, kurių nariai yra išrinkti į Seimą pagal tą patį rinkimų sąrašą, sudaro vieną frakciją. Tai pagrįsta tokiais argumentais: 1) nors frakcijos dažniausiai ir su
105
KONSTITUCINĖ TEISĖ
daromos pagal parlamentarų partiškumą, tačiau tai nereiškia, jog frakcija yra partija Seime (t. y. frakcijas formuoja politinės partijos); 2) pagal Konstitucijos nuostatą „Pareigas eidami Seimo nariai […] negali būti varžomi jokių mandatų” kiekvienas Seimo narys turi laisvą mandatą, t. y. turi galimybę veikti savarankiškai, nežiūrint jį rinkimuose rėmusios ar iškėlusios partijos (taip pat kitų asmenų) reikalavimų ar nurodymų. Toks išaiškinimas leidžia teigti, kad Seimo statute negalima drausti ar riboti: 1) steigti frakcijas, nesvarbu, pagal kurį kandidatų sąrašą Seimo narys gavo mandatą daugiamandatėje apygardoje ar kokiai partijai jis priklauso frakcijos steigimo metu; 2) keisti narystę frakcijoje, t. y. frakcijos nariui išstoti iš tam tikros frakcijos ir pereiti į kitą frakciją, nesvarbu, kad jis neišrinktas pagal jos kandidatų sąrašą daugiamandatėje apygardoje ar nėra gavęs jos paramos vienmandatėje apygardoje. Įdomu tai, kad pagal Seimo statuto 1998 m. redakciją minimalus frakcijos narių skaičius yra septyni, o pagal ankstesnę redakciją buvo trys. Toks skaičiaus padidinimas šiek tiek apsunkina frakcijų steigimą, bet iš principo nepaneigia jų steigimo laisvės ir kartu laisvo mandato. Todėl frakcijų narių skaičiaus padidinimas Konstituciniam Teismui buvo visai priimtinas (Konstitucinio Teismo 2001 m. sausio 25 d. nutarimas). Kalbant apie frakcijas, svarbu paminėti opozicines frakcijas, kurios nuo kitų frakcijų skiriasi viešai pareikštu nusistatymu Vyriausybės atžvilgiu. Pagal Seimo statutą (41 str.) opozicinės yra tokios frakcijos (ar jų koalicijos), kurios: 1) pasiskelbia, kad nesutinka su Vyriausybės programa; 2) paskelbia politines deklaracijas, kuriose išdėstomos jas nuo Seimo politinės daugumos skiriančios nuostatos; 3) paskelbia alternatyvias Vyriausybės programas. Visa tai rodo ypatingą Seimo struktūros reglamentavimą, kuriuo siekiama užtikrinti demokratinę parlamentinės valios formavimo ir išraiškos tvarką. Pasak Konstitucinio Teismo, „parlamentui nustatant frakcijų formavimo tvarką turėtų būti atsižvelgiama į […] mažumos gynimo principą, parlamentinės opozicijos minimalius reikalavimus” (Konstitucinio teismo 1993 m. lapkričio 26 d. nutarimas). Šiuo požiūriu verta dėmesio opozicijos lyderio pareigybė: jei opozicinė frakcija (ar jų koalicija) turi daugiau kaip 1/2 Seimo politinei mažumai priklausančių Seimo narių, tokios frakcijos (koalicijos) seniūnas (vadovas) yra vadinamas Seimo opozicijos lyderiu ir turi tam tikras specifines teises (Seimo statuto 41 str.). Tokia teisė, pavyzdžiui, atsispindi Seimo statuto 108 straipsnio nuostatoje: „Be eilės vieną kartą kalbėti diskusijoje gali Respublikos Prezidentas, Seimo Pirmininkas, Ministras Pirmininkas ir Seimo opozicijos lyderis”. Minėtoji pareigybė, pasak Konstitucinio Teismo, savaime neprieštarauja Konstitucijai, tačiau jos pavadinimas nėra gana aiškus, nes susidaro įspūdis, kad opozicijos lyderis gali kalbėti visų opozicinių frakcijų vardu, nepaisant, ar jos visos ar ne visos dalyvavo jį renkant (Konstitucinio Teismo 2001 m. sausio 25 d. nutarimas). Seimo Pirmininkas Konstitucijoje įvardijamas: 1) kaip pirmininkaujantis Seimo posėdžiams (66 str. 1 d.); 2) kaip promulguojantis (t. y. pasirašantis ir oficialiai skelbiantis) Seimo statuto pataisas ir įstatymus, jei to per 10 dienų dėl objektyvių priežasčių nepadaro Respublikos Prezidentas (70 str. 2 d., 71 str. 2 d. ir 4 d.); 3) kaip laikinai einantis Respublikos Prezidento pareigas (šiam mirus ar atsistatydinus, žr. Konstitucijos 89 str. 1 d.) ar laikinai pavaduojantis jį (laikinai išvykus į užsienį arba susirgus; žr. Konstitucijos 89 str. 2 d.); 4) kaip skiriantis kandidatus į Konstitucinio Teismo teisėjus (Konstitucijos 103 str. 1 d.); 5) kaip priklausantis Valstybės gynimo
106
Pagrindinės valstybės valdžios institucijos
tarybai, svarstančiai svarbiausius valstybės gynybos klausimus (140 str. 1 d.). Seimo statute (187 str.) nustatyta, kad Seimo Pirmininkas renkamas iš Seimo narių pirmame naujo išrinkto Seimo posėdyje slaptu balsavimu, o kandidatus raštišku pareiškimu gali siūlyti ne mažiau kaip 1/10 Seimo narių. Kandidatas laikomas išrinktu, jeigu už jį balsavo daugiau kaip pusė balsavusių (taigi ne visų išrinktų) Seimo narių, o jei nė vienas kandidatas negauna tokios balsų daugumos, pakartotinai balsuojant išrinktu laikomas tas, kuris gauna daugiau balsų (žr. Seimo statuto 191 str.). Seimo statute (24 str.) yra reikalavimas, kad Seimo Pirmininkas sustabdytų savo veiklą Seimo narių frakcijoje. Dar, kalbant apie Seimo pirmininką, tikslinga paminėti Konstitucinio Teismo išaiškinimą, kad Seimo statuto nuostata, suteikianti Seimo posėdžio pirmininkui galimybę savo nuožiūra nutraukti kiekvieną Seimo nario klausimą, jei toks klausimas, Seimo Pirmininko manymu, „yra panašus į Seimo nario pareiškimą ar nuomonės sakymą”, prieštarauja Seimo nario laisvo mandato ir visų Seimo narių lygybės principams (Konstitucinio Teismo 2001 m. sausio 25 d. nutarimas). Seimo Pirmininko pavaduotojas Konstitucijoje įvardijamas: 1) kaip galintis pirmininkauti Seimo posėdžiams (bet ne vadovauti Seimui ar atlikti kitus Seimo Pirmininko įgaliojimus; Konstitucijos 66 str. 1 d.); 2) kaip Seimo pavedimu galintis eiti Seimo Pirmininko pareigas, jei šis laikinai eina Respublikos Prezidento pareigas (jei Respublikos Prezidentas mirė, atsistatydino, buvo pašalintas iš pareigų apkaltos proceso tvarka ar Seimas nutarė, kad jo sveikatos būklė neleidžia jam eiti valstybės vadovo pareigų; Konstitucijos 89 str. 1 d.). Pažymėtina, kad Seimo Pirmininko pavaduotojas, išvykus Seimo Pirmininkui ar dėl kitų pirmininko nebuvimo priežasčių (jos nėra esminis klausimas), negali vietoj Seimo Pirmininko pasirašyti priimtą įstatymą, jei to dėl objektyvios situacijos (pvz., išvykos į užsienį) per 10 dienų nepadarė Respublikos Prezidentas (žr. Konstitucijos 71 str. 2 d.). Tai išaiškino Konstitucinis Teismas (Konstitucinio Teismo 2002 m. birželio 19 d. nutarimas). Be to, įdomu, kad Konstitucijoje vartojama vienaskaita „Seimo Pirmininko pavaduotojas”, o Seimo statutas numato keletą pavaduotojų. Konstitucinis Teismas išaiškino, kad Konstitucijos nuostata apie Seimo Pirmininko pavaduotojo įgaliojimą pirmininkauti posėdžiams „turi organizacinės funkcijos prasmę, nes […] nurodoma, kas vadovauja Seimo posėdžiams, todėl negalima daryti išvados, kad šia norma yra reglamentuojami struktūros klausimai, t. y. nustatomas Seimo Pirmininko pavaduotojų skaičius” (Konstitucinio Teismo 1994 m. vasario 24 d. nutarimas). Vadinasi, Konstitucija leidžia Seimui savarankiškai nustatyti Seimo Pirmininko pavaduotojų skaičių. Antai Seimo statuto 1998 m. redakcijoje (24 str.) nustatyta, kad „Seimo Pirmininko pavaduotojų yra ne daugiau kaip penki”, o „Seimo Pirmininko teikimu Seimas vieną iš Seimo Pirmininko pavaduotojų paskiria Seimo Pirmininko pirmuoju pavaduotoju”.
SEIMO DARBO TVARKA Konstitucija suteikia Seimui įgaliojimą nustatyti savo darbo tvarką Seimo statutu (74 str.). Toks įgaliojimas nėra absoliutus, nes Konstitucijoje tiesiogiai reglamentuojami kai kurie Seimo darbo tvarkos klausimai (pvz., įstatymų priėmimas, priimtų įstatymų vetavimas ir pakartotinis svarstymas; 69-72 str.). Be to, pažymėtina, kad Seimo nario laisvo mandato principas (Konstitucijos 59 str. 4 d.) draudžia Seimui nustatyti, pavyzdžiui, tokią Seimo nario pašalinimo iš posėdžio salės (ar posė
107
KONSTITUCINĖ TEISĖ
džio pirmininko galimybės nutraukti Seimo narių pasisakymus) tvarką, kuri akivaizdžiai pažeistų ar sudarytų prielaidas pažeisti laisvo mandato esmę. Antai Seimo statute (21 str.) nustatyta, kad Seimo nario pašalinimas (iš posėdžio), kurį už šiurkštų ar tęstinį Seimo darbo trukdymą (prievartos naudojimą ar kvietimą ją naudoti, grasinimą ar viešą įžeidimą) gali siūlyti Seimo posėdžio pirmininkas ar Seimo Etikos ir procedūrų komisija, skiriamas Seimo posėdyje dalyvaujančių Seimo narių daugumos sprendimu, kuris priimamas be svarstymo (t. y. be įtikinėjimo ar įrodinėjimo), o pašalinimo trukmė negali būti ilgesnė kaip dvi posėdžių dienos (jei Seimo nario pašalinimas neįmanomas be apsaugos darbuotojų palydos, posėdžio pirmininko sprendimu, pašalinimo iš salės laikas gali būti pratęstas iki 5 posėdžių). Toks reglamentavimas iš principo nesudaro prielaidų Seimo Pirmininkui ar Seimo politinei daugumai savivaliauti ar pažeisti Seimo nario laisvo mandato principą. Į Seimo darbo tvarką galima pažiūrėti siauresniu ir platesniu požiūriu. Pagal šią tvarką, viena vertus, galima apibrėžti įstatymų leidybos procedūrą (siaurasis aspektas), antra vertus, spręsti konstitucinės kompetencijos (pvz., užsienio politikos klausimų svarstymo; Konstitucijos 67 str. 16 p.) ir vidinės autonomijos (pvz., komitetų ar frakcijų sudarymo) klausimus (platusis aspektas). Toliau nagrinėjama siauroji darbo tvarkos samprata. Konstitucija (68-72 str.) rodo, kad įstatymų leidyba – tai „visuma juridiškai reikšmingų veiksmų, būtinų, kad būtų priimtas įstatymas, ir atliekamų tam tikra […] logine ir laiko seka” (Konstitucinio Teismo 1993 m. lapkričio 8 d. nutarimas). Konstitucijoje tiesiogiai numatytos penkios įstatymų leidybos procedūros stadijos: 1. Inicijavimo stadija. Ją pradėti gali (t. y. „įstatymų leidybos iniciatyvos teisę” turi) kiekvienas Seimo narys (ar jų grupė), Respublikos Prezidentas, Vyriausybė ir 50 tūkstančių piliečių (Konstitucijos 68 str.). Anot Konstitucinio Teismo, „tinkamam subjektui pareiškus įstatymo sumanymą, įstatymų leidybos institucijos – parlamento pareiga yra pradėti svarstyti pateiktą projektą”. 2. Svarstymo stadija. Pagal Seimo statutą (141-143 str.) svarstymo stadijos pradžia gali būti suprantama dvejopai. Svarstymas prasideda, viena vertus, kai, projekto iniciatoriui trumpai (iki 10 minučių) apibūdinus projektą Seimo posėdyje, Seimo nariai pradeda jam teikti klausimus, t. y. prasideda trumpa (iki 10 minučių) pirminių klausimų pateikimo iniciatoriui ir jo atsakymų atkarpa. Antra vertus, svarstymo pradžia galima laikyti tą momentą, kai, iniciatoriui atsakius į jam pateiktus klausimus, priimamas Seimo sprendimas „pradėti projekto svarstymo procedūrą” (143 str.). Pastarasis požiūris, atrodo, labiau priimtinas, nes po projekto pateikimo ir atsakymo į klausimus gali būti priimtas ir Seimo sprendimas „atmesti projektą nurodant motyvus” (143 str.). Pažymėtina, kad įstatymo projektas gali būti atmestas ne narių balsų dauguma. Be to, svarbu ir tai, kad pagal Seimo statutą projekto svarstymą sudaro ne tik svarstymas Seimo posėdyje, bet ir pagrindiniame komitete, kurį Seimas paskiria konkrečiam projektui svarstyti, ar kituose komitetuose. Verta dėmesio ir tai, kad įstatymų projektai gali būti svarstomi skubos tvarka arba ypatingos skubos tvarka, jei taip nusprendžia Seimas pagal motyvuotą teikimą (skubos tvarka svarstyti teikia Respublikos Prezidentas, Vyriausybė, Seimo Pirmininkas, Seimo opozicijos lyderis, Seimo valdyba, pagrindinis komitetas ar frakcija, o ypatingos skubos tvarka – Respublikos Prezidentas, Vyriausybė ar Seimo Pirmininkas). Pažymėtina, kad Konstitucija tiesiogiai nenumato Seimo posėdžio kvorumo, t. y. tam tikro posėdyje dalyvaujančių Seimo narių skaičiaus,
108
mažesne kaip 1/4 visų Seimo
Pagrindinės valstybės valdžios institucijos
kuriam nesant įstatymų projektų svarstymas laikytinas neteisėtu, o priimti sprendimai – negaliojančiais. Kitaip tariant, Konstitucija nereikalauja, kad pradedant posėdį dalyvautų daugiau kaip pusė ar ne mažiau kaip trečdalis visų Seimo narių. Tokį kvorumą iš principo galėtų nustatyti Seimo statutas – tai ne tiek konstitucinis, kiek politinis ir dalykinis Seimo vidinės darbo tvarkos klausimas. Tiesa, jei Seimo statutas nustatytų tam tikrą teisėto posėdžio dalyvių minimumą, tai dar neturėtų reikšti, kad toks minimumas visais atvejais būtinas ir priimant įstatymus. Šiuo atveju kiltų neaiškumas, ar Seimo statutas nekonkuruoja su Konstitucijos nuostata, kad įstatymams priimti pakanka bet kokios (nekvalifikuotos!) posėdyje dalyvaujančių Seimo narių balsų daugumos (Konstitucijos 69 str. 2 d.). Pažymėtinas ir Konstitucinio Teismo išaiškinimas (Konstitucinio Teismo 2001 m. sausio 25 d. nutarimas), kad Konstitucija leidžia Seimo statute nustatyti, jog Seimo posėdyje, pradėjus „įstatymo priėmimo procedūrą”, būtų svarstomos ne kiekvieno Seimo nario, bet ne mažiau kaip 1/5 Seimo narių pateiktos pastabos. Šiuo atveju svarbiausia tai, kad nėra ribojama kiekvieno Seimo nario įstatymų leidybos iniciatyvos teisė (Konstitucijos 68 str. 1 d.), t. y. yra reglamentuojama tik antroji – svarstymo – stadija, kurią pradėjus kiekvieno Seimo nario įstatymų leidybos iniciatyvos teisė būna įgyvendinama. 3. Priėmimo stadija. Apie ją Konstitucijoje užsimenama trumpai: „Įstatymai laikomi priimtais, jeigu už juos balsavo dauguma Seimo narių, dalyvaujančių posėdyje” (Konstitucijos 69 str. 2 d.). Vadinasi, įstatymams priimti nustatytas paprastos Seimo narių balsų daugumos principas. Tiesa, kai kuriems įstatymams priimti Konstitucijoje nustatytas kvalifikuotos (t. y. fiksuotos) daugumos principas. Konstituciniam įstatymui (juo laikytinas arba įstatymas, nurodytas Konstitucijoje kaip konstitucinis, arba įstatymas, Seimo įrašytas į konstitucinių įstatymų sąrašą, kurį Seimas turi patvirtinti ne mažesne kaip 3/5 Seimo narių balsų dauguma; tokio sąrašo Seimas iki 2004 m. liepos mėn. nebuvo nustatęs – tai viena didžiausių Konstitucijos įgyvendinimo problemų; daugiau apie tai žr. Konstitucinio Teismo 2002 m. gruodžio 24 d. nutarimą) priimti būtina, kad už jį balsuotų daugiau kaip pusė visų Seimo narių (Konstitucijos 69 str. 3 d.). Respublikos Prezidento grąžintas įstatymas gali būti priimtas, nepriimant Respublikos Prezidento teiktų pataisų ir papildymų, daugiau kaip 1/2 visų Seimo narių balsų dauguma, o grąžintas konstitucinis įstatymas – ne mažiau kaip 3/5 visų Seimo narių balsų dauguma (Konstitucijos 72 str. 2 d.). Įstatymui dėl Konstitucijos pataisos priimti būtina ne mažesnė kaip 2/3 visų Seimo narių balsų dauguma (balsuojant už projektą du kartus, tarp kurių daroma ne mažesnė kaip trijų mėnesių pertrauka) (Konstitucijos 148 str.). Įstatymas dėl Lietuvos Respublikos tarptautinės sutarties, kuria keičiamos valstybės sienos, ratifikavimo gali būti priimtas, jei už jį balsuoja ne mažiau kaip 4/5 visų Seimo narių (Konstitucijos 10 str. 2 d., 138 str. 1 d. 1 p.). Pažymėtina, kad įstatymų nuostatas taip pat galima priimti referendumu (Konstitucijos 69 str. 4 d.). Tokia galimybe per pirmąjį 1992 m. Konstitucijos galiojimo dešimtmetį nebuvo pasinaudota. 4. Promulgavimo (t. y. pasirašymo ir oficialaus paskelbimo) stadija. Šioje stadijoje iš principo turėtų veikti tik Respublikos Prezidentas. Jis po įstatymo įteikimo per 10 dienų gali pasirinkti vieną iš dviejų alternatyvių sprendimų: arba pasirašyti ir oficialiai paskelbti įstatymą, arba jį motyvuotai grąžinti Seimui pakartotinai svarstyti (Konstitucijos 71 str. 1 d.). Tiesa, tam tikrais išimtiniais atvejais (kai yra neišvengiama būtinybė, pvz., Respublikos Prezidentui išvykus į užsienį) įstatymą pro
109
KONSTITUCINE TEISĖ
mulguoti gali Seimo Pirmininkas. Tai numatyta Konstitucijoje: „Jei nurodytu laiku (per dešimt dienų -aut.) Seimo priimto įstatymo Respublikos Prezidentas negrąžina ir nepasirašo, toks įstatymas įsigalioja po to, kai jį pasirašo ir oficialiai paskelbia Seimo Pirmininkas” (71 str. 2 d.). Ši nuostata nereiškia, kad Respublikos Prezidentas, būdamas valstybės vadovu (Konstitucijos 77 str. 1 d.) ir kartu valstybės stabilumo ir nuspėjamumo simboliu, turi galimybę savo nuožiūra spręsti, ar promulgavimo stadijoje jam ką nors daryti (vetuoti ar pasirašyti) ar nieko nedaryti. Tai išaiškino Konstitucinis Teismas: „Konstitucijos 71 straipsnio 2 dalies formuluotė […] nėra interpretuotina kaip reiškianti Respublikos Prezidento teisę nepasirašyti ir oficialiai nepaskelbti Seimo priimto įstatymo ir kartu motyvuotai negrąžinti jo Seimui pakartotinai svarstyti” (Konstitucinio Teismo 2002 m. birželio 19 d. nutarimas). Promulgavimo stadijoje gali dalyvauti ir Seimo Pirmininko pavaduotojas, jei jis tai daro kaip Seimo pavedimu laikinai einantis Seimo Pirmininko pareigas (kai Seimo Pirmininkas laikinai eina Respublikos Prezidento pareigas; Konstitucijos 89 str. 1 d.). Jokiais kitais atvejais (pvz., kai Seimo Pirmininkas, turintis pasirašyti Respublikos Prezidento per 10 dienų dėl objektyvių priežasčių nepasirašytą įstatymą, yra išvykęs) Seimo Pirmininko pavaduotojas negali promulguoti įstatymų. 5. Įsigaliojimo stadija. Konstitucijoje parašyta, kad įstatymai įsigalioja juos pasirašius ir oficialiai paskelbus (taigi paskelbimo dieną), jeigu „pačiais įstatymais nenustatoma vėlesnė įsigaliojimo diena” (70 str. 1 d.). Pažymėtina, kad įstatymo galią turintis Seimo statutas (ir jo pataisa, įforminta gana gremėzdiškai: „Seimo statutas dėl Seimo statuto pakeitimo”), kurį pasirašo Seimo Pirmininkas (ne Respublikos Prezidentas), įsigalioja kitą dieną po jo paskelbimo, jei jame nenustatyta kita įsigaliojimo tvarka. Be to, reikia neužmiršti, kad „galioja tik paskelbti įstatymai” (7 str. 2 d.).
PARLAMENTINIS APKALTOS PROCESAS Kalbant apie Seimą, tikslinga aptarti apkaltos procesą, pagal kurį Seimas 3/5 visų narių balsų dauguma (t. y. ne mažiau kaip 85 balsais) gali panaikinti bet kurio Seimo nario mandatą (teoriškai – ir Seimo Pirmininko, kuris, logiškai mąstant, prieš apkaltos pabaigą turėtų būti atstatydintas, jei pats savo noru neatsistatydina iš šių pareigų), pašalinti iš užimamų pareigų Respublikos Prezidentą ar trijų teismų (Konstitucinio Teismo, Aukščiausiojo Teismo ir Apeliacinio teismo) teisėjus, „šiurkščiai pažeidusius Konstituciją arba sulaužiusius priesaiką, taip pat paaiškėjus, jog padarytas nusikaltimas” (Konstitucijos 74 str.). Pažymėtina, kad Konstitucijoje nenumatyti subjektai, kurie gali inicijuoti apkaltą. Juos iki 2004 m. įvardijo Seimo statutas, kuriame buvo pasakyta, kad apkaltą gali inicijuoti ne mažiau kaip 1/4 visų Seimo narių, Respublikos Prezidentas, o dėl Aukščiausiojo Teismo ir Apeliacinio teismo teisėjų – taip pat ir teisėjų garbės teismas. Toks reglamentavimas pagal Konstitucinio Teismo 2004 m. išaiškinimą iš dalies prieštarauja Konstitucijai, nes parlamentinės apkaltos iniciatyva gali kilti tik iš Seimo narių (bet ne iš Respublikos Prezidento ar teismų savivaldos institucijos). Taigi dabar apkaltą gali siūlyti ne mažiau kaip ketvirtadalis parlamentarų, t. y. 36 Seimo nariai. Parodyta iniciatyva savaime nereiškia, kad apkaltos procesas yra pradėtas. Iniciatyvai turi pritarti posėdyje dalyvaujančių Seimo narių balsų dauguma (Seimo statuto 244 str., 245 str.). Pažymėtina ir tai, kad apkalta yra ne teismo procesas, kuriame gali būti pareiškiamas kal
110
Pagrindinės valstybės valdžios institucijos
tinimas ir skiriama kriminalinė bausmė, bei parlamentinė procedūra, kurios galimi teisiniai padariniai (pašalinimas iš pareigų, mandato atėmimas ir draudimas ateityje užimti renkamas valstybės politikų pareigas) yra konstitucinės sankcijos (bet ne baudžiamosios teisės sankcijos, t. y. bausmės). Be to, svarbus Konstitucinio Teismo išaiškinimas, kuriame sakoma, kad Seimas, atsižvelgdamas į tris skirtingus apkaltos pagrindus (šiurkštus Konstitucijos pažeidimas, priesaikos sulaužymas ir įtarimas padarius nusikaltimą), gali nustatyti nevienodą apkaltos organizavimą. Pavyzdžiui, Seimas gali pradėti supaprastintą apkaltą, jei generalinis prokuroras pareiškia Seime, kad paaiškėjo padarytas nusikaltimas. Tada Seimas leidžia patraukti Seimo narį (ar kurį nors teisėją, jei jis savo noru neatsistatydina) baudžiamojon atsakomybėn ir, atsisakęs atlikti reikiamą tyrimą (nes tai atliks prokurorai arba teismas), nusprendžia palaukti ir spręsti konstitucinės atsakomybės (mandato atėmimo, pašalinimo iš pareigų) klausimą apkaltos tvarka atsižvelgiant į tai, ar ikiteisminio ir teisminio tyrimo metu bus nustatyta, kad Seimo narys kaltas padaręs nusikaltimą. Tiesa, Konstitucinis Teismas išaiškino, kad supaprastinta apkalta prieštarauja Konstitucijai (teisinės valstybės principui), jei jos tvarka nenumato nuteistojo Seimo nario galimybės gintis Seime, t. y. dalyvaujant Seimo posėdyje teikti paaiškinimus ar įgyvendinti „paskutinės replikos teisę”. Tai ir lėmė, kad Seimo narys Audrius Butkevičius, nuteistas ir atliekantis laisvės atėmimo bausmę (bet dėl to nepraradęs mandato, nes Konstitucijos 63 straipsnyje, nurodančiame visus Seimo nario įgaliojimo pasibaigimo pagrindus, nėra žodžių „kai įsiteisėja jį apkaltinęs teismo nuosprendis”), 1999 m. įgijo konstitucinę teisę gintis, atvykęs iš laisvės atėmimo vietos į Seimo posėdį (kuriame supaprastintos apkaltos tvarka buvo siekiama balsuoti dėl jo mandato atėmimo), ir jame pateikti, jo manymu, svarbius paaiškinimus. Apkalta 1999 m. birželio 15 d. patyrė fiasko, nes už ją balsavo 70 Seimo narių, o reikėjo mažiausiai 85 balsų. Pažymėtina, kad Seimas, remdamasis Konstitucinio Teismo išvada, 2004 m. balandžio 6 d. pašalino Respublikos Prezidentą iš pareigų už šiurkštų Konstitucijos pažeidimą ir priesaikos sulaužymą pagal tris kaltinimus (už kuriuos atitinkamai balsavo 86, 86 ir 89 Seimo nariai). Šiuo atveju Seimas išsprendė politinį klausimą (pašalinti ar nepašalinti), o teisinį klausimą (dėl šiurkštaus Konstitucijos pažeidimo ir priesaikos sulaužymo fakto) prieš tai buvo galutinai išsprendęs Konstitucinis Teismas ir pateikęs išvadą. Taigi Seimo nariai negalėjo ginčytis (diskutuoti) dėl Konstitucinio Teismo išvados, kurioje galutinai buvo išspręstas teisinis klausimas (fakto nustatymas).
SEIMO NARIO KONSTITUCINĖ PADĖTIS Jau minėta, kad Seimo narys turi laisvą mandatą ta prasme, kad jis, vadovaudamasis Konstitucija, valstybės interesais ir savo sąžine, negali būti varžomas jokių kitų mandatų (59 str. 4 d.), t. y. politinių partijų, rinkėjų ar finansinių rėmėjų nurodymų, reikalavimų (taip pat Seimo nario įsipareigojimų ar pažadų jiems prieš rinkimus). Toks mandatas – tai priešingybė imperatyviam mandatui, kuris buvo įtvirtintas LTSR konstitucijoje (jo esmė – deputatas turi vykdyti rinkėjams duotus pažadus, vadinamuosius rinkėjų priesakus, o jei jis to nedaro, gali būti „bet kuriuo metu atšauktas daugumos rinkėjų sprendimu”; LTSR 1978 m. Konstitucijos 92 str., 96 str.).
111
KONSTITUCINĖ TEISĖ
Išrinktas Seimo narys visas parlamentines teises (pvz., teisę teikti įstatymų projektus, balsuoti) įgyja tik prisiekęs, kad jis bus ištikimas Lietuvos Respublikai (59 str. 2 d.). Jei kuris nors Seimo narys neprisiekia, Seimas priima nutarimą, pagal kurį jis netenka mandato. Pažymėtina, kad Seimo statute (4 str.) nustatytas terminas, per kurį Seimo narys privalo prisiekti. Taigi Seimo narys turi prisiekti ne vėliau kaip per vieną mėnesį nuo pirmojo posėdžio, kuriame jis dalyvavo. Seimo nario teisės yra tokios: 1) individualios, reiškiančios, kad kiekvienas Seimo narys turi teisę (nesvarbu, ar ji įgyvendinama drauge su kitais Seimo nariais) inicijuoti įstatymų leidybą (Konstitucijos 68 str. 1 d.) ar pateikti paklausimą Vyriausybės nariams, kitų institucijų, kurias sudaro Seimas, vadovams (Konstitucijos 61 str. 1 d.). Pateikti paklausimus Lietuvos Aukščiausiojo Teismo ar Konstitucinio Teismo pirmininkams negalima, nes tai iš esmės būtų kišimasis į teisėjo ar teismo veiklą, o tai yra draudžiama (Konstitucijos 114 str. 1 d.). Seimo statute (213 str.) yra detalizuota paklausimo sąvoka. Jame rašoma, kad paklausimu laikytinas toks rašytinis kreipimasis, kuriame reikalaujama pateikti informaciją apie tam tikros institucijos veiklą ir priimtus sprendimus ir keliamas klausimas, dėl kurio Seimo narys kreipėsi į valstybės institucijas, tačiau, jo nuomone, jis nebuvo tinkamai išnagrinėtas arba buvo išspręstas neigiamai; atsakymo į paklausimą terminas negali viršyti 10 dienų (jei paklausimo gavėjas siūlo ir paklausimą pateikęs Seimo narys sutinka – vieno mėnesio). Pažymėtina, kad paklausimas (ir atsakymas į jį) nagrinėjamas Seimo posėdyje; 2) kolektyvinės, reiškiančios, kad tam tikro gausumo Seimo narių grupė, pavyzdžiui, ne mažesnė kaip 1/5 Seimo narių, turi teisę kreiptis į Konstitucinį Teismą dėl visų jo jurisdikcijai priskirtų teisės aktų (Seimo, Vyriausybės ir Respublikos Prezidento aktų) konstitucingumo įvertinimo (Konstitucijos 106 str. 1-3 d.) ar pateikti sesijos metu interpeliaciją Ministrui Pirmininkui ar ministrui, kuria reikalaujama paaiškinti šių pareigūnų priimtų sprendimų motyvus (Konstitucijos 61 str. 2 d.). Seimo statute (220 str.) parašyta, kad Vyriausybės narys, gavęs interpeliaciją, privalo ne vėliau kaip per dvi savaites perduoti Seimo pirmininkui raštišką atsakymą, kuris Seimo sesijos metu apsvarstomas ne vėliau kaip per penkias darbo dienas. Apsvarstęs atsakymą, Seimas gali nutarti, kad jis nepakankamas ir pusės visų Seimo narių balsų dauguma pareikšti nepasitikėjimą Ministru Pirmininku ar ministru, šitaip priverčiant jį atsistatydinti (Konstitucijos 61 str. 3 d.). Reikėtų aptarti ir Seimo nario veiklos garantijas (teisės normomis išreikštas organizacines ir finansines priemones). Konstitucijoje numatyta: 1) piniginio atlyginimo garantija (60 str. 3 d.), pagal kurią Seimo nario darbas ir su juo susijusios išlaidos (pvz., atsirandančios reguliariai vykstant susitikti su rinkėjais) atlyginamos iš valstybės biudžeto (Konstitucijos 60 str. 2 d.). Pažymėtina, kad Konstitucija (60 str. 2 d.) draudžia Seimo nariui gauti kitą (papildomą) atlyginimą, išskyrus atlyginimą už kūrybinę veiklą. Seimo statute (15 str.) tai detalizuota. Jame parašyta, kad „atlyginimu už kūrybinę veiklą laikomas autorinis atlyginimas už meno kūrinius bei jų atlikimą, už publikacijas bei knygas, už medžiagą radijo ir televizijos laidoms, taip pat atlygis už pedagoginę ir mokslinę veiklą ne Seimo, jo komitetų ir komisijų posėdžių metu”; 2) Seimo nario asmens neliečiamumo (imuniteto) garantija (62 str. 1 d.), pagal kurią negalima, viena vertus, Seimo nario patraukti baudžiamojon atsakomybėn ir suimti ar kitaip suvaržyti laisvę be Seimo sutikimo (Konstitucijos 62 str. 2 d.), o
112
Pagrindinės valstybės valdžios institucijos
antra vertus, jo persekioti už balsavimus ar kalbas (62 str. 3 d.). Tai negalioja, kai Seimo narys šmeižia ar įžeidinėja kitą asmenį. Įžeidimą reikia suprasti kaip tyčinį Seimo posėdžio ar komiteto posėdžio dalyvio garbės ir orumo įžeidimą žodžiu, raštu ar veiksmu (pvz., antausiu), o šmeižtą – kaip skleidimą melagingų prasimanymų, tiesos neatitinkančių žinių, žeminančių kito asmens garbę ir orumą. Kitos garantijos (pvz., gyvenimas Seimo viešbutyje) detalizuotos Seimo narių darbo sąlygų įstatyme. Seimo nario ketverių metų įgaliojimų laikas pradedamas skaičiuoti, kai naujas išrinktas Seimas susirenka į pirmąjį posėdį. Šis laikas baigiasi, kai į pirmąjį posėdį susirenka kitas, naujas, Seimas (Konstitucijos 59 str. 1 d.). Seimo nario įgaliojimai nutrūksta pirma laiko, kai jis atsistatydina, miršta, Seimo rinkimai pripažįstami negaliojančiais (tai gali pripažinti Seimas, remdamasis Konstitucinio Teismo išvada, kurį pateikiama per 72 valandas, kai to paprašo Seimas ar Respublikos Prezidentas; Konstitucijos 105 str. 3 d. 1 p., 106 str. 5 d.) ir kitais Konstitucijos 63 straipsnyje numatytais atvejais.
SEIMO KONTROLIERIAI Kalbant apie Seimą pažymėtini ir Seimo kontrolieriai (politologijoje ar lyginamosios teisės moksluose jie vadinami ombudsmenais; tokia specifinė kontrolės idėja pirmiausia įgyvendinta Skandinavijos valstybėse). Jų pareiga – tirti „piliečių skundus dėl valstybės ir savivaldybių pareigūnų (išskyrus teisėjus) piktnaudžiavimo ar biurokratizmo” (Konstitucijos 73 str. 1 d.). Įstatymų leidėjas, priėmęs Seimo kontrolierių įstatymą, nustatė, kad piktnaudžiavimas reiškia piktnaudžiavimą tarnybine padėtimi. Į piktnaudžiavimo sąvoką įeina: 1) savivalė, t. y. kai veikiama ar neveikiama neturint tam įgaliojimų arba viršijant turimus įgaliojimus; 2) savanaudiški tikslai, t. y. kai veikiama ar neveikiama siekiant pasisavinti ar kitiems perleisti svetimas lėšas ar kitokį turtą; 3) subjektyvios asmeninės paskatos, t. y. kai veikiama ar neveikiama dėl keršto, pavydo, karjerizmo ar pan. Biurokratizmas reiškia pareigūno veikimą ar neveikimą, kai „sistemingai laikomasi nereikalingų ar išgalvotų formalumų, nepagrįstai atsisakoma spręsti […] pavaldžius dalykus, vilkinama priimti sprendimus”. Pažymėtina, kad biurokratizmu įstatyme laikomas ir toks neveikimas, kai „atsisakoma informuoti asmenį apie jo teises” (Seimo kontrolierių įstatymo 2 str. 3 d.). Konstitucija tiesiogiai įtvirtina tik vieną Seimo kontrolierių įgaliojimą – teisę siūlyti teismui atleisti kaltus pareigūnus iš užimamų pareigų. Tačiau Seimo kontrolierių įstatymas (4 str. 2 d.) praplečia Konstitucijos išlygą „išskyrus teisėjus”, numatydamas Seimo kontrolierių nesikišimą ir į ikiteisminio tyrimo pareigūnų procesinius sprendimus (t. y. į anksčiau vadintų tardytojais ir kvotėjais, kuriems skirtų Konstitucijos nuostatų neliko po Konstitucijos 118 straipsnio 2003 m. kovo 20 d. pataisos, procesinę veiklą). Be to, pažymėtina, kad įstatymas neleidžia Seimo kontrolieriams tirti Respublikos Prezidento, Seimo narių, Ministro Pirmininko, valstybės kontrolieriaus ir kolegialius sprendimus priimančių Vyriausybės bei savivaldybių tarybų veiklos. Verta dėmesio įstatymo nuostata, pagal kurią: 1) „anoniminiai skundai nenagrinėjami, jeigu Seimo kontrolierius nenusprendžia kitaip”; 2) skundai, paduoti praėjus vienerių metų terminui nuo skundžiamų veiksmų padarymo, „nenagrinė
113
KONSTITUCINĖ TEISĖ
jami, jeigu Seimo kontrolierius nenusprendžia kitaip”; 3) Seimo kontrolierius gali pareikšti ieškinius teisme, kad būtų arba kompensuojami skundo autoriaus moraliniai ir materialiniai nuostoliai, arba kalti pareigūnai būtų atleisti iš užimamų pareigų; 4) Seimo kontrolierius turi būti nepriekaištingos reputacijos, turėti aukštąjį teisinį išsimokslinimą ir ne mažesnį kaip 5 metų teisinio darbo arba darbo valstybės valdžios bei valdymo institucijose (taigi nebūtinai tik teisinio darbo siauruoju požiūriu) stažą (Seimo kontrolierių įstatymo 5 str., 17 str., 18 str., 22 str.).
4.2. RESPUBLIKOS PREZIDENTAS IR VYRIAUSYBĖ
RESPUBLIKOS PREZIDENTAS IR VYRIAUSYBĖ KAIP VYKDOMOJI VALDŽIA Iš pirmo žvilgsnio Respublikos Prezidentas ir Vyriausybė yra atskiros ir pagal kompetenciją savarankiškos institucijos. Tačiau iš Konstitucijos 5 straipsnio 1 dalies nuostatos matyti, kad minėtosios institucijos labiausiai simbolizuoja vykdomąją valdžią, tiksliau – atlieka valdymo (administravimo) funkciją. Tai turint omeny galima geriau suprasti, kodėl: 1) skiriant ir atleidžiant ministrus tiesiogiai nedalyvauja Seimas – tai sprendžia du subjektai: Ministras Pirmininkas, teikdamas ministrų kandidatūras, ir Respublikos Prezidentas, pritardamas ar nepritardamas pateiktoms kandidatūroms (Konstitucijos 92 str. 2 d.); 2) Ministras Pirmininkas ar tam tikros valdymo srities ministras privalo pasirašyti (kontrasignuoti) kai kuriuos Respublikos Prezidento dekretus (Konstitucijos 85 str.); 3) Respublikos Prezidentas gali kreiptis į Konstitucinį Teismą tik dėl Vyriausybės aktų, t. y. tų aktų, kurie priskirti jo jurisdikcijai, konstitucingumo (Konstitucijos 106 str. 3 d.). Visa tai nereiškia, kad Vyriausybė yra pavaldi Respublikos Prezidentui.
RESPUBLIKOS PREZIDENTO KONSTITUCINĖ PADĖTIS Konstitucijos nuostatos leidžia Respublikos Prezidentą vertinti ir kaip valstybės valdžios instituciją (5 str. 1 d.), kuri, pavyzdžiui, pateikdama metinį pranešimą Seime (84 str. 18 p.) neprivalo turėti ar gauti Seimo, tuo labiau Vyriausybės pasitikėjimą. Konstitucijos nuostatos leidžia Respublikos Prezidentą vertinti ir kaip pareigūną – valstybės vadovą (77 str. 1 d.), kuris: 1) gali būti perrinktas penkerių metų kadencijai tik vieną kartą iš eilės (78 str. 3 d.); 2) privalo sustabdyti savo veiklą politinėje partijoje (83 str. 2 d.) ir todėl turi būti iš dalies depolitizuotas. Pažymėtina, kad Respublikos Prezidentas turi ypatingą asmens neliečiamumo garantiją: jo imunitetas platesnis nei Seimo nario ar ministro, nes yra visiškai ribojama galimybė patraukti jį administracinėn atsakomybėn. Šiuo atveju svarbus Konstitucinio Teismo išaiškinimas, kad Operatyvinės veiklos įstatyme numatytas nusikalstamą veiklą imituojantis elgesio modelis (t. y. „nusikaltimo inscenizavimas”) gali būti panaudotas prieš Seimo narį, bet negali būti panaudotas prieš Respublikos Prezidentą (Konstitucinio Teismo 2000 m. gegužės 8 d. nutarimas). Paprasčiau tariant, Konstitucija neleidžia prokurorams ar kitiems pareigūnams imitaciniais veiksmais provokuoti Respublikos Prezidentą daryti nusikaltimą.
114
vienasmenę
Pagrindinės valstybės valdžios institucijos
RESPUBLIKOS PREZIDENTO KOMPETENCIJA Įvairiapusę valstybės vadovo kompetenciją rodo jo konstitucinių įgaliojimų sąrašas Konstitucijos 84 straipsnyje. Jame išvardyti net 24 punktai. Respublikos Prezidento įgaliojimus galima susisteminti į 5 grupes: 1. Įgaliojimai, susiję su užsienio politika ir tarptautinėmis sutartimis. Juos įgyvendindamas Respublikos Prezidentas pasirašo ir teikia Seimui ratifikuoti Lietuvos Respublikos tarptautines sutartis, Vyriausybės teikimu skiria (atšaukia) Lietuvos diplomatinius atstovus ir sprendžia „pagrindinius užsienio politikos klausimus” (84 str. 1-3 p.). Kuris užsienio politikos klausimas yra pagrindinis ir kuris nepagrindinis tai ne tiek konstitucinis, kiek Vyriausybės, Respublikos Prezidento ir Seimo, kuris „svarsto kitus užsienio politikos klausimus” (97 str. 16 p.), nusistovėjusios praktikos ir partnerystės klausimas. Konstitucinis Teismas išaiškino, kad Respublikos Prezidentas turi išimtinę teisę teikti ratifikuoti Lietuvos Respublikos tarptautines sutartis Seimui, tuo tarpu pasirašyti tokias sutartis gali ir jis, ir Ministras Pirmininkas (Konstitucinio Teismo 1995 m. spalio 17 d. nutarimas). 2. Įgaliojimai, susiję su Vyriausybės formavimu ir jos veikla bei to nulemti santykiai su Seimu (84 str. 4-9 p.). Apie tokius įgaliojimus šiame knygos skirsnyje bus kalbama toliau, aptariant Vyriausybės sudarymo tvarką. 3. Įgaliojimai, susiję su valstybės pareigūnų skyrimu (atleidimu) bei to nulemti santykiai su Seimu. Respublikos Prezidentas Seimo pritarimu (dėl to Respublikos Prezidentui būtina dekretu kreiptis į Seimą, pateikiant siūlomą kandidatūrą) skiria kariuomenės vadą ir saugumo tarnybos (ją įstatymų leidėjas pavadino Valstybės saugumo departamentu) vadovą. Be to, Respublikos Prezidentas teikia Seimui (kad šis skirtų) valstybės kontrolieriaus ir Lietuvos banko valdybos pirmininko kandidatūras (84 str. 10-14 p.). 4. Įgaliojimai, susiję su vadovavimu ginkluotosioms pajėgoms ir valstybės gynybai (84 str. 15-18 p.). Čia pažymėtina ir tai, kad Respublikos Prezidentas yra Valstybės gynimo tarybos, kuri svarsto ir koordinuoja svarbiausius valstybės gynybos klausimus, vadovas (140 str.). 5. Mišrūs įgaliojimai. Paskutiniai, t. y. 19-24 punktai rodo, jog valstybės vadovas atlieka integravimo funkciją (ją išreiškia malonės, apdovanojimų ir pilietybės teikimas), sąlyginę „notaro” funkciją (t. y. įstatymų promulgavimą) bei organizavimo funkciją (nustato Seimo rinkimų datą, šaukia neeilinę Seimo sesiją).
VYRIAUSYBĖS SUDARYMAS Vyriausybės sudarymo procedūra sudėtinga. Joje dalyvauja dvi institucijos (Respublikos Prezidentas ir Seimas) ir iš dalies trečias subjektas – Ministras Pirmininkas. Be to, ji vyksta keliais etapais: 1. Respublikos Prezidentas dekretu teikia Ministro Pirmininko kandidatūrą Seimui. Šiuo atveju Respublikos Prezidentas kaip valstybės vadovas (Konstitucijos 77 str. 1 d.) iš principo turi vengti nestabilumo ar neprognozuojamos situacijos, t. y. parinkti ir teikti tik tokį kandidatą, kuriam akivaizdžiai pritars Seimo narių dauguma. Kaip Respublikos Prezidentas sužino naujai išrinkto Seimo politinės daugumos nuomonę ar pageidavimus? Tikėtina, kad iš pokalbių ar derybų su daugiau
115
KONSTITUCINĖ TEISĖ
sia rinkėjų balsų surinkusių Seimo narių grupių, spėjusių ar dar nespėjusių susiburti į frakciją ar frakcijas, atstovais. Pagal Seimo statutą (192 str.) Seimo posėdyje „klausimai Respublikos Prezidentui, po to, kai jis pristato Ministro Pirmininko kandidatūrą, neužduodami”; pristatytas kandidatas atsako į Seimo narių klausimus (iki 40 min.), ir ne vėliau kaip per savaitę nuo kandidatūros pristatymo turi būti surengtas kitas Seimo posėdis sprendimui priimti. 2. Seimas nutarimu pritaria arba nepritaria Respublikos Prezidento pateiktai Ministro Pirmininko kandidatūrai. Iki šiol nebuvo situacijos, kad Seimas nepritartų. Jei taip atsitiktų, Respublikos Prezidentas turėtų iš naujo teikti kitą kandidatą. 3. Respublikos Prezidentas dekretu skiria kandidatą, gavusį Seimo pritarimą, Ministru Pirmininku. 4. Ministras Pirmininkas teikia (raštišku kreipimusi ar kitokiu dokumentu – tai nėra detaliai reglamentuota) Respublikos Prezidentui ministrų kandidatūras, o Respublikos Prezidentas, jei sutinka su jomis, dekretais skiria pateiktus kandidatus į ministrus (jų skaičių lemia ministerijų skaičius; ministerijų sąrašas pateiktas Vyriausybės įstatyme) ir atskiru dekretu tvirtina visos Vyriausybės sudėtį (Konstitucijos 92 str. 2 ir 3 d.). Šiuo atveju pažymėtina, kad Konstitucijoje ir įstatymuose nereglamentuota, kokia tvarka sprendžiami nesutarimai, galintys iškilti Respublikos Prezidentui nepritarus pateiktoms ministrų kandidatūroms. Ar valstybės vadovas būna politiškai aktyvus parenkant ministrus, pageidaujant alternatyvių kandidatūrų ar kitaip darant įtaką Vyriausybės sudėčiai, priklauso nuo jo asmenybės ar politinės strategijos. 5. Seimas nutarimu pritaria ar nepritaria Ministro Pirmininko per 15 dienų nuo jo paskyrimo pristatytai (raštiniu kreipimusi ar kitokiu dokumentu – tai nėra detaliai reglamentuota) svarstyti Vyriausybės programai. Ši programa pagal Konstitucinio Teismo išaiškinimą yra „teisinis dokumentas, kuriame išdėstytos tam tikro laikotarpio valstybės veiklos gairės”; jis turi būti skelbiamas „Valstybės žiniose” (Konstitucinio Teismo 1998 m. sausio 10 d. nutarimas). Pažymėtina, kad Vyriausybė laikoma gavusi įgaliojimus veikti (tampa legitimuota), jei jos programai pritaria dauguma Seimo posėdyje dalyvaujančių Seimo narių (Konstitucijos 90 str. 5 d.). Tokį reikalavimą legitimacijai gauti galima vertinti kritiškai, nes pagal tautos suvereniteto principą Vyriausybės programai iš esmės turėtų pritarti dauguma visų Seimo narių, o ne 30, 50 ar 70 posėdyje dalyvaujančių narių.
VYRIAUSYBĖS KOMPETENCIJA Vyriausybės įgaliojimai Konstitucijoje įtvirtinti tik 7 punktais (žr. 94 str.). Taigi susidaro klaidingas įspūdis, kad Seimo ir Respublikos Prezidento įgaliojimai, išreikšti gausybe punktų, yra svarbesni ar reikšmingesni valdant. Akivaizdu, kad Vyriausybės įgaliojimas tvarkyti krašto reikalus (Konstitucijos 94 str. 1 p.) apima daug veiklos barų, kuriuos galima būtų apibūdinti daugybe punktų. Šis pavyzdys rodo, kad Vyriausybė – bendrosios (valdymo) kompetencijos institucija – gali imtis įvairių iniciatyvų, jei jos tik išplaukia iš Vyriausybės veiklos programos ir nepažeidžia Seimo išskirtinės prerogatyvos reglamentuoti svarbiausius žmogaus teisių ir laisvių bei kitų konstitucinių vertybių įgyvendinimo ar ribojimo klausimus. Vyriausybės įgaliojimas ir įpareigojimas vykdyti įstatymus (94 str. 2 p.) rodo, kad Vyriausybė yra vykdomosios valdžios institucija, kuri, nors ir būdama pagal kompetenciją savarankiška, privalo vykdyti ir tai, kam ji nepritaria ar pritaria iš
116
Pagrindinės valstybės valdžios institucijos
dalies. Jei Vyriausybė to nedaro, svarbią reikšmę įgauna jos parlamentinė kontrolė Seimo narių įgaliojimai kontroliuoti Vyriausybę, teikiant paklausimus ar interpeliacijas, kraštutiniu atveju – Seimui reiškiant nepasitikėjimą Ministru Pirmininku ar Vyriausybe (Konstitucijos 101 str. 3 d. 2 p.). Svarbus šiuo atveju ir Konstitucinio Teismo išaiškinimas, kad „Vyriausybės pareiga priimti poįstatyminius aktus, būtinus įstatymams įgyvendinti, kyla iš Konstitucijos, o esant įstatymų leidėjo pavedimui – ir iš įstatymų bei Seimo nutarimų dėl įstatymų įgyvendinimo” (Konstitucinio Teismo 2001 m. spalio 30 d. nutarimas). Vienas pagrindinių Vyriausybės, kaip valdžios, įgaliojimų yra rengti (sudaryti) valstybės biudžetą, kuris pagal Konstitucinio Teismo išaiškinimą yra „centralizuotas finansų išteklių fondas, kuriame sukaupiama ir perskirstoma dalis nacionalinių pajamų” (Konstitucinio Teismo 1996 m. vasario 28 d. nutarimas). Vykdydama tokį įgaliojimą, Vyriausybė stengiasi įgyvendinti savo politinius prioritetus, nors ji negali ignoruoti ir tokių Konstitucinio Teismo išaiškinimų: „Vyriausybė, rengdama valstybės biudžeto projektą, turi atsižvelgti į […] Seimo, Respublikos Prezidento, Teismų pateiktus asignavimų iš valstybės biudžeto […] dydžius”; „jeigu Vyriausybės parengtame valstybės biudžeto projekte Seimui, Respublikos Prezidentui, Teismams būtų numatyta akivaizdžiai mažiau, negu jų reikia […], būtų sudarytos prielaidos pažeisti konstitucinį valdžių padalijimo principą ir Konstitucijos 5 straipsnį” (Konstitucinio Teismo 2002 m. liepos 11d. nutarimas). Sudarytą biudžetą įstatymu patvirtina Seimas (jam Vyriausybė biudžeto projektą turi pateikti ne vėliau kaip prieš 75 dienas iki biudžetinių metų pabaigos, t. y. gruodžio 31 dienos; Konstitucijos 129 str., 130 str.). Dar vienas Vyriausybės įgaliojimas – ministerijų koordinavimas (Konstitucijos 94 str. 3 p.), kuris rodo klasikinį centralizuotą valstybės valdymą. Tai reiškia, kad Vyriausybės veikla jos posėdžiais nesiriboja ir toliau pasireiškia Ministro Pirmininko, vadovaujančio Vyriausybei (Konstitucijos 97 str.), ir ministrų, vadovaujančių ministerijoms (Konstitucijos 98 str.), bendradarbiavimu.
4.3. TEISMAI
TEISMO SAMPRATA Pagal Konstitucijos nuostatą „Valstybės valdžią Lietuvoje vykdo […] Teismas” (5 str. 1 d.) sunku kategoriškai teigti, kad žodis „Teismas” apima ir Lietuvos Respublikos Konstitucinį Teismą, ir visus kitus teismus, numatytus Konstitucijos 111 straipsnio 1 ir 2 dalyse. 1 dalyje minimi Lietuvos Aukščiausiasis Teismas, Lietuvos apeliacinis teismas, apygardos ir apylinkės teismai – visus juos įstatymų leidėjas vadina bendrosios kompetencijos teismais (Teismų įstatymo 12 str.). 2 dalyje numatyta galimybė steigti specializuotus teismus administracinėms, darbo, šeimos ar kitų kategorijų byloms nagrinėti (iš tokių neįsakmiai numatytų teismų dabar įsteigti tik administraciniai teismai). Tokio keblumo nebūtų, jei Konstitucijos IX skirsnyje „Teismas” būtų įterptos nuostatos, susijusios su Konstitucinio Teismo teisine padėtimi. Tačiau tokios nuostatos pateiktos atskirame (VIII) skirsnyje „Konstitucinis Teismas”. Be to, Konstitucijoje (104 str. 4 d.) teigiama, kad Konstitucinio Teismo teisėjų asmens neliečiamybė yra adekvati ne kitų teisėjų, bet Seimo narių asmens neliečiamybei. Visa tai leidžia teigti, kad: 1) siauresniu požiūriu Konstitucinis Teis
117
KONSTITUCINĖ TEISĖ
mas yra specifinė valstybės teisminės kontrolės institucija, kuri nėra bendrosios teismų sistemos grandis; 2) platesniu požiūriu Konstitucinis Teismas tiek pagal savo įgaliojimus, tiek pagal pavadinimą yra teismas tikrąja to žodžio prasme, nes yra konstitucinio teisingumo vykdymo institucija.
KONSTITUCINIS TEISMAS Konstitucinį Teismą sudaro devyni devyneriems metams Seimo paskirti teisėjai, kurių kandidatūras iš nepriekaištingos reputacijos piliečių, turinčių „ne mažesnį kaip 10 metų teisinio ar mokslinio pedagoginio darbo pagal teisininko specialybę stažą” (Konstitucijos 103 str. 3 d.) teikia trys subjektai: Respublikos Prezidentas, Seimo Pirmininkas ir Aukščiausiojo Teismo pirmininkas. Kas treji metai trys teismo teisėjai pakeičiami, todėl teisėjų kadencijos pabaiga nesutampa. Pirmą kartą (1993 m.) skiriant teisėjus trys iš jų buvo paskirti devyneriems, trys – šešeriems, trys – trejiems metams. Toks reguliarus teismo atnaujinimas vienu trečdaliu garantuoja konstitucinės jurisprudencijos (t. y. teismo teiginių ir koncepcijų sistemos) tęstinumą ir leidžia išvengti nestabilumo, kuris galėtų kilti, jei iš karto pasikeistų visi teisėjai. Konstitucinio Teismo kompetencija dvilypė. Vienav ertus, jis tiria ir sprendžia, ar atitinka Konstituciją Seimo, Respublikos Prezidento ir Vyriausybės aktai (įstatymai, Seimo statutas, Seimo nutarimai, Respublikos Prezidento dekretai, Vyriausybės nutarimai) ir kiti aktai, pavyzdžiui, Vyriausybės posėdyje priimtas sprendimas, kuris nėra įformintas kaip nutarimas, bet įrašytas į posėdžio darbotvarkę (šiuo atveju vykdoma vadinamoji abstrakčioji, t. y. konkretaus ginčo ar gyvenimiškos situacijos nesprendžianti, Konstitucijos ir ginčijamo akto normų atitikties kontrolė). Tai padaręs (t. y. išnagrinėjęs bylą), Konstitucinis Teismas priima nutarimą, kuris yra ne tik galutinis ir neskundžiamas (Konstitucijos 107 str. 2 d.), bet ir sukelia konkretų padarinį: po oficialaus nutarimo paskelbimo („Valstybės žiniose”, kituose leidiniuose ar per informacinę agentūrą ELTA) antikonstituciniu pripažintas aktas negali būti taikomas (aišku, jei jis nėra vienkartinio taikymo; Konstitucijos 107 str. 1 d.). Antra vertus, Konstitucinis Teismas teikia išvadas dėl: 1) įtariamo Seimo rinkimų įstatymo ir Respublikos Prezidento rinkimų įstatymo pažeidimo per tam tikrus rinkimus; 2) abejojamos Respublikos Prezidento sveikatos būklės ir jo užimamų pareigų atitikties; 3) abejojamos Lietuvos Respublikos tarptautinių sutarčių ir Konstitucijos atitikties; 4) asmenų, prieš kuriuos Seime dėl šiurkštaus Konstitucijos pažeidimo pradėta apkaltos procedūra, konkrečių veiksmų prieštaravimo Konstitucijai (Konstitucijos 105 str. 3 d.). Šiais atvejais Konstitucinio Teismo išvados, nors ir yra „galutiniai ir neskundžiami” teisinių klausimų sprendimai (Konstitucijos 107 str. 2 d.), savaime nesukelia konkrečių padarinių, nes „remdamasis Konstitucinio Teismo išvadomis, […] klausimus galutinai politiškai sprendžia Seimas” (Konstitucijos 107 str. 3 d.). Tiesa, Seimas negali kvestionuoti Konstitucinio Teismo išvados, t. y. rengti diskusijas ar ginčus dėl išvados pagrįstumo, objektyvumo ir pan. Pažymėtina, kad Konstitucinio Teismo nutarimai negalioja atgaline data. Tai reiškia, kad įstatymo, kurį pirmiau taikant buvo apribotos konstitucinės asmenų teisės (pvz., pensinio amžiaus asmenų teisės laisvai pasirinkti darbą neprarandant teisėtai priklausančios pensijos dalies) ar susiaurinti konstituciniai principai (pvz., teisėjų nepriklausomumo principas, pažeistas sumažinus teisėjų darbo užmokestį), pripažinimas prieštaraujančiu Konstitucijai savaime nereiškia, kad: 1) naikinami visi
118
Pagrindinės valstybės valdžios institucijos
teisiniai santykiai, susiformavę taikant antikonstitucinį įstatymą; 2) visiems asmenims, kurių konstitucinės teisės (ar iš jų kylantys teisėti interesai) buvo pažeistos taikant antikonstitucinį įstatymą, pagal jų reikalavimus teismai ar valdymo institucijos privalo kompensuoti materialinę ar moralinę žalą (pvz., išmokėti visą pensiją ar teisėjo darbo užmokesčio dalį, negautą pritaikius antikonstitucinį įstatymą). Tiesa, Konstitucinis Teismas, nagrinėdamas Lietuvos Respublikos pilietybės neteisėto suteikimo Jurij Borisov bylą, išaiškino, kad jo nutarimo negaliojimas atgaline data nėra absoliutus. Vadinasi, gali būti išimtys, pavyzdžiui, minėto J. Borisov pripažinimas nesančiu Lietuvos Respublikos piliečiu, nes jam pilietybę Respublikos Prezidentas suteikė pažeisdamas Konstituciją ir įstatymus (Konstitucinio Teismo 2003 m. gruodžio 30 d. nutarimas). Visa tai – vienas sudėtingiausių konstitucinės teisės klausimų ir moksliniu, ir praktiniu požiūriu. Inicijuoti ginčijamo akto konstitucingumo patikrinimą gali keturi subjektai: 1) Seimas, kaip kolegiali institucija, priimdamas nutarimą dėl kreipimosi į Konstitucinį Teismą. Pažymėtina, kad Seimas gali kreiptis dėl visų aktų, priklausančių Konstitucinio Teismo jurisdikcijai, konstitucingumo, o jo kreipimasis sustabdo ginčijamo akto galiojimą (Konstitucijos 107 str. 4 d.); 2) Respublikos Prezidentas, kuris įgaliotas kreiptis tik dėl Vyriausybės aktų. Respublikos Prezidentas negali inicijuoti įstatymų konstitucingumo patikrinimo. Jo dekretas, kuriuo kreipiamasi, gali sustabdyti tik ginčijamo Vyriausybės akto galiojimą (aišku, jei toks aktas nėra vienkartinio taikymo); 3) ne mažiau kaip 1/5 visų Seimo narių, t. y. ne mažiau kaip 29 Seimo nariai, galintys kreiptis dėl aktų, priklausančių Konstitucinio Teismo jurisdikcijai, konstitucingumo; 4) bendrosios kompetencijos ar administraciniai teismai, sustabdę nagrinėjamą bylą dėl joje taikytino akto konstitucingumo. Jie gali kreiptis dėl visų aktų, priklausančių Konstitucinio Teismo jurisdikcijai, konstitucingumo (106 str., 110 str. 2 d.). Inicijuoti išvados teikimą gali du subjektai: 1) Seimas, galintis kreiptis dėl visų keturių minėtų išvadų, ir 2) Respublikos Prezidentas, turintis galimybę kreiptis dėl dviejų išvadų – Seimo rinkimų įstatymo pažeidimo ir Lietuvos Respublikos tarptautinės sutarties konstitucingumo. Pažymėtina, kad Konstitucija nesuteikia teisės kreiptis į Konstitucinį Teismą: 1) kiekvienam žmogui, manančiam, kad valstybės institucija pažeidė jo prigimtines (ar konstitucines pilietines) teises ir laisves; 2) savivaldybėms, universitetams, valstybės pripažintoms religinėms organizacijoms ar kitiems asmenims, turintiems konstitucinį statusą ir manantiems, kad šį statusą pažeidė įstatymų leidėjas ar kita valstybės institucija. Tiesa, tiek kiekvienas žmogus, tiek ir bet kuri organizacija gali netiesiogiai inicijuoti bylos svarstymą Konstituciniame Teisme, t. y. argumentuotai paprašyti, kad bendrosios kompetencijos ar administracinis teismas sustabdytų bylos nagrinėjimą ir kreiptųsi į Konstitucinį Teismą, prašydamas įvertinti byloje taikytino akto konstitucingumą (110 str. 2d.). Konstitucinio Teismo įgaliojimų vykdymas detalizuotas Konstitucinio Teismo įstatyme. Jame pažymėta: 1) bylos nagrinėjimas turi būti baigtas ir galutinis nutarimas priimtas ne vėliau kaip per 4 mėnesius nuo prašymo gavimo dienos, jeigu Konstitucinis Teismas nenustato kitaip (29 str. 2 d.). Taigi įstatyme iš esmės nenustatytas maksimalus terminas bylai išnagrinėti; 2) byla gali būti nagrinėjama tik kolegialiame posėdyje, jei jame dalyvauja ne mažiau kaip 2/3 visų jo teisėjų (19 str. 1 d.); 3) visi „sprendimai priimami ne mažesne kaip pusės posėdyje dalyvaujančių teisėjų balsų
119
KONSTITUCINĖ TEISE
dauguma” (19 str. 3 d.); 4) teisėjai neturi teisės paskelbti „nuomonių, pareikštų pasitarimų kambaryje” (53 str. 3 d.); 5) teisėjas už pareigų nevykdymą ir nedalyvavimą teismo posėdžiuose be pateisinamos priežasties negali būti traukiamas drausminėn atsakomybėn: už tokius veiksmus jam Konstitucinio Teismo (bet ne jo pirmininko) sprendimu gali būti taikomos materialinės sankcijos, t. y. iki 50 procentų sumažintas atlyginimas už praėjusį mėnesį (12 str.). Kalbant apie Konstitucinį Teismą, verta atkreipti dėmesį į Teismų įstatymo (60 str.) nuostatą, kurioje sakoma, kad buvęs Konstitucinio Teismo teisėjas „be egzamino ir atrankos gali būti paskirtas” bet kurio bendrosios kompetencijos ar administracinio teismo (tiek Aukščiausiojo, tiek apylinkės) teisėju.
BENDROSIOS KOMPETENCIJOS TEISMAI Teismų sistema (siauresniu požiūriu – be Konstitucinio Teismo) nustatyta Konstitucijos 111 straipsnyje. Ten nevartojamas žodžių junginys „bendrosios kompetencijos teismai”. Teismų įstatyme yra detalizuota, kad Aukščiausiasis Teismas, Apeliacinis teismas, apygardos ir apylinkės teismai yra „bendrosios kompetencijos teismai, nagrinėjantys civilines ir baudžiamąsias bylas” (12 str. 2 d.). Ten nurodytos ir išimtys, rodančios, jog nereikėtų sureikšminti teiginio, kad aptariami teismai nagrinėja tik civilines ir baudžiamąsias bylas. Taigi: 1) apylinkės teismai nagrinėja ir jų kompetencijai įstatymų priskirtas administracinių teisės pažeidimų bylas; 2) kiekvienas bendrosios kompetencijos teismas, nagrinėdamas civilinę bylą, kartu gali nuspręsti ir dėl individualaus administracinio akto teisėtumo (Teismų įstatymo 12 str. 3 d.). Konstitucijoje nedetalizuojama nė vieno bendrosios kompetencijos teismo kompetencija, struktūra (skyrių sudarymas) ar teisėjų skaičius. Tai nustatyti įgaliojamas įstatymų leidėjas (Konstitucijos 111 str. 4 d.). Jis ir nustatė, kad Lietuvos Aukščiausiajame Teisme turi būti Civilinių ir Baudžiamųjų bylų skyriai, į kuriuos teisėjus paskiria šio teismo Pirmininkas (atsižvelgdamas į teisėjų darbo krūvį skyriuose), ir Senatas, į kurį įeina šio teismo pirmininkas (jis pirmininkauja Senate), minėtų skyrių pirmininkai ir po septynis abiejų skyrių didžiausią Aukščiausiojo Teismo teisėjo darbo stažą turinčius teisėjus (Teismų įstatymo 22 str. 2 d. ir 3 d., 24 str.). Kalbant apie Aukščiausiojo Teismo statusą, pažymėtina, kad šis teismas: 1) yra vienintelis kasacinės instancijos teismas įsiteisėjusiems bendrosios kompetencijos teismų sprendimams (nuosprendžiams, nutartims, nutarimams ir įsakymams) peržiūrėti; 2) formuoja vienodą teismų praktiką, skelbdamas savo biuletenyje nutartis, į kurias privalo atsižvelgti teismai ir valdymo institucijos. Tokios nutartys gali būti trejopos: trijų teisėjų kolegijos nutartys, išplėstinės septynių teisėjų kolegijos nutartys bei skyrių plenarinių sesijų nutartys. Pažymėtina, kad Aukščiausiojo Teismo leidžiamame biuletenyje „Teismų praktika” skelbiamos ne visos nutartys, bet tik tos, kurias skelbti pritarė dauguma atitinkamo skyriaus teisėjų. Taip pat pažymėtina, kad minėtas Aukščiausiojo Teismo Senatas tvirtina teismų praktikos apibendrinimo apžvalgas ir teikia rekomendacinius išaiškinimus. Antai Senato 1997 m. birželio 13 d. nutarime Nr. 5 išaiškinta, kad motyvuojamojoje sprendimo dalyje „teismas neturi apsiriboti vien tik įrodymų išvardinimu, o privalo, nurodydamas įrodomąją reikšmę, išdėstyti jų turinį” (Teismų praktika. Nr. 7. P.13-22). Be to, Aukščiausiasis Teismas turi įgaliojimą analizuoti ir apibendrinti bendrosios kompetencijos teismų praktiką taikydamas Europos Sąjungos teisės normas.
120
Pagrindinės valstybės valdžios institucijos
Aukščiausiajame Teisme bylas nagrinėja trijų teisėjų kolegija, išplėstinė septynių teisėjų kolegija arba šio teismo skyriaus plenarinė sesija. Šio teismo teisėją skiria Seimas Respublikos Prezidento teikimu. Kandidatūras parenka ir Respublikos Prezidentui pasiūlo Aukščiausiojo Teismo pirmininkas. Turint omenyje Konstitucinio Teismo 1999 m. pateiktą išaiškinimą, kad Konstitucijoje neleidžiama įstatymu suteikti teisingumo ministrui teisės teikti Respublikos Prezidentui teisėjų kandidatūrų (nes be tokio teikimo Respublikos Prezidentas negali įgyvendinti savo konstitucinių įgaliojimų, numatytų Konstitucijos 84 str. 11 p.), svarbi nuostata, kad Aukščiausiojo Teismo pirmininko siūloma kandidatūra „Respublikos Prezidento nesaisto” (Teismų įstatymo 73 str. 2 d.). Aukščiausiojo Teismo teisėjus skiria (atleidžia) Seimas Respublikos Prezidento teikimu. Tai detalizuota Teismų įstatymo nuostatose, kuriose teigiama, kad Aukščiausiojo Teismo teisėju gali būti skiriamas teisėjas, turintis tam tikrą teisėjo darbo atitinkamame teisme stažą: bendrosios kompetencijos apygardos teisme ar apygardos administraciniame teisme – ne mažiau kaip aštuonerių metų, Apeliaciniame teisme arba Vyriausiajame administraciniame teisme – ne mažiau kaip penkerių metų. Taigi akivaizdu, kad, pavyzdžiui, apylinkės teismo teisėjas (turintis teisėjo darbo stažą tik šiame teisme) negali būti skiriamas Aukščiausiojo Teismo teisėju, nesvarbu, koks jo darbo stažas. Aukščiausiojo Teismo teisėju gali būti skiriamas ir teisės krypties socialinių mokslų daktaras ar habilituotas daktaras, turintis ne mažesnį kaip dešimties metų atitinkamą teisinio pedagoginio darbo stažą. Beje, Aukščiausio Teismo teisėjais gali būti skiriami ir advokatai bei Generalinės prokuratūros prokurorai, atitinkamai turintys penkiolikos metų advokato ar prokuroro darbo stažą (Teismų įstatymo 68 str.). Lietuvos apeliacinis teismas, kuriame taip pat yra du skyriai (Civilinių bylų ir Baudžiamųjų bylų skyriai), yra apeliacinė instancija byloms dėl apygardos teismų sprendimų (nuosprendžių, nutarčių, nutarimų ir įsakymų) apskųsti. Be to, šis teismas nagrinėja prašymus dėl užsienio valstybių ir tarptautinių teismų bei arbitražų sprendimų pripažinimo ir vykdymo Lietuvos Respublikoje. Siame teisme bylas nagrinėja trijų teisėjų kolegija, o teisėjus skiria Respublikos Prezidentas Seimo pritarimu. Įstatyme nustatyta, jog į Apeliacinio teismo teisėjo pareigas skiriami teisėjai, kurie įrašyti į „teisėjų karjeros siekiančių asmenų registrą”, o kandidatai turi turėti tam tikrą teisėjo darbo bendrosios kompetencijos apygardos teisme ar apygardos administraciniame teisme stažą – ne mažiau kaip ketverių metų (Teismų įstatymo 67 str., 72 str.). Apygardos teismai yra apeliacinė instancija byloms dėl apylinkės teismų sprendimų (nuosprendžių, nutarčių, nutarimų ir įsakymų) apskųsti. Be to, šie teismai yra pirmoji tam tikrų civilinių ir baudžiamųjų bylų, kurios pagal įstatymus priklauso jų kompetencijai, instancija. Apygardos teisme bylas nagrinėja trijų teisėjų kolegija, o įstatymų nustatytais atvejais – vienas teisėjas. Apygardos teismų teisėjus skiria (atleidžia) Respublikos Prezidentas, atsižvelgdamas į Teismų tarybos patarimą. Į apygardos teismo teisėjo pareigas gali būti skiriami teisėjai, kurie įrašyti į „teisėjų karjeros siekiančių asmenų registrą” ir turi ne mažesnį kaip penkerių metų apylinkės teismo teisėjo darbo stažą (Teismų įstatymo 65 str., 66 str.). Apylinkės teismai yra pirmoji civilinių ir baudžiamųjų bylų, kurios pagal įstatymus priklauso jų kompetencijai, instancija. Prie apylinkės teismo yra ir hipotekos skyrius. Šio skyriaus veiklos teritoriją, kuri gali nesutapti su apylinkės teismo veiklos teritorija, nustato teisingumo ministras (Teismų įstatymo 16 str.). Apylinkės teis
121
KONSTITUCINĖ TEISĖ
me bylas nagrinėja vienas teisėjas, atskirais įstatymų numatytais atvejais bylas gali nagrinėti trijų teisėjų kolegija. Šio teismo teisėju Respublikos Prezidentas, atsižvelgdamas į Teismų tarybos patarimą, gali skirti nepriekaištingos reputacijos pilietį, turintį aukštąjį universitetinį teisinį išsilavinimą, pateikusį sveikatos pažymėjimą, turintį ne mažesnį kaip penkerių metų teisinio darbo stažą ir išlaikiusį pretendentų į teisėjus egzaminą (Teismų įstatymo 51 str.). Asmuo pirmą kartą į teisėjo pareigas skiriamas penkeriems metams. Pasibaigus šiam terminui (ir nustatyta tvarka įvertinus, kad asmuo tinka dirbti teisėju), Respublikos Prezidentas gali tą patį asmenį skirti apylinkės teismo teisėju (be egzamino ir atrankos) iki to laiko, kol jam sukaks 65 metai. Į apylinkės teismo, kaip ir kitų minėtų teismų, teisėjo pareigas gali būti skiriami ir teisės krypties socialinių mokslų daktarai bei habilituoti daktarai, turintys atitinkamą teisinio pedagoginio darbo stažą (Teismų įstatymo 66-69 str.)
ADMINISTRACINIAI TEISMAI Įstatymų nuostatose pasakyta, kad administracinis teismas yra teismas, skirtas nagrinėti „byloms dėl ginčų, kylančių iš administracinių teisinių santykių” (Teismų įstatymo 12 str. 4 d.), pavyzdžiui, spręsti, ar tam tikru rašytiniu ar nerašytiniu aktu (realia veika) „nebuvo pažeistas įstatymas ar kitas teisės aktas, ar administravimo subjektas neviršijo kompetencijos, taip pat ar aktas (veika) neprieštarauja tikslams bei uždaviniams, dėl kurių institucija buvo įsteigta ir gavo atitinkamus įgaliojimus” (Administracinių bylų teisenos įstatymo 3 str.). Aiškiau pasidaro į pagalbą pasitelkus administracinių teismų nagrinėjamų bylų pavyzdžius. Tuose teismuose nagrinėjamos, pavyzdžiui, bylos dėl mokesčių sumokėjimo ar išieškojimo (ar finansinių sankcijų taikymo), dėl tarnautojų ir jų darbdavės – valstybės institucijos tarnybinių ginčų, dėl Vyriausiosios tarnybinės etikos komisijos sprendimų, dėl valstybės ar savivaldybės tarnautojų neteisėtais veiksmais padarytos turtinės ir moralinės žalos asmenims atlyginimo, dėl užsieniečių skundų dėl pabėgėlio statuso, dėl valstybinio administravimo institucijų vilkinimo (t. y. sąmoningo delsimo) atlikti jų kompetencijai priskirtus veiksmus (Administracinių bylų teisenos įstatymo 15 str.). Pažymėtina, kad administraciniai teismai netiria Respublikos Prezidento, Seimo, Seimo narių ar Vyriausybės veiklos, tuo labiau prokurorų ar ikiteisminio tyrimo pareigūnų procesinių veiksmų. Konstitucijoje nekonstatuojama, kad yra (veikia ar turi veikti) administraciniai teismai. Nuostatą, kad „gali būti įsteigti specializuoti teismai” (Konstitucijos 111 str. 2 d.) įstatymų leidėjas įgyvendino 1999 m. priimdamas Administracinių teismų įsteigimo įstatymą (Žin. 1999, Nr. 13-309). Pirminėje šio įstatymo redakcijoje buvo numatyta tokia administracinių teismų sistema: apygardos administraciniai teismai, Vilniaus aukštesnysis administracinis teismas ir Apeliacinio teismo Administracinių bylų skyrius. Tokia sistema pakeista 2000 m. priėmus Administracinių teismų įsteigimo įstatymo pataisas (Žin. 2000, Nr. 85-2567), t. y. panaikinus Vilniaus aukštesnįjį administracinį teismą ir Apeliacinio teismo Administracinių bylų skyrių. Dabar, be penkių apygardos administracinių teismų, veikia dar Lietuvos vyriausiasis administracinis teismas. Įstatyme yra detalizuota Lietuvos vyriausiojo administracinio teismo kompetencija, kuri pasireiškia trejopai: šis teismas, viena, yra apeliacinė instancija byloms dėl apygardos administracinių teismų sprendimų (nutarimų ir nutarčių) apskųsti, antra, apeliacinė instancija administracinių teisės pažeidimų byloms dėl bendrosios kompetenci
122
Pagrindinės valstybės valdžios institucijos
jos apylinkės teismų nutarimų apskųsti, trečia, pirmoji ir galutinė instancija administracinėms byloms, priklausančioms jo kompetencijai. Be to, Vyriausiasis administracinis teismas yra galutinė instancija administracinėms byloms pagal skundus dėl Vyriausiosios rinkimų komisijos sprendimų ar neveikimo. Šis teismas gali būti laikomas ir savotiška kasacine instancija, nes gali įstatymų nustatytais atvejais patenkinti asmenų prašymus atnaujinti procesą užbaigtose, t. y. įsiteisėjusiose teismo sprendimu, administracinėse bylose, tarp jų – ir administracinių teisės pažeidimų bylose (Administracinių bylų teisenos įstatymo 20 str. 2 d., Teismų įstatymo 31 str.). Vyriausiajame administraciniame teisme bylas nagrinėja trijų teisėjų kolegija, sudėtingas bylas teismo pirmininko iniciatyva – išplėstinė penkių teisėjų kolegija ar šio teismo plenarinė sesija, kurioje dalyvauja ne mažiau kaip 2/3 teismo teisėjų. Pažymėtina, kad Vyriausiasis administracinis teismas leidžia biuletenį „Administracinių teismų praktika”, kuriame skelbiamais to teismo plenarinės sesijos, trijų teisėjų kolegijos ar išplėstinės penkių teisėjų kolegijos priimtais sprendimais, nutarimais ar nutartimis formuoja vieningą administracinių teismų praktiką. Vyriausiojo administracinio teismo teisėju skiriamas asmuo turi būti įrašytas į „teisėjų karjeros siekiančių asmenų registrą” ir turėti bendrosios kompetencijos apygardos teisme ar apygardos administraciniame teisme ne mažiau kaip ketverių metų teisėjo darbo stažą (Teismų įstatymo 67 str., 72 str.). Apygardos administracinis teismas pagal įstatymus yra pirmoji instancija administracinėms byloms, kurios nepriklauso pirmiau aptartų administracinių teismų kompetencijai, tirti. Pavyzdžiui, apygardos teismai sprendžia bylas pagal skundus dėl įgaliotų pareigūnų (bet ne apylinkės teismų) nutarimų administracinių teisės pažeidimų bylose. Vilniaus apygardos administracinis teismas ypatingas tuo, kad jis, palyginti su kitais keturiais apygardos administraciniais teismais, turi išimtinę kompetenciją spręsti pirmąja instancija bylas, kurios priklauso administracinių teismų kompetencijai, jei pareiškėjas ar atsakovas yra ministerija, Vyriausybės įstaiga ar kitas centrinis administravimo subjektas (išskyrus bylas dėl šių subjektų priimtų norminių administracinių aktų teisėtumo – tai spręsti yra išimtinė Vyriausiojo administracinio teismo kompetencija). Pagal Konstituciją (112 str. 4 d.) visus specializuotų, taigi administracinių, teismų teisėjus skiria Respublikos Prezidentas (be Seimo pritarimo). Įstatyme įtvirtinta, jog į administracinių teismų teisėjo pareigas skiriami teisėjai, kurie įrašyti į „teisėjų karjeros siekiančių asmenų registrą” ir turi ne mažesnį kaip penkerių metų apylinkės teismo teisėjo darbo stažą (Teismų įstatymo 65 str., 66 str.). Taigi reikalavimai specializuotų teismų apygardos lygmens ir bendrosios kompetencijos teisėjams yra vienodi. Verta atkreipti dėmesį, kad kol pagal įstatymą aukščiausioji ir galutinė instancija administracinėms byloms tirti buvo Apeliacinio teismo Administracinių bylų skyrius, kurio teisėjus (kaip ir kitus šio teismo Civilinių ir Baudžiamųjų bylų skyrių teisėjus) skirdavo Respublikos Prezidentas Seimo pritarimu (Konstitucijos 84 str. 11 p., 112 str. 3 d.), konstitucinis Respublikos Prezidento įgaliojimas skirti ir atleisti administracinių teismų teisėjus atrodė diskutuotinas. Panaikinus tokį skyrių ir įsteigus Vyriausiąjį administracinį teismą, jo teisėjus skiria (atleidžia) Respublikos Prezidentas (be Seimo pritarimo) (Konstitucijos 112 str. 4 d.). Tačiau visa tai dabar tapo diskutuotina kitu požiūriu, nes bendroji konstitucinė teismų sistema („Lietuvos Respublikos teismai yra Lietuvos Aukščiausiasis Teismas, Lietuvos apeliacinis teismas, apygardų ir apylinkių teismai”; Konstitucijos 111 str. 1 d.) pagal Konstituciją turi būti derinama su: 1) Seimo, kaip kolegialios valstybinės bendruomenės at
123
KONSTITUCINĖ TEISĖ
stovybės, įgaliojimais skirti ar dalyvauti skiriant galutinės apeliacinės ir kasacinės instancijų teismų teisėjus (Apeliacinio teismo, Aukščiausiojo Teismo teisėjus); 2) parlamentine apkalta, kuri gali būti taikoma galutinės apeliacinės ir kasacinės instancijų teismų teisėjams (Aukščiausiojo Teismo ir Apeliacinio teismo teisėjams). Tai, kad Respublikos Prezidentas skiria Vyriausiojo administracinio teismo (galutinės instancijos administracinėms byloms tirti) teisėjus be Seimo pritarimo, viską šiek tiek sujaukia, nes nublanksta Apeliacinio teismo teisėjai (žiūrint hierarchijos).
PROKURATŪRA TEISMŲ ATŽVILGIU Konstitucinio Teismo pateiktame išaiškinime sakoma, kad „prokurorai nėra teisėjai, jie nevykdo teisingumo. Konstitucijoje jiems patikėta specifinė funkcija, kurios negalima tapatinti su teisminės valdžios įgyvendinimu” (Konstitucinio Teismo 1999 m. gruodžio 21 d. nutarimas). Pagal Konstitucijos 84 straipsnio 11 punkto ir 118 straipsnio 2003 m. priimtas pataisas: 1) prokurorai vykdo tokias konstitucines funkcijas: organizuoja ikiteisminį tyrimą ir vadovauja jam bei palaiko valstybinį kaltinimą baudžiamosiose bylose (118 str. 1 d.). Tai detalizuojančiame Prokuratūros įstatyme teigiama, kad, pavyzdžiui, prokurorai turi teisę nutarimu iškelti ikiteisminio tyrimo pareigūnui drausmės bylą. Be to, Konstitucijos 118 straipsnyje įtvirtinta nuostata, kad įstatymo nustatytais atvejais prokuroras gina asmens, visuomenės ir valstybės teises bei teisėtus interesus (118 str. 2 d.). Žodžiai „įstatymo nustatytais atvejais” šiek tiek detalizuoti Prokuratūros įstatyme (19 straipsnyje „Viešojo intereso gynimas”). Jame numatyta, kad prokurorai, nustatę asmens, visuomenės, valstybės teisių ir teisėtų interesų pažeidimą, viešąjį interesą gina arba savo iniciatyva, arba pagal asmens, valstybės (savivaldybės) institucijos ar įstaigos pareiškimą (pranešimą, pasiūlymą) ar skundą, neatsižvelgdami į tai, ar viešąjį interesą konkrečiu atveju privalėjo ginti kitų institucijų pareigūnai (svarbiausia yra tai, kad šie pareigūnai „nesiėmė priemonių pažeidimams pašalinti”). Pagal Prokuratūros įstatymą (19 str. 2 d.) ginti viešąjį interesą galima, pavyzdžiui, įgyvendinant tokius įgaliojimus: prokurorui kreipiantis į teismą su ieškiniu, prokurorui dalyvaujant teismui nagrinėjant civilines bylas ir kartu prokurorų pareikštus civilinius ieškinius baudžiamosiose bylose, prokurorui apskundžiant teismo nutartis (nutarimus, sprendimus), jam priimant nutarimus dėl fizinių asmenų iškeldinimo iš gyvenamųjų patalpų, prokuroro nutarimu reikalaujant atlikti tam tikro valstybės pareigūno (tarnautojo) veiklos tarnybinį patikrinimą; 2) prokuroras konstituciniu lygmeniu tarsi prilygintas teisėjui pagal nepriklausomumą. Konstitucijoje parašyta: „prokuroras, vykdydamas savo funkcijas, yra nepriklausomas ir klauso tik įstatymo” (plg. Konstitucijos 109 str. 2 ir 3 d.), todėl teisinga Prokuratūros įstatymo (4 str.) nuostata, kad prokuratūrai vadovaujantis generalinis prokuroras „atsiskaito Respublikos Prezidentui ir Seimui”, o Vyriausybei teikia informaciją. Pažymėtina, kad naują prokurorų nepriklausomumo konstitucinę garantiją nevisiškai atitinka tai, kad, viena vertus, prokurorams yra privalomos generalinio prokuroro patvirtintos rekomendacijos dėl ikiteisminio tyrimo ar valstybinio kaltinimo praktikos (Prokuratūros įstatymo 8 str. 1 p., 16 str. 2 d.), o antra vertus, Generalinė prokuratūra vadovauja teritorinėms prokuratūroms. Be to, diskutuotina, ar prokurorų ir kartu Lietuvos prokuratūros nepriklausomumą atitinka tai, kad „prokuratūros veiklos prioritetus nustato ir prokuratūros parlamen
124
Pagrindinės valstybės valdžios institucijos
tinę kontrolę atlieka Seimas” (Prokuratūros įstatymo 4 str. 2 d.). Pažymėtina ir tai, kad pagal Prokuratūros įstatymą (20 str. 2 d.) nelaikoma, jog neleistinai kišamasi į teisėjo ar teismo veiklą, jei prokuroras įgyvendina teisę „pranešti asmenims, kurie pagal Teismų įstatymą atlieka teismų administracinės veiklos priežiūrą, apie atvejus, kai teismas nesiima reikiamų priemonių bylai tinkamai išnagrinėti”. Verta dėmesio ir tai, kad pradėti ikiteisminį tyrimą dėl prokuroro padarytos nusikalstamos veikos gali tik generalinis prokuroras (Prokuratūros įstatymo 12 str. 3 d.); 3) konstituciniu lygmeniu nustatyta generalinio prokuroro skyrimo ir atleidimo tvarka: generalinį prokurorą skiria ir atleidžia Respublikos Prezidentas Seimo pritarimu (Konstitucijos 84 str. 11 p., 118 str. 5 d.). Tokia nuostata turi stabilizuoti generalinio prokuroro skyrimo tvarką, kuri kelis kartus keitėsi 1994-2000 m. Štai 1994 m. spalio mėnesį Prokuratūros įstatyme nustatyta, kad generalinį prokurorą skiria (atleidžia) Respublikos Prezidentas, tų pačių metų lapkričio mėnesį – kad tai daro Seimas Respublikos Prezidento teikimu, 1997 m. – kad generalinį prokurorą septyneriems metams skiria (ir atleidžia) Seimas pagal Seimo Teisės ir teisėtvarkos komiteto, kuriam kandidatūras siūlė Lietuvos Aukščiausiojo Teismo pirmininkas ir teisingumo ministras, teikimą, 2000 m. – kad generalinį prokurorą septyneriems metams skiria (ir atleidžia) Respublikos Prezidentas Seimo pritarimu. Tokiu nestabiliu reglamentavimu įstatymų leidėjas pažeidė teisėtų lūkesčių principą. Tai išaiškino Konstitucinis Teismas, pažymėdamas, kad „nė vienas generalinis prokuroras nėra šių pareigų ėjęs visą įstatymo nustatytą įgaliojimų laiką. Teisės aktuose, kuriais generaliniai prokurorai buvo atleidžiami iš pareigų, šių pareigūnų atleidimas nebuvo siejamas su jų darbo vertinimu”. Be to, „įstatymų leidėjas, įstatymu nustatęs generalinio prokuroro įgaliojimų trukmę, neturi teisės nustatyti bet kokių generalinio prokuroro atleidimo iš pareigų nepasibaigus įgaliojimų terminui pagrindų” (Konstitucinio Teismo 2003 m. sausio 24 d. nutarimas). Prokuratūros įstatyme (22 str.) nustatyti reikalavimai kandidatui į generalinio prokuroro (ir jo pavaduotojo) pareigybę. Jis, be kita ko, turi būti ne jaunesnis kaip 35 metų ir turėti ne mažesnį kaip dešimties metų tam tikro teisinio darbo (tarnybos prokuroru ar teisėjo darbo, ar teisinio pedagoginio darbo su tam tikru moksliniu laipsniu) stažą; 4) konstituciniu lygmeniu detalizuota prokuratūros sistema (sandara): „Lietuvos Respublikos prokuratūra yra Generalinė prokuratūra ir teritorinės prokuratūros” (Konstitucijos 118 str. 4 d.). Tai detalizuota Prokuratūros įstatyme (6-9 str.), kuriame teigiama, kad teritorinės prokuratūros yra dvejopos: apygardos ir apylinkės prokuratūros. Pažymėtina, kad Generalinė prokuratūra formuoja vienodą prokurorų praktiką (pvz., atliekant nusikalstamų veikų ikiteisminį tyrimą ar palaikant valstybinį kaltinimą baudžiamosiose bylose), gina viešąjį interesą, vadovauja teritorinėms prokuratūroms (Prokuratūros įstatymo 8 str. 1 p.). Be to, naujojoje Prokuratūros įstatymo redakcijoje neliko ankstesnių nuostatų, kad prokuratūros veikia „prie teismų” (t. y. Generalinė prokuratūra – prie Aukščiausiojo Teismo, apygardos prokuratūros – prie apygardos teismų, apylinkės prokuratūros – prie apylinkės teismų).
ADVOKATŪRA Konstitucijoje tiesiogiai neužsimenama apie advokatūrą – advokatų visumą. Todėl gali atrodyti, kad nėra pagrindo, kalbant apie teismus, vertinti advokatų organizaciją, kuri nėra valstybės institucija, o advokatas nėra valstybės tarnautojas ar teisėsaugos pareigūnas ir tėra specifinės profesinės veiklos subjektas, negaunantis iš
125
KONSTITUCINĖ TEISĖ
valstybės darbo užmokesčio. Tačiau žiūrint į Konstitucijos visumą, svarbi nuostata (31 str. 6 d.), kurioje teigiama, kad „asmeniui, kuris įtariamas padaręs nusikaltimą, ir kaltinamajam nuo jų sulaikymo arba pirmosios apklausos momento garantuojama teisė į gynybą, taip pat ir (išryškinta-aut.). Ši nuostata įterpta į straipsnį, kuriame sujungti esminiai teisingumo vykdymo baudžiamosiose bylose principai, pavyzdžiui, asmens nekaltumo prezumpcijos principas („asmuo laikomas nekaltu, kol jo kaltumas neįrodytas įstatymo nustatyta tvarka ir pripažintas įsiteisėjusiu teismo nuosprendžiu” (31 str. 1 d.). Taigi advokatūrą galima gretinti su teismais, tuo labiau, kad Advokatūros įstatyme skelbiama, jog advokatai yra nepriklausoma Lietuvos teisinės sistemos dalis. Pagal naują (2004 m.) Advokatūros įstatymą (4 str.) advokato veikla yra teisinių paslaugų teikimas, bet ne komercinės ūkinės veiklos vykdymas. Advokatu gali tapti nepriekaištingos reputacijos teisininkas, išlaikęs advokato kvalifikacinius egzaminus (pagal Teisingumo ministerijos nustatytą programą ir tvarką, suderintą su Lietuvos advokatūra) po to, kai įgijo tam tikrą teisinį išsilavinimą (teisės magistro arba teisininko profesinį kvalifikacinį laipsnį, t. y. vienpakopį teisinį universitetinį išsilavinimą), atliko dvejų metų advokato padėjėjo praktiką arba (jei tokios praktikos neatliko) sukaupė tam tikro teisinio darbo stažą (ne mažesnį kaip dvejų metų teisinio darbo pagal Lietuvos Respublikos Vyriausybės patvirtintą pareigybių sąrašą stažą). Be to, advokatu lengvatine tvarka (t. y. be kvalifikacinių egzaminų) gali būti pripažintas asmuo, turintis penkerių metų teisinio darbo stažą. Visa tai kiekvienu konkrečiu atveju vertina Lietuvos advokatūros institucija – Advokatų taryba, kuri priima pagal suinteresuoto asmens pareiškimą sprendimą pripažinti (ar nepripažinti) pareiškėją advokatu. Pažymėtina, kad verstis advokato praktika gali tik civilinę atsakomybę apsidraudęs (minimali draudimo suma – 100 000 litų „vienam draudiminiam įvykiui”) ir kontorą (t. y. „advokato darbo vietą” individualiai veiklai) įsteigęs (arba sutartį dėl nuolatinės darbo vietos su jau veikiančia kontora sudaręs) advokatas (Advokatūros įstatymo 20 str., 21 str.). Advokatui draudžiama tiesiogiai ar netiesiogiai reklamuoti savo profesinę veiklą. Leidžiama susitarti su klientais dėl užmokesčio taip, kad šio užmokesčio dydis priklausytų nuo bylos baigties tik civilinėse bylose ir tose baudžiamosiose bylose, kuriose pateikiamas ieškinys. Tada, kai advokatai atlieka savo profesines pareigas, jie negali būti tapatinami su savo klientais (Advokatūros įstatymo 46 str.).
126
t e i s ė t u r ė t i a d v o k a t ą
PAGRINDINES SĄVOKOS
Konstitucinė teisė – pamatinių vertybių ir jas atitinkančių taisyklių (normų, principų) sistema, t. y. tautos suvereniteto ir prigimtinių žmogaus teisių ir laisvių, jų apsaugos, gynimo ir įgyvendinimo institucines bei organizacines priemones įtvirtinančios Konstitucijos ir ją galutinai ir neskundžiamai aiškinančio Konstitucinio Teismo sprendimuose išplėtotų principų bei koncepcijų visuma. Konstituciniai santykiai – konstituciškai reikšmingi (neteisiniai) arba tiesiogiai teisės normų reglamentuojami (teisiniai) esminiai žmogaus ir valstybės institucijų santykiai, susiję su prigimtinėmis vertybėmis ar konstituciniais principais (taip pat pagrindinių konstitucinio lygmens valstybės institucijų santykiai, kolegialios valstybės institucijos nario ir kitų šios institucijos narių santykiai bei kolegialios valstybės institucijos nario ir šios institucijos santykiai). Konstitucija – ypatinga teisės akto forma, t. y. pagrindinis ir svarbiausias valstybės teisės aktas, valstybinės bendruomenės ir jos atstovų kompromisinė sutartis dėl vertybių ir jas atitinkančių taisyklių (normų, principų), darančių įtaką žmogaus ir visuomenės teisiniam saugumui. Konstitucija ne tik įtvirtina esmines bendruomenės institucines ar organizacines formas, bet ir yra valstybinės bei socialinės valdžios ribojimo aktas, skirtas apsaugoti ir ginti individą. Pilietybė – tai žmogaus narystės valstybėje teisinė forma, t. y. nuolatinis asmens politinis teisinis ryšys su konkrečia valstybe, grindžiamas abipusėmis teisėmis bei pareigomis ir iš jų išplaukiančiu savitarpio pasitikėjimu, ištikimybe bei gynyba. Politinė partija – įregistruotas, ne mažiau kaip 1000 (aktyviąją rinkimų teisę turinčių) piliečių jungiantis susivienijimas, siekiantis padėti formuoti ir išreikšti piliečių interesus ir politinę valią bei turintis suvažiavime patvirtintą programą, statutą ir išrinktus vadovaujamuosius organus. Laisvas mandatas – tai kiekvieno Seimo nario, kaip visos teisinės tautos atstovo, vadovavimasis (laikantis Konstitucijos ir įstatymų) savo sąžine ir nesusivaržymas jokiais mandatais, t. y. pažadais ar įsipareigojimais jį rėmusiems ar kitokiems asmenims. Referendumas – visuotinis piliečių balsavimas dėl svarbaus ir reikšmingo valstybei ir visuomenei klausimo. Referendumą skelbia Seimas savo iniciatyva arba kai to reikalauja ne mažiau kaip 300 tūkstančių rinkimų teisę turinčių piliečių. Respublika – kelių valstybės valdžią vykdančių institucijų bendro valstybės valdymo forma, grindžiama reguliariu ir demokratišku aukščiausio valstybės pareigūno perrinkimu. Suverenitetas – valstybės vardu veikiančių institucijų galimybė savo teritorijoje realizuoti teisinį ir politinį savarankiškumą, t. y. vykdyti pagrindines valstybės valdžios funkcijas, nepaneigiant universalių žmogaus teisių ir laisvių. Teisinė valstybė (plačiuoju požiūriu) – tai pagal Konstituciją, konstitucingus įstatymus ir valdymo aktus bei nepriklausomų teismų sprendimus gyvuojanti valsty
127
KONSTITUCINĖ TEISĖ
bė, kurioje asmuo teisinėmis priemonėmis (skundu, pareiškimu) gali realiai gintis nuo patiriamos savivalės, teisinio netikrumo (nesaugumo) ar teisėtų lūkesčių pažeidimų. Cenzūra – spaudos, kino filmų, radijo ir televizijos laidų, teatro spektaklių ir kitų viešų renginių turinio kontrolė, kad nebūtų platinamos tam tikros žinios ir idėjos. Seimas – kolegiali valstybinės bendruomenės („teisinės tautos”) atstovybė, parlamentas, kurio kiekvienas narys atstovauja Tautai (ne savivaldybei, apskričiai, tam tikrai partijai, profesijai, religijai ar lyčiai), o visi nariai (kartu paėmus) yra Tautos atstovybė. Seimas yra vienintelė institucija, turinti įgaliojimą leisti įstatymus. Respublikos Prezidentas – ne tiek tautos, kiek valstybės atstovas, ė valstybinės valdžios institucija, turinti užtikrinti valstybės valdžios institucijų bendradarbiavimą ir savo autoritetu personifikuoti (sužmoginti, suasmeninti) valstybės institucijų ir pareigūnų sistemą. Vyriausybė – kolegiali valstybės vykdomosios valdžios institucija, kurią sudaro Ministras Pirmininkas ir ministrai. Parlamentinė kontrolė – Seimo narių įgaliojimai kontroliuoti Vyriausybę, pirmiausia – tvirtinant Vyriausybės parengtą valstybės biudžeto projektą, teikiant paklausimus ar interpeliacijas, kraštutiniu atveju – reiškiant Seimo nepasitikėjimą ministru, Ministru Pirmininku ir kartu visa Vyriausybe.
KLAUSIMAI
1. Kokie yra konstitucinės teisės šaltiniai? 2. Kodėl žmogaus teisių ir laisvių bei kitų konstitucinių vertybių esminiai klausimai negali būti reglamentuojami poįstatyminiais aktais? 3. Kokios yra stambiausios Lietuvos Respublikos Konstitucijos sudedamosios dalys? 4. Kokie yra pagrindiniai Konstitucijos bruožai? 5. Kaip suprantate teiginį, kad Konstitucija yra tiesiogiai taikomas aktas? 6. Kokios pagrindinės Konstitucijos atliekamos funkcijos? 7. Kas yra Konstitucijos blanketinės nuostatos? Pateikite pavyzdžių. 8. Kokie yra Lietuvos Respublikos pilietybės įgijimo ir netekimo būdai? 9. Paaiškinkite vietos savivaldybių ir valstybės ryšius pagal Lietuvos Respublikos Konstituciją. Ar apskrities viršininkas yra vietos savivaldos teisę įgyvendinantis subjektas? 10. Kuo skiriasi vienmandatė rinkimų apygarda nuo daugiamandatės? 11. Paaiškinkite valdžių padalijimo principo esmę pagal Lietuvos Respublikos Konstituciją.
128
vienasmen
Pagrindinės sąvokos, klausimai, užduotys
12. Kokiais atvejais gali būti pateisinama cenzūra demokratinėje visuomenėje? 13. Kokios yra pagrindinės Lietuvos Respublikos piliečio konstitucinės pareigos? 14. Kuo skiriasi Seimo komitetai nuo frakcijų? 15. Kokios yra pagrindinės įstatymų leidybos procedūros stadijos? 16. Kuo skiriasi ir kuo panašios Respublikos Prezidento ir Vyriausybės funkcijos (kompetencija) ? 17. Kas yra interpeliacija? 18. Kas gali kreiptis į Konstitucinį Teismą dėl teisės akto (kokio) konstitucingumo?
UŽDUOTYS
1. Konstitucijoje numatyta Seimo galimybė 3/5 visų narių balsų dauguma atimti Seimo nario mandatą ar pašalinti iš pareigų Respublikos Prezidentą, Konstitucinio Teismo, Aukščiausiojo Teismo ir Apeliacinio teismo teisėjus, „šiurkščiai pažeidusius Konstituciją arba sulaužiusius priesaiką, taip pat paaiškėjus, jog padarytas nusikaltimas” (Konstitucijos 74 str.). Kokią funkciją atlieka šios Konstitucijos nuostatos, t. y. kas iš jų išplaukia (nors nėra tiesiogiai, pažodžiui nurodyta)? 2. Nubraižykite schemą ir joje pavaizduokite konstitucinių santykių subjektų tarpusavio ryšius. 3. Išanalizuokite Lietuvos Respublikos pilietybės įstatymą ir nurodykite, kokiais atvejais asmuo gali turėti dvigubą pilietybę – Lietuvos Respublikos ir bet kurios kitos užsienio valstybės. 4. Išrinkite Konstitucijos nuostatą, kurią, Jūsų nuomone, reikėtų pakeisti. Suformuluokite savo siūlomą Konstitucijos pataisą ir nurodykite konkrečius žingsnius, kurių imtumėtės siekdami pakeisti Konstituciją. 5. Padiskutuokite, ar eutanazija nepažeidžia žmogaus teisės į gyvybę. 6. Konstitucinis Teismas 1998 m. priėmė nutarimą, kuriuo pripažino, kad Baudžiamajame kodekse numatyta mirties bausmė prieštarauja Konstitucijai. Į bendrosios kompetencijos teismą kreipėsi asmenų, kuriems įvykdyta mirties bausmė tarp 1992 ir 1998 m., artimieji giminaičiai, prašydami priteisti žalą dėl neteisėto mirties bausmės įvykdymo. Ką turėtų atsakyti bendrosios kompetencijos teismas?
129
1. ADMINISTRACINĖS TEISĖS SAMPRATA
1.1. ADMINISTRACINĖS TEISĖS REGULIAVIMO DALYKAS
Savo pobūdžiu teisės reguliuojami visuomeniniai santykiai yra labai įvairūs, jie susiklosto įvairiose valstybinio ir visuomeninio gyvenimo srityse bei sferose. Todėl, atsižvelgiant į šių visuomeninių santykių įvairovę bei jų reguliavimo ypatumus (metodą), teisė, kaip tam tikros rūšies socialinių normų sistema, skaidoma į atskiras šakas, kurių kiekviena turi savo specifinį reguliavimo dalyką – tam tikrų visuomeninių santykių grupę. Administracinės teisės, kaip savarankiškos teisės šakos, reguliavimo dalykas yra ypatinga, specifinė visuomeninių santykių grupė. Norint apibrėžti administracinę teisę ir išryškinti jos reguliavimo dalyką, reikia iš visos teisės sistemos išskirti tam tikrą grupę visuomeninių santykių, kurie turi šiai teisės šakai būdingų požymių. Administracinės teisės reguliavimo dalyką iš esmės nusako jau šios teisės šakos pavadinimas. Žodis „administracinė” kilęs iš lotynų kalbos žodžio administratio, kuris reiškia „valdymas, vadovavimas”. Taigi galima sakyti, kad administracinė teisė – tai valdymo teisė, arba teisės šaka apie valdymą, o jos reguliavimo dalykas yra visuma visuomeninių santykių, kurie susiklosto valdymo procese. Vis dėlto toks administracinės teisės reguliavimo dalyko apibrėžimas yra per platus, nepakankamai tikslus, nes sąvoka „valdymas” vartojama įvairiais aspektais, pavyzdžiui, ja nusakomi ne tik valdymo procesai, vykstantys valstybės viduje, bet ir už jos ribų. Valdymo procesus tiria įvairios visuomenės ir gamtos mokslų šakos filosofija, ekonomika, teisė, mechanika, bionika ir kt. Sąvoka „valdymas” jose traktuojama gana nevienodai. Siekiant apibūdinti administracinės teisės reguliavimo dalyką, pirmiausia reikia atsižvelgti į visuomeninių santykių teisinio reguliavimo sferą. Ši sfera – tai teisės normų reguliuojami valdymo pobūdžio santykiai, kurie yra susiję su specifine ir savarankiška valstybės veiklos rūšimi – viešuoju valdymu, arba valstybės vykdomosios valdžios įgyvendinimo veikla, kuri dažnai vadinama vykdomąja veikla. Lietuvos Respublikos viešojo administravimo įstatymo 3 straipsnyje sakoma: „Viešasis administravimas – įstatymais ir kitais teisės aktais reguliuojama valstybės ir vietos savivaldos institucijų, kitų įstatymais įgaliotų subjektų vykdomoji veikla, skirta įstatymams, kitiems teisės aktams, vietos savivaldos institucijų sprendimams įgyvendinti, numatytoms viešosioms paslaugoms administruoti” (Valstybės žinios, 1999, Nr. 601945). Taigi administracinės teisės reguliavimo dalykas yra visuma visuomeninių santykių, kurie susiklosto viešojo valdymo srityse, arba valstybės vykdomosios valdžios įgyvendinimo (vykdomosios veiklos) procese. Pagrindinė viešojo valdymo – valstybės vykdomosios valdžios subjektų (vyriausybės, ministerijų, vyriausybės įstaigų, savi
133
ADMINISTRACINE TEISE
valdybių institucijų ir kt.) paskirtis -įstatymų, kitų teisės aktų, vietos savivaldos institucijų sprendimų įgyvendinimas bei viešųjų paslaugų administravimas (plačiau apie viešojo valdymo sampratą, jo turinį, funkcijas, santykį su valstybės vykdomosios valdžios sąvoka Žr.: poskyryje „Viešasis valdymas”). Reikia pasakyti, kad administracinės teisės reguliavimo dalykas apima ne tik viešąjį valdymą, arba vykdomąją veiklą. Administracinės teisės normos reguliuoja ir tokius visuomeninius santykius, kurie susiklosto už viešojo valdymo (vykdomosios veiklos) ribų. Tai reiškia, kad administracinės teisės reguliavimo dalykui priskiriami ir kai kurie ne valdymo pobūdžio teisiniai santykiai. Tokiai ne valdymo pobūdžio teisinių santykių grupei, kuriuos reguliuoja administracinės teisės normos, priklauso specifiniai procesiniai santykiai, atsirandantys apylinkės teismams (teisėjams) ir kitiems subjektams nagrinėjant administracinių teisės pažeidimų bylas ir taikant pažeidėjams administracines nuobaudas bei kitas administracinio poveikio priemones, taip pat administraciniams teismams nagrinėjant skundus dėl priimtų sprendimų administracinių teisės pažeidimų bylose ir sprendžiant bylas dėl ginčų, kylančių iš administracinių teisinių santykių. Tokio pobūdžio teisinius santykius reguliuoja Lietuvos Respublikos administracinių teisės pažeidimų kodekso (ATPK) procesinės normos, Lietuvos Respublikos administracinių bylų teisenos įstatymas (toliau Administracinių bylų teisenos įstatymas) ir kiti įstatymai. Atsižvelgiant į tai, kad administracinė teisė reguliuoja įvairius teisinius santykius, galima išskirti tokias pagrindines administracinių teisinių santykių grupes: 1. Visuomeniniai santykiai, kurie susiklosto viešojo valdymo srityje, t. y.: 1) valstybės, savivaldybių institucijoms ir kitiems įstatymų įgaliotiems subjektams įgyvendinant valstybės vykdomosios valdžios uždavinius ir funkcijas socialinės kultūrinės, ūkinės, administracinės politinės veiklos srityse; 2) vidaus valdymo teisiniai santykiai, kurie susiklosto visų valstybės institucijų – Seimo, Respublikos Prezidento, Vyriausybės, ministerijų, teismų, prokuratūros, taip pat savivaldybių institucijų vidaus valdymo (administravimo) veikloje (struktūros tvarkymas, personalo valdymas, materialinių finansinių išteklių valdymas ir naudojimas, raštvedybos tvarkymas), kad nurodytos institucijos galėtų tinkamai vykdyti joms iškeltus pagrindinius uždavinius ir funkcijas (leisti įstatymus, vykdyti teisingumą, vadovauti socialinei, kultūrinei, ūkinei, administracinei, politinei veiklai, taip pat dalyvauti įmonių, įstaigų, teikiančių gyventojams socialines, kultūros, švietimo paslaugas, administracijos veikloje). 2. Teisiniai procesiniai santykiai, reguliuojami administracinės teisės normų, kurie susiklosto apylinkės teismams (teisėjams) ir kitiems subjektams nagrinėjant administracinių teisės pažeidimų bylas ir skiriant pažeidėjams administracinio poveikio priemones, taip pat administraciniams teismams nagrinėjant skundus dėl priimtų sprendimų administracinių teisės pažeidimų bylose ir sprendžiant bylas dėl ginčų, kylančių iš administracinių teisinių santykių. Reikia pabrėžti, kad administracinė teisė, reguliuodama visuomeninius santykius, visų pirma yra tiesiogiai ir neatskiriamai susijusi su viešuoju valdymu. Visuomeniniai santykiai, susiklostantys bei reguliuojami viešojo valdymo sferoje, sudaro administracinės teisės reguliavimo dalyko branduolį. Administracinės teisės normos įtvirtina pagrindinius viešojo valdymo principus, jo subjektų sudarymo tvarką, nustato šių subjektų teisinį statusą, jų uždavinius ir funkcijas, teises, pareigas, atsakomybę, veiklos formas ir metodus.
134
Administracinės teisės samprata
Viešojo valdymo subjektų vykdomoji veikla yra labai plati, ji apima įvairias socialinio kultūrinio, ūkinio, administracinio valstybinio darbo sritis. Šioje veikloje sprendžiami svarbiausi valstybės, visos visuomenės uždaviniai – kuriama bei plėtojama materialinė tautos ūkio bazė, tobulinami kiti visuomeniniai teisiniai santykiai, laiduojamas šalies saugumas, stiprinama teisėtvarka bei asmens teisių apsauga. Šių uždavinių negalima būtų išspręsti be teisinio, ypač administracinio teisinio, reguliavimo. Administracinės teisės normos reguliuoja valstybės tarnybos organizavimo ir jos veiklos santykius, nustato valstybės tarnautojų statuso įgijimo sąlygas ir tvarką, jų teises, pareigas, atsakomybę. Administracinės teisės normose neapsiribojama vien tik viešojo valdymo subjektų teisinio statuso ir veiklos reglamentavimu. Šiose teisės normose taip pat įtvirtinamos plačios kitų teisės subjektų – visuomeninių organizacijų, kitų visuomeninių susivienijimų, piliečių bei kitų asmenų teisės viešojo valdymo srityje ir nustatomos jų pareigos. Be to, administracinės teisės normose sukonkretinamos svarbiausios konstitucinės piliečių teisės, laisvės ir pareigos, sudaromos reikiamos organizacinės ir teisinės prielaidos joms įgyvendinti. Labai svarbu, kad administracinės teisės normos padeda užtikrinti teisėtumą, saugoti ir ginti visuomenės interesus, piliečių bei kitų asmenų teises viešojo valdymo srityje. Šiose normose nustatytos būtinos organizacinės ir teisinės teisėtvarkos užtikrinimo sąlygos, administracinių poveikio priemonių taikymo pagrindai ir procesinė tvarka.
1.2. ADMINISTRACINĖS TEISĖS REGULIAVIMO METODAS
Kitas kriterijus administracinės teisės reguliuojamų visuomeninių santykių specifikai apibūdinti ir administracinei teisei nuo kitų teisės šakų atriboti yra jų visuomeninių santykių reguliavimo metodas. Šis metodas parodo, koks yra teisės normų reguliuojamų visuomeninių santykių dalyvių tarpusavio ryšių pobūdis, jų padėtis vienas kito atžvilgiu. Paprastai skiriami du pagrindiniai teisinio reguliavimo metodai: civilinis teisinis ir administracinis teisinis. Administracinio teisinio metodo esmę sudaro tai, kad reguliuojamo santykio šalys nėra viena kitai lygios, jų tarpusavio teisinė padėtis paprastai išreiškiama valdžios ir pavaldumo kategorijomis. Tuo tarpu civilinio teisinio metodo esmę sudaro tai, kad civilinės teisės normų reguliuojamo visuomeninio santykio šalys paprastai yra lygiavertės viena kitos atžvilgiu. Kitaip sakant, administracinio teisinio santykio esmę sudaro ne atitinkamo santykio šalių organizacinis (hierarchinis) ir administracinis pavaldumas viena kitai, o tai, kad viena santykio šalis turi valdingus įgaliojimus spręsti tam tikrus klausimus bei priimti juridiškai privalomus sprendimus netgi tiesiogiai nepavaldžių subjektų atžvilgiu. Tai lemia pati viešojo valdymo prigimtis.
1.3. ADMINISTRACINĖS TEISĖS ŠALTINIAI
Administracinės teisės šaltiniai yra tie teisės aktai, kuriuose esama administracinės teisės normų. Administracinė teisė reguliuoja labai daug ir įvairių visuomeninių santykių. Pats administracinės teisės reguliavimo dalykas, jos reguliuojamų visuo
135
ADMINISTRACINE TEISE
meninių santykių įvairovė lemia ir jos normų gausumą, jų hierarchiją bei formas. Taigi administracinės teisės šaltiniai yra: 1) Lietuvos Respublikos Konstitucija. Administracinės teisės šaltinis yra ne visos konstitucinės normos, o tik tos, kuriose įtvirtinta viešojo valdymo institucijų sistema, jų sudarymo bei veiklos principai, kompetencijos pagrindai, piliečių teisės ir pareigos viešojo valdymo srityje; 2) ratifikuotos tarptautinės sutartys; 3) Lietuvos Respublikos įstatymai ir Seimo nutarimai, kurių normos reguliuoja visuomeninius santykius viešojo valdymo srityje. Administracinės teisės šaltiniams taip pat priskiriami kai kurie įstatymais patvirtinti kodeksai (Administracinių teisės pažeidimų kodeksas, Muitinės kodeksas, Kelių transporto kodeksas ir kt.), statutai (pvz., Lietuvos Respublikos vidaus tarnybos statutas); 4) Vyriausybės nutarimai, jais tvirtinami nuostatai, taisyklės; 5) ministrų ir kitų viešojo valdymo institucijų vadovų įsakymai, įsakymais patvirtintos taisyklės, instrukcijos; 6) Savivaldybių tarybų sprendimai, kitų savivaldybių institucijų vadovų įsakymai. Be to, kai kurie teisės žinovai laikosi nuomonės, kad administracinės teisės šaltinis yra ir Konstitucinio Teismo nutarimai.
1.4. ADMINISTRACINĖS TEISĖS SĄVOKOS APIBRĖŽIMAS
Galima sakyti, kad administracinė teisė yra savarankiška teisės šaka (teisės sistemos rūšis), kurios normos reguliuoja visuomeninius santykius viešojo valdymo sferoje, t. y.: 1) valstybės vykdomosios valdžios įgyvendinimo procese; 2) visų valstybės ir savivaldybių institucijų vidaus valdymo veikloje; 3) savivaldybių institucijų bei įstaigų veikloje; be to, 4) jos normos reguliuoja visuomeninius santykius apylinkės teismams bei kitiems subjektams nagrinėjant administracinių teisės pažeidimų bylas ir administraciniams teismams nagrinėjant bylas bei skundus dėl ginčų, kylančių iš administracinių teisinių santykių. Administracinė teisė yra susijusi su kitomis teisės šakomis. Ypač jai artima konstitucinė teisė, kuri yra nacionalinės teisės sistemos branduolys. Svarbiausios jos normos kartu yra ir administracinės teisės pagrindas, jos juridinės galios šaltinis. Konstitucinės teisės normose įtvirtinti visų teisės subjektų teisinės padėties ir veiklos pagrindai, nustatyta valstybės vykdomosios valdžios institucijų (tarp jų ir savivaldybių) sistema, t. y. centrinės ir teritorinės valdymo institucijos, jų sudarymo ir funkcionavimo principai, šių institucijų santykiai su kitomis valstybės aparato institucijomis. Administracinė teisė reguliuoja santykius, susijusius su viešojo valdymo subjektų (institucijų) sudarymu, jų funkcionavimu, jų veiklos koordinavimu, kontrole bei priežiūra. Administracinės teisės normos reguliuoja valstybės tarnybos organizavimo ir jos veiklos santykius, nustato valstybės tarnautojų statuso įgijimo sąlygas, jų teises, pareigas ir atsakomybę. Jose taip pat nustatyti administracinės atsakomybės pagrindai, jos poveikio priemonės ir procesinė jų taikymo tvarka.
136
2. ADMINISTRACINĖS TEISĖS NORMOS IR SANTYKIAI
2.1. ADMINISTRACINĖS TEISĖS NORMOS SAMPRATA IR STRUKTŪRA
Administracinės teisės normos yra savarankiška teisės normų rūšis. Joms būdingi bendrieji teisės normų požymiai – tai valstybės nustatytos ir saugomos bendro pobūdžio privalomos elgesio taisyklės, kurios nustato reguliuojamų visuomeninių santykių subjektų teises ir pareigas, leistiną ir ginamą elgesį ir tokio elgesio ribas.-Tačiau administracinės teisės normos turi ir savitų požymių. Pirma, jos reguliuoja visuomeninius santykius viešojo valdymo srityje, nors dalis tokių normų reguliuoja specifinius nevaldymo pobūdžio santykius (pvz., teismams nagrinėjant administracines bylas). Antra, šios normos reguliuoja visuomeninius santykius administraciniu teisiniu metodu. Trečia, kadangi administracinės teisės normose nustatytos reguliuojamo visuomeninio santykio subjektų teisės ir pareigos, tai suteikia šiems subjektams įgaliojimus veikti valstybės vardu. Ketvirta, viešojo valdymo pobūdis lemia šių teisės normų imperatyvumą, kuris gali pasireikšti trejopai: 1) kaip tiesioginis paliepimas, įpareigojantis subjektus elgtis tik nurodytu būdu; 2) kaip galimybė pasirinkti vieną iš nurodytų elgesio variantų; 3) kaip suteikimas subjektui teisės elgtis savo nuožiūra, tačiau neperžengiant teisės normos apibrėžtų elgesio ribų. Taigi apibendrinus galima teigti, kad administracinės teisės normos yra savarankiškos ir specifinės teisės normos, kurios, taikant administracinį teisinį metodą, reguliuoja visuomeninius santykius viešojo valdymo srityje, taip pat specifinius (procesinius) nevaldymo pobūdžio santykius. Administracinės teisės norma turi savo vidinę struktūrą, t. y. šiuos elementus: 1) hipotezę, 2) dispoziciją ir 3) sankciją. Hipotezė -tai normos dalis, nurodanti konkrečias aplinkybes ir sąlygas, kuriomis ši norma turi būti vykdoma ar taikoma. Kitaip sakant, hipotezėje nurodomos aplinkybės ar juridiniai faktai, kurių pagrindu atsiranda atitinkami administraciniai teisiniai santykiai. Pavyzdžiui, Valstybės tarnybos įstatymo 9 straipsnyje pasakyta: „Asmuo, priimamas į valstybės tarnautojo pareigas, turi atitikti šiuos bendruosius reikalavimus: 1) turėti Lietuvos Respublikos pilietybę; 2) mokėti lietuvių kalbą; 3) būti ne jaunesnis kaip 18 metų ir ne vyresnis kaip 62 metų ir 6 mėnesių”. Pagal apibrėžtumo laipsnį administracinės teisės normos hipotezė gali būti absoliučiai ar sąlygiškai apibrėžta arba visiškai neapibrėžta. Neapibrėžtos hipotezės dažniausiai yra tose normose, kurios reglamentuoja viešojo valdymo subjektų teisinę padėtį arba jų tarpusavio santykius. Dispozicija -tai svarbiausia administracinės teisės normos dalis, nustatanti tam tikrą elgesio taisyklę. Čia suformuluotos reguliuojamo visuomeninio santykio subjektų teisės, pareigos, įgaliojimai, draudimai, apribojimai. Pavyzdžiui, Valstybės tarnybos įstatymo 15 straipsnio 1 dalyje sakoma: „Valstybės tarnautojai privalo: 1) laikytis Lietuvos Respublikos Konstitucijos ir įstatymų; 2) būti lojalūs Lietuvos valstybės ir jos konstitucinei santvarkai; 3) gerbti žmogaus teises ir laisves, tarnauti visuomenės interesams” ir t. t. Dispozicija gali būti absoliučiai apibrėžta, sąlygiškai apibrėžta ir neapibrėžta.
137
ADMINISTRACINE TEISE
Sankcija -tai valstybinio poveikio (administracinio, drausminio) priemonės, kurios taikomos asmenims, pažeidusiems dispoziciją (teisės normoje nustatytą elgesio taisyklę). Pavyzdžiui, ATPK 128 straipsnyje nustatyta: „Transporto priemonių vairavimas neturint teisės jų vairuoti ar neturint teisės vairuoti šios rūšies transporto priemonių užtraukia baudą nuo trijų iki penkių šimtų litų”. Pagal poveikio tikslus ir pobūdį administracinės teisės normų sankcijos gali būti skirstomos į administracines nuobaudas (bauda, specialiųjų teisių atėmimas, administracinis areštas ir kt.) ir administracines kardomąsias priemones (administracinis asmens sulaikymas, nušalinimas nuo transporto priemonių vairavimo ir kt.). Administracinės teisės normos negalima tapatinti su teisės aktu ar jo straipsniu. Ji ypatinga tuo, kad atskiros jos dalys (ypač sankcija) neretai yra suformuluotos ne tame pačiame, o skirtinguose akto straipsniuose arba net kituose teisės aktuose (žr., pvz., ATPK 161, 162 str.).
2.2. ADMINISTRACINĖS TEISĖS NORMŲ RŪŠYS Administracinės teisės normos gali būti rūšiuojamos pagal įvairius kriterijus. Teisinėje literatūroje dažniausiai jos grupuojamos pagal: 1) teisės normos turinį; 2) normos taikymo sferą; 3) subjektus, kuriems taikomos normos; 4) normų tarpusavio santykį (subordinaciją); 5) poveikio subjektams pobūdį; 6) galiojimo ribas. Pagal turinį normos skirstomos į materialiąsias ir procesines administracines teisės normas. Materialiosios teisės normos nustato subjektų teisinę padėtį, jų teises, pareigas. Procesinės administracinės teisės normos reglamentuoja materialiųjų teisės normų realizavimo tvarką. Pagal taikymo sferą administracinės teisės normos skirstomos į bendrąsias, specialiąsias ir ypatingąsias. Bendrosios normos reguliuoja tam tikrą grupę vienarūšių santykių arba nustato bendrus teisinio reguliavimo pradmenis. Specialiosios normos konkretizuoja bendrąsias normas, atsižvelgiant į tam tikros rūšies visuomeninių santykių ypatumus. Pavyzdžiui, ATPK 12 straipsnyje sakoma, kad administracinio teisės pažeidimo subjektai gali būti fiziniai asmenys, kuriems iki administracinio teisės pažeidimo padarymo sukako šešiolika metų. ATPK 13 straipsnyje nurodomi nepilnamečių, jų tėvų administracinės atsakomybės ypatumai. Ypatingosios normos reguliuoja visuomeninius santykius, kai susidaro nepaprastos aplinkybės (pvz., stichinės nelaimės, karo padėtis ir pan.). Pagal subjektus, kuriems taikomos administracinės teisės normos, šios skirstomos į reguliuojančias: 1) viešojo valdymo institucijų (valstybės, savivaldybių) teisinę padėtį, jų veiklos formas ir metodus; 2) valstybės tarnautojų administracinį teisinį statusą; 3) visuomeninių ir kitų nevalstybinių organizacijų teisių padėtį viešojo valdymo srityje; 4) piliečių ir užsieniečių administracinį teisinį statusą. Pagal tarpusavio santykį (subordinaciją) administracinės teisės normos skirstomos į konstitucines normas bei įstatymus ir poįstatymines normas. Aukščiausią teisinę galią turi konstitucinės normos ir normos-įstatymai. Pastarosiomis remiantis kuriamos poįstatyminės normos. Pagal poveikio subjektams pobūdį administracinės teisės normos skirstomos į įpareigojančias, įgaliojančias, draudžiamąsias. Įpareigojančios normos reikalauja iš teisės subjektų atlikti tam tikrus normoje numatytus veiksmus. Įgaliojančios normos suteikia teisės subjektams teisę atlikti tam tikrus veiksmus arba elgtis
138
Administracinės teisės normos ir santykiai
savo nuožiūra normos apibrėžtose ribose. Draudžiamosios normos nustato tam tikrus elgesio apribojimus arba draudžia tam tikrus veiksmus. Be nurodytų rūšių, administracinės teisės normos dar gali būti skatinamosios ir rekomendacinės. Skatinamosiose normose numatytos skatinamosios priemonės subjektams. Rekomendacinėse normose subjektams siūloma atlikti tam tikrus veiksmus. Jos nėra privalomos ir iš esmės taikomos nepavaldiems subjektams.
2.3. ADMINISTRACINĖS TEISĖS NORMŲ GALIOJIMAS
Administracinės teisės normos įsigalioja kartu su teisiniu aktu, kuriame jos suformuluotos. Jų galiojimas susijęs su tam tikru laiku, erdve (teritorija) bei asmenimis. Momentas, nuo kada teisiniai aktai (kartu ir normos) įsigalioja, apibrėžiamas nevienodai. Tai nustatyta Lietuvos Respublikos įstatyme „Dėl Lietuvos Respublikos įstatymų ir kitų teisės aktų skelbimo ir įsigaliojimo tvarkos”. Administracinės teisės normos atgaline data negalioja, t. y. jos netaikomos faktams, atsiradusiems iki normos įsigaliojimo, išskyrus atvejus, kai atgalinis galiojimas įtvirtintas pačiame norminiame akte. Pagal galiojimo laiką administracinės teisės normos skirstomos į terminuotas ir neterminuotas. Terminuotos normos galiojimo laikas yra tiksliai apibrėžiamas teisės akte. Tačiau dauguma administracinės teisės normų yra neterminuotos, t. y. jų galiojimo laikas neapibrėžtas. Administracinės teisės normos nustoja galios: 1) pasibaigus jų galiojimo laikui (terminuotų normų); 2) panaikinus ar pakeitus teisinį aktą ar jo dalį, kuriame yra administracinių teisės normų; 3) Konstituciniam Teismui pripažinus teisės aktą ar jo dalį prieštaraujančiu Konstitucijai ar įstatymams; 4) administraciniam teismui pripažinus teisės aktą ar jo dalį prieštaraujančiu įstatymui; 5) pasikeitus aplinkybėms, dėl kurių buvo išleistas teisės aktas. Normų galiojimo erdvė (teritorija) priklauso nuo valstybės ir savivaldybių institucijų, leidžiančių norminius aktus, kompetencijos bei veiklos teritorijos. Administracinės teisės normos gali būti numatytos visiems teisės subjektams arba tam tikrai jų grupei. Tos normos, kurios galioja visiems teisės subjektams, paprastai vadinamos bendrosiomis. Jas dažniausiai leidžia valstybinės valdžios ir bendrosios kompetencijos viešojo valdymo institucijos. Normos, numatytos tam tikrai subjektų grupei, priskirtinos prie specialiųjų normų.
2.4. ADMINISTRACINĖS TEISĖS NORMŲ ĮGYVENDINIMAS Savo socialinę paskirtį administracinės teisės normos atlieka tada, kai jos yra įgyvendinamos. Teisės moksle skiriamos keturios administracinės teisės normų įgyvendinimo formos: 1) normų laikymasis; 2) normų vykdymas; 3) normų panaudojimas; 4) normų taikymas. Paprasčiausias administracinės teisės normų įgyvendinimo būdas – jų laikymasis. Tai savanoriškas teisės subjekto paklusimas normų reikalavimams. Šitokiu būdu paprastai įgyvendinamos draudžiamosios normos (subjektas atsisako draudžiamų veiksmų). Normos vykdomos, kai jos yra įpareigojančios. Tada subjektas jas įgyvendina aktyviais veiksmais atlikdamas tam tikras pareigas.
139
ADMINISTRACINE TEISE
Panaudojimo būdu įgyvendinamos įgaliojančios normos. Tai subjektyvių teisių įgyvendinimo forma. Tokiu atveju subjektas gali savo nuožiūra pasirinkti įstatymuose ar kituose teisės aktuose nustatytus administracinių teisės normų realizavimo būdus (pvz., su skundu kreiptis į administracinių ginčų komisiją ar į apygardos administracinį teismą). Sudėtingiausias administracinės teisės normų įgyvendinimo būdas – jų taikymas. Svarbiausia tai, kad administracinės teisės normas taiko tik kompetentingos valstybės ar savivaldybių institucijos (pareigūnai), nes šį būdą naudoti galima tik turint specialius įstatyme ar kitame teisės akte nustatytus įgaliojimus. Be to, taikant administracinės teisės normas turi būti priimamas atitinkamas teisės aktas arba normos realizavimas įforminamas kita teisine forma (pvz., išduodant piliečiui pasą tam tikruose dokumentuose padaromi atitinkami įrašai).
2.5. ADMINISTRACINIŲ TEISINIŲ SANTYKIŲ SAMPRATA IR YPATUMAI
Administraciniai teisiniai santykiai – tai administracinės teisės normų sureguliuoti visuomeniniai santykiai. Jiems yra būdinga tai, kad: 1) jie paprastai susiklosto specifinėje socialinio gyvenimo sferoje – viešajame valdyme; 2) administraciniams teisiniams santykiams būdingas privalomasis subjektas, t.y. viena šio santykio šalis paprastai turi būti viešojo valdymo subjektas (valstybės, savivaldybės institucija, jos pareigūnas) ar kitas teisės subjektas, turintis valstybinius valdingus įgaliojimus. Tai sudaro viešojo valdymo esmę, išreiškia jo valdingą pobūdį. Antroji administracinio teisinio santykio šalis gali būti bet kuris administracinės teisės subjektas; 3) jie dažniausiai atsiranda vienos šalies iniciatyva, o antrosios šalies sutikimas nėra būtinas, išskyrus atskirus įstatymų numatytus atvejus. Tokią iniciatyvą, jei yra teisės normoje numatytos sąlygos, gali pareikšti bet kuris administracinės teisės subjektas. Tačiau tai nereiškia, kad administracinio teisinio santykio šalys lygiateisės, nes tik privalomasis subjektas gali spręsti atitinkamus klausimus; 4) ginčai, kylantys tarp administracinio teisinio santykio šalių, gali būti sprendžiami ne tik administracine, bet ir teismine tvarka. Įsteigus Lietuvoje administracinius teismus ir priėmus Administracinių bylų teisenos įstatymą, daugelis ginčų, kylančių iš administracinių teisinių santykių, nagrinėjami administraciniuose teismuose; 5) administracinio teisinio santykio šalis, pažeidusi teisės normas, atsako ne antrajai santykio šaliai, bet valstybei.
2.6. ADMINISTRACINIŲ TEISINIŲ SANTYKIŲ RŪŠYS
Administraciniai teisiniai santykiai yra labai įvairūs. Juos galima suskirstyti pagal: 1) ryšio tarp administracinio teisinio santykio šalių pobūdį; 2) paskirtį; 3) turinį; 4) subjektų pobūdį; 5) gynimo būdą. Pagal ryšio tarp santykio šalių pobūdį administraciniai teisiniai santykiai yra vertikalūs ir horizontalūs. Vertikalūs (subordinaciniai) santykiai susiklosto tarp subjektų, kurie vieni kitiems pavaldūs organizacine tvarka. Tai santykiai tarp aukštesniųjų ir žemesniųjų viešojo valdymo institucijų, tarp vadovo ir jam pavaldžių tarnautojų. Tai valdžios ir pavaldumo santykiai, kurie atsiranda įgyvendinant viešojo valdymo funkcijas.
140
Administracinės teisės normos ir santykiai
Horizontalūs administraciniai teisiniai santykiai susiklosto tarp tiesiogiai nepavaldžių vienas kitam subjektų. Tai santykiai tarp vienodo lygmens viešojo valdymo institucijų, pavyzdžiui, dviejų ministerijų, valstybės ir savivaldybių institucijų, viešojo valdymo institucijų ir visuomeninių organizacijų. Kitaip sakant, tokie santykiai susiklosto tarp viešojo valdymo ir kitų teisės subjektų, kurie tarp savęs nėra susiję organizacinio pavaldumo tvarka. Pagal tikslinę paskirtį skiriami pozityvūs ir deliktinio pobūdžio administraciniai teisiniai santykiai. Pozityvūs yra tie teisiniai santykiai, kurie atsiranda viešojo valdymo institucijoms, jų pareigūnams sprendžiant įvairius organizacinius uždavinius, vadovaujant ūkinei, socialinei, kultūrinei veiklai, padedant įgyvendinti piliečių subjektines teises ir pan. Deliktinio pobūdžio administraciniai teisiniai santykiai atsiranda dėl teisės pažeidimų. Pagal konkretų turinį administraciniai teisiniai santykiai skirstomi į turtinio ir neturtinio pobūdžio santykius. Turtinio pobūdžio teisiniai santykiai atsiranda, pavyzdžiui, viešojo valdymo institucijoms paskirstant bei perduodant kitoms organizacijoms tam tikrą turtą, materialinius, finansinius išteklius. Neturtiniai teisiniai santykiai susiklosto viešojo valdymo subjektams sprendžiant švietimo, kultūros, sveikatos priežiūros bei kitus panašaus pobūdžio klausimus. Pagal subjektų pobūdį administraciniai teisiniai santykiai skirstomi į santykius, atsirandančius tarp valstybės institucijų, tarp valstybės institucijų ir tarnautojų, tarp valstybės ir savivaldybių institucijų, tarp valstybės institucijų ir visuomeninių organizacijų, tarp valstybės institucijų ir piliečių ir t. t. Atsižvelgiant į gynimo būdą, administraciniai teisiniai santykiai skirstomi į ginamus administracine tvarka ir ginamus teismo tvarka.
2.7. ADMINISTRACINIŲ TEISINIŲ SANTYKIŲ ATSIRADIMO, PASIKEITIMO IR PASIBAIGIMO PAGRINDAI
Administraciniai teisiniai santykiai atsiranda, pasikeičia ir pasibaigia juridinių faktų pagrindu. Šie faktai – tai tam tikros aplinkybės, nurodytos administracinės teisės normoje. Juridiniai faktai – tai įvykiai ir veiksmai. Įvykiai – tai tokie reiškiniai, kurie įvyksta be žmonių valios (pvz., gaivalinė nelaimė, natūrali žmogaus mirtis, laiko termino pasibaigimas). Veiksmai (neveikimas) – tai sąmoningi valiniai žmonių elgesio aktai, žmonių valingos veiklos rezultatas. Veiksmai, atsižvelgiant į jų santykį su galiojančiomis teisės normomis, gali būti teisėti ir neteisėti. Teisėti veiksmai yra tie, kurie atitinka teisės normų reikalavimus. Jie savo ruožtu skirstomi į juridinius aktus ir juridinius poelgius. Juridiniais aktais vadinami tokie veiksmai, kuriais tiesiogiai, sąmoningai siekiama teisinių rezultatų. Tokie, pavyzdžiui, yra viešojo valdymo institucijų leidžiami individualūs valdymo aktai. Juridiniai poelgiai yra tokie veiksmai, kuriais subjektas tiesiogiai nesiekia tam tikrų teisinių rezultatų, tačiau tų poelgių rezultatai yra tokie reikšmingi, kad sukelia teisinius padarinius. Veiksmai, pažeidžiantys teisės normas, vadinami neteisėtais. Tai teisės pažeidimai, kurių pagrindu kyla administraciniai teisiniai santykiai.
141
3. VIEŠASIS VALDYMAS
3.1. VIEŠOJO VALDYMO SAMPRATA IR PAGRINDINIAI BRUOŽAI
Kaip jau minėta, sąvoka „valdymas” plačiai vartojama įvairiose visuomenės ir gamtos mokslų šakose – filosofijoje, ekonomikoje, teisėje, mechanikoje, bionikoje ir kt. Plačiąja prasme ši sąvoka reiškia įvairius valdymo procesus, kurie vyksta visuomenėje, gyvojoje ir negyvojoje gamtoje, ir juos atitinkamais aspektais tiria įvairūs mokslai. Valdymo procesai visada vyksta tam tikroje organizuotoje sistemoje. Sistema susideda iš valdymo subjekto (valdančiojo) ir valdymo objekto (valdomojo). Valdymo subjektas, spręsdamas tam tikrus uždavinius, daro kryptingą poveikį valdymo objektui. Valdymo procesams organizuoti reikalinga informacija, tam tikros žinios. Remdamasis tokia informacija, valdymo subjektas daro tam tikrus sprendimus ir juos siunčia valdymo objektui, ir šis privalo tuos sprendimus vykdyti. Tokiu būdu valdymo procese tarp valdymo subjekto ir valdymo objekto palaikomas nuolatinis ryšys ir sprendžiami valdymo uždaviniai bei vykdomos tam tikros funkcijos. Taigi valdymo struktūrą sudaro jo subjektas, objektas ir funkcijos. Atsižvelgiant į tai, galima teigti, kad valdymas – tai tam tikroje sistemoje pasireiškiantis tikslingas bei kryptingas poveikis, kurį valdymo subjektas daro valdymo objektui, siekdamas išlaikyti sistemą tam tikroje organizacinėje struktūroje, išspręsti reikiamus uždavinius, pasiekti užsibrėžtą tikslą. Mokslinėje literatūroje išskiriamos tokios trys valdymo rūšys: 1) mechaninis (mašinų, technologijos procesų) valdymas; 2) biologinis (procesų, vykstančių gyvuosiuose organizmuose) valdymas; 3) socialinis (žmonių, jų kolektyvų veiklos, elgesio) valdymas. Su teisės reiškiniais artimiausiai susijęs socialinis valdymas, kuris atsirado susiformavus žmonių visuomenei ir pasidarė būtina jos socialine funkcija. Tai, kad socialinis valdymas yra būtinas, lemia bendra žmonių veikla bei jų gyvenimo poreikiai. Socialinio valdymo paskirtis yra visuomenės ar jos dalies bendro gyvenimo bei veiklos organizavimas ir koordinavimas, atsižvelgiant į bendrus visuomenės tikslus bei interesus. Kadangi socialinį valdymą įgyvendina žmonės, jų kolektyvai, nuo žmonių veiklos ypatumų priklauso jo specifika. Tik žmonės sugeba vertinti aplinkybes, pasirinkti tinkamiausius elgesio variantus, veikti tikslingai ir valingai. Socialiniam valdymui būdinga tai, kad bendros veiklos žmonių elgesys yra pajungiamas vieningai valdančiajai – valdymo subjekto – valiai. Didelę reikšmę šiame procese turi tokios valdymo formos, kaip organizacinė veikla, norminis reguliavimas, kontrolė, priežiūra, poveikio priemonių taikymas socialinės veiklos dalyviams, jų skatinimas, auklėjimas, kurių pagalba siekiama teigiamai paveikti žmonių sąmonę, valią ir elgesį. Todėl socialinio valdymo objektu reikia laikyti ne atskirus individus, jų grupes, o jų visuomeniškai reikšmingą elgesį, veiksmus. Šiuolaikinėje visuomenėje egzistuoja įvairių formų bei paskirties socialinio valdymo sistemos. Tai valstybinės bei savivaldybių institucijos ir visuomeninės bei kitos nevalstybinės organizacijos. Atsižvelgiant į tai, skiriamos dvi stambios socialinio valdymo rūšys – viešasis valdymas ir visuomeninis.
142
Viešasis valdymas
Visuomeninis valdymas – tai savanoriškumo principu susikūrusių gyventojų visuomeninių organizacijų bei kitokių nevalstybinių susivienijimų organizacinė veikla. Jis vykdomas ne valstybės, o atitinkamų asmenų, susivienijusių į visuomeninę ar kitą nevalstybinę organizaciją, vardu. Be to, visuomeninis valdymas vykdomas ne teisinėmis formomis, todėl tokioje veikloje priimami valdymo aktai nėra teisiniai, jie privalomi ne visiems teisės subjektams, o tik atitinkamų visuomeninių ar kitų nevalstybinių organizacijų nariams. Tokių valdymo aktų vykdymas remiasi ne valstybine prievarta, bet visuomeninio, moralinio pobūdžio poveikio priemonių taikymu. Viešasis valdymas yra savarankiška socialinio valdymo rūšis. Ši veikla turi viešąjį, arba valstybinį, interesą, jos esmę ir pobūdį lemia valstybės prigimtis. Svarbiausieji viešojo valdymo požymiai yra: 1) valdymo procese įgyvendinami valstybės uždaviniai, funkcijos ir interesai; 2) valdymo funkcijas vykdo specialūs subjektai, kuriuos formuoja valstybė įstatymų nustatyta tvarka; 3) šie subjektai veikia valstybės vardu ar jos pavedimu; 4) šiems subjektams suteikiami valstybės valdingi įgaliojimai; 5) tokių subjektų kompetenciją bei veiklos ribas nustato valstybė įstatymais ir kitais aktais. Valstybės vieningos valdžios veikla skaidoma į tris pagrindines rūšis: įstatymų leidžiamąją, įstatymų vykdomąją ir teismo veiklą. Šis valstybės valdžios veiklos skaidymas turi tvirtą teisinį pagrindą – jis įtvirtintas Lietuvos Respublikos Konstitucijoje ir kituose įstatymuose. Viešasis valdymas iš esmės yra susijęs su valstybės vykdomosios valdžios įgyvendinimu (jos vykdomąja veikla). Viešasis valdymas, kaip valstybės vykdomosios valdžios įgyvendinimo veikla, apibrėžtas 1999 m. birželio 17 d. Lietuvos Respublikos viešojo administravimo įstatyme. Jo 3 straipsnio 1 dalyje rašoma: „Viešasis administravimas – įstatymais ir kitais teisės aktais reguliuojama valstybės ir vietos savivaldos institucijų, kitų įstatymais įgaliotų subjektų vykdomoji veikla, skirta įstatymams, kitiems teisės aktams, vietos savivaldos institucijų sprendimams įgyvendinti, numatytoms viešosioms paslaugoms administruoti”. Iš įstatyme pateikto apibrėžimo galima daryti išvadą, kad vadinamasis viešasis administravimas iš esmės gali būti traktuojamas kaip viešasis valdymas, nors viešojo administravimo sąvoka, apibrėžta minėtame įstatyme, yra siauresnė už viešojo valdymo sąvoką. Viešasis valdymas neapsiriboja vien tik viešojo administravimo sritimis, nurodytomis minėto įstatymo 5 straipsnyje, tai yra administraciniu reglamentavimu (administracinio reglamentavimo teisės aktų, tam tikrų taisyklių priėmimu), vidaus administravimu ir viešųjų paslaugų teikimo administravimu. Viešasis valdymas apima ir tokias organizacinio darbo formas, kaip vadovavimas ūkinei, socialinei kultūrinei, administracinei politinei veiklai. Viešajam valdymui priklauso valstybės tarnybos organizavimas, valdymo aktų leidimas, kontrolės ir administracinės priežiūros vykdymas, administracinio ir drausminio poveikio priemonių taikymas, viešosios tvarkos ir visuomenės saugumo užtikrinimas, asmens teisių gynimas ir kt. Savo turiniu valstybės vykdomosios valdžios veikla (vykdomoji veikla) sudaro tik viešojo valdymo branduolį. Pagal Konstituciją ir kitus įstatymus valstybės vykdomąją valdžią iš esmės įgyvendina tokie specialieji subjektai: Vyriausybė, ministerijos, Vyriausybės įstaigos, kitos valstybės ir savivaldybių institucijos bei įstaigos, kurios turi atitinkamus valdingus įgaliojimus ir paprastai veikia valstybės vardu. Tuo tarpu kai kurie viešojo valdymo veiksmai atliekami ir kitose valstybės instituci
143
ADMINISTRACINE TEISE
jose – Seime, teismuose, prokuratūroje, nors šios valstybės institucijos vykdomosios valdžios funkcijų nevykdo. Pavyzdžiui, Seimą, kaip įstatymų leidžiamąją instituciją, aptarnauja jo administracinis aparatas – kanceliarija ir kiti padaliniai, kurie tvarko raštvedybą, rūpinasi darbo personalu, patalpomis, įranga ir pan., kad būtų užtikrintos normalios šios įstatymų leidžiamosios institucijos veiklos sąlygos. Panašios vidaus valdymo funkcijos vykdomos ir teismuose, kurių pirmininkai organizuoja teisėjų ir aptarnaujančio personalo darbą, užtikrina darbo drausmę, rūpinasi ūkiniais reikalais ir pan. Šiais atvejais teismų pirmininkai, kai jie procesine tvarka nesvarsto bylų, yra ne teisminės valdžios atstovai, o administratoriai, kurie užtikrina reikiamas teismų vidaus veiklos sąlygas. Tokia minėtų institucijų vidinio valdymo veikla yra pagalbinė ir ji pagal savo pobūdį ir vykdymo subjektus valstybės vykdomajai valdžiai nepriskiriama, nors ją reglamentuoja įstatymai ir kitos administracinės teisės normos. Viešojo administravimo įstatyme (3 str. 3 d.) vidaus valdymas apibūdintas taip: „Vidaus administravimas – administravimo veikla, kuria užtikrinamas valstybės ar vietos savivaldos konkrečios institucijos, įstaigos, tarnybos ar organizacijos savarankiškas funkcionavimas (struktūros tvarkymas, personalo valdymas, turimų materialinių ir finansinių išteklių valdymas ir naudojimas, raštvedybos tvarkymas), kad jos galėtų tinkamai vykdyti joms priskirtus viešojo administravimo ar kitos valstybės veiklos uždavinius”. Reikia pabrėžti, kad valstybės vykdomosios valdžios veiklai nepriskiriamos įmonių, taip pat įstaigų, teikiančių socialines paslaugas (švietimo, mokslo, kultūros, sveikatos priežiūros ir kt.) administracijos valdymo funkcijos, nes šių organizacijų administracija nėra valstybės vykdomosios valdžios subjektas. Taigi dar kartą pravartu pasakyti, kad viešojo valdymo turinį sudaro: 1) valstybės ir vietos savivaldos institucijų vykdomoji veikla (viešojo valdymo branduolys); 2) kitų įstatymais įgaliotų subjektų vykdomoji veikla; 3) visų valstybės ir savivaldybių institucijų vidaus valdymo funkcijos; 4) įmonių ir socialinių įstaigų administracijos valdymo pobūdžio veikla. Viešajam valdymui, kaip savarankiškai valstybės veiklos rūšiai, būdingi tokie pagrindiniai bruožai: 1) viešasis valdymas visų pirma yra vykdomojo pobūdžio veikla. Svarbiausia jo paskirtis – įgyvendinti įstatymus ir kitus teisės aktus; 2) viešąjį valdymą vykdo specialieji subjektai -valstybės ir vietos savivaldos vykdomosios valdžios institucijos (jų pareigūnai), kurių vykdomoji veikla sudaro viešojo valdymo branduolį; 3) kai kurias valdymo pobūdžio funkcijas vykdo visų valstybės ir savivaldybių institucijų administracija. Tai vidaus valdymo funkcijos, kurios yra viešojo valdymo sudėtinė dalis; 4) viešasis valdymas -poįstatyminė veikla. Ji vykdoma remiantis įstatymais ir praktiškai įgyvendinant įstatymus bei kitus teisės aktus. Įstatymai ir kiti jų pagrindu priimti teisės aktai nustato valdymo kryptis, apimtį, jo formas ir metodus; 5) viešojo valdymo veiklos sfera labai plati. Ji apima visas svarbiausias valstybinio gyvenimo sritis – tautos ūkinę, socialinę kultūrinę ir administracinę politinę. Be to, viešojo valdymo sferoje piliečiai įgyvendina daugumą jiems priklausančių teisių ir pareigų, joje saugomos ir ginamos asmens teisės ir teisėti interesai; 6) viešasis valdymas – tai kasdienė, nenutrūkstama, operatyvi, organizacinio pobūdžio veikla. Valdymo procese mobilizuojamos ir nukreipiamos kolektyvinių ir individualių teisės subjektų pastangos valdymo uždaviniams spręsti.
144
Viešojo valdymo subjektai 3.2. VIEŠOJO VALDYMO PRINCIPAI
Viešojo valdymo principai – tai pagrindinės idėjos, nuostatos, kuriomis remiantis turi būti sudaryta ir turi funkcionuoti viešojo valdymo sistema bei šio valdymo subjektų veikla. Principais laikytinos tik tokios idėjos, vadovavimą įtvirtinančios nuostatos, reikalavimai, kurie yra objektyvūs, atspindi realią tikrovę, moksliškai pagrįsti. Kadangi viešojo valdymo sfera labai plati, tai įvairūs ir jo principai. Pagrindiniai viešojo valdymo principai yra įtvirtinti galiojančiuose įstatymuose. Jų turi būti laikomasi visose viešojo valdymo srityse, jais turi būti vadovaujamasi organizuojant viešojo valdymo subjektų sistemą bei vykdant praktinę viešojo valdymo veiklą. Lietuvos Respublikos viešojo administravimo įstatyme (4 str.) įtvirtinti tokie svarbiausi valdymo principai: 1.Įstatymų viršenybės principas. Jis reiškia, kad viešojo valdymo subjektų kompetencija turi būti nustatyta įstatymo, o veikla atitikti įstatyme išdėstytus teisinius pagrindus. Valdymo aktai, susiję su asmenų teisių ir pareigų įgyvendinimu, visais atvejais turi būti pagrįsti įstatymais. 2. Objektyvumo principas. Jo esmė yra ta, kad sprendimo priėmimas ir kiti oficialūs viešojo valdymo veiksmai turi būti nešališki, objektyvūs. 3. Proporcingumo principas. Jis reiškia, kad administracinio sprendimo mastas bei griežtumas turi būti proporcingi valdymo tikslui. Pavyzdžiui, konkreti drausminio poveikio priemonė turi būti nustatyta bei taikoma atsižvelgiant į padaryto teisės pažeidimo pobūdį bei jo pavojingumo visuomenei laipsnį. 4. Nepiktnaudžiavimo valdžia principas. Jis reiškia, kad viešojo valdymo institucijoms draudžiama vykdyti veiklą, kuriai jos neturi suteiktų įgaliojimų, arba priimti sprendimus pagal savo kompetenciją siekiant tikslų, kurie įstatyme nenumatyti. 5. Tarnybinio bendradarbiavimo principas. Jis reiškia, kad viešojo valdymo institucijos, rengdamos administracinius aktus, prireikus teikia viena kitai reikalingą informacinę ir kitokią pagalbą. Be Viešojo administravimo įstatyme nurodytų principų, dar galima paminėti teisėtumo, socialinio teisingumo, asmeninės atsakomybės, viešumo, lygiateisiškumo, lojalumo, politinio neutralumo, karjeros ir kitus principus, kurie yra įtvirtinti kituose įstatymuose. Visi šie principai yra universalūs, vienodai svarbūs, jų turi būti laikomasi visose viešojo valdymo srityse, visų valdymo institucijų, jų pareigūnų ir kitų viešojo valdymo subjektų veikloje.
4. VIEŠOJO VALDYMO SUBJEKTAI
4.1. VIEŠOJO VALDYMO SUBJEKTŲ SAMPRATA
Viešojo valdymo subjektai – tai valstybės ir savivaldybių institucijos, įstaigos, tarnybos, valstybės tarnautojai (pareigūnai), kiti subjektai, kuriems įstatymuose numatyta įgyvendinti valstybės vykdomosios valdžios uždavinius ar atskiras šios valdžios funkcijas.
145
ADMINISTRACINE TEISE
Viešojo valdymo subjektų sistema plati. Ją sudaro kolektyviniai ir fiziniai, valstybiniai ir nevalstybiniai subjektai. Svarbiausius viešojo valdymo subjektus formuoja atstovaujamosios valdžios institucijos – Seimas ir savivaldybių tarybos. Todėl viešojo valdymo subjektai už savo veiklą yra atsakingi ir atskaitingi atstovaujamosios valdžios institucijoms (Konstitucijos 96, 119 str.). Tam, kad viešojo valdymo subjektai galėtų sėkmingai vykdyti valstybės vykdomosios valdžios uždavinius, jiems įstatymai suteikia plačius valdingo pobūdžio valstybės įgaliojimus. Viešojo valdymo subjektų veikla yra labai įvairiapusė. Jie savo kompetenciją įgyvendina nustatytoje teritorijoje ir tam tikrose valdymo srityse bei šakose. Savo veiklą šie subjektai vykdo įstatymų nustatytomis formomis ir metodais. Viešojo valdymo subjektams yra būdingos visos trys valstybinės veiklos teisinės formos: teisės kuriamoji, teisės vykdomoji (taikomoji) ir teisės apsaugos. Tačiau kalbant apie teisės normų kūrimą, svarbu pabrėžti, kad ši veikla nėra tolygi įstatymų leidžiamosios institucijos – Seimo analogiškai veiklai. Viešojo valdymo subjektai kuria teisės normas remdamiesi ir vadovaudamiesi galiojančiais įstatymais. Taigi viešojo valdymo subjektų priimami teisės aktai visada yra poįstatyminiai. Viešojo valdymo subjektams būdinga ir tai, kad jų veikloje dominuoja valdžiospavaldumo santykiai, kurie paprastai reguliuojami administraciniais metodais. Tam, kad priimti įstatymai būtų įvykdyti viešojo valdymo srityje, be įtikinimo, moralinio ir materialinio skatinimo priemonių, dažnai naudojamos įvairios administracinės bei drausminės poveikio priemonės. Jas taiko kompetentingi viešojo valdymo subjektai ir teismas. Viešojo valdymo subjektų teisinė padėtis turi tam tikrų ypatumų. Valdžios atstovaujamųjų institucijų ir teismų teisinė padėtis nustatoma tik įstatymais, tuo tarpu viešojo valdymo subjektų teisinis statusas apibrėžiamas ne tik įstatymais, bet ir valdymo aktais. Viešojo valdymo subjektų vidinė organizacinė struktūra įvairi ir sudėtinga. Taip yra todėl, kad valdymo subjektų yra labai daug, jie įvairūs, be to, kiekvieno jų veikla turi savo ypatumų. Kai kurie viešojo valdymo subjektai (pvz., Vyriausybė) sprendžia bendrosios kompetencijos uždavinius, kiti subjektai atlieka kontrolės, priežiūros, vadovavimo specialiosioms valdymo šakoms funkcijas. Atsižvelgiant į valdymo ypatumus, galima teigti, kad viešojo valdymo subjektai tai valstybės ir savivaldybių institucijos, įstaigos, tarnybos, valstybės tarnautojai (pareigūnai) ir kiti įstatymuose numatyti subjektai, kurių tiesioginė paskirtis – įgyvendinti valstybės vykdomosios valdžios uždavinius arba atskiras jos funkcijas, vadovauti socialinės kultūrinės, ūkinės ir administracinės politinės veiklos sritims. Paprastai viešojo valdymo subjektai yra pavaldūs Vyriausybei. Tačiau funkcionuoja ir kai kurios Seimo įsteigtos ir tik jam atskaitingos, Vyriausybei nepavaldžios valstybės institucijos. Šios institucijos turi specialųjį statusą ir vykdo tam tikras valdymo funkcijas. Joms priklauso, pavyzdžiui, Lietuvos bankas, Valstybės kontrolė, Valstybės saugumo departamentas, Valstybinė lietuvių kalbos komisija ir kt. Viešojo valdymo subjektams taip pat priskiriama visų valstybės institucijų (Seimo, teismų), valstybės ir savivaldybių įstaigų administracija. Viešojo valdymo subjekto teisinę padėtį rodo jo kompetencija. Į viešojo valdymo subjekto kompetenciją įeina uždaviniai, funkcijos, teisės ir pareigos, veiklos formos ir metodai.
146
Viešojo valdymo subjektai
Viešojo valdymo subjekto uždaviniai – tai nurodymas, ką jis turi pasiekti savo veikloje arba užtikrinti savo veikla. Viešojo valdymo subjekto funkcijos – tai pagrindinės jo veiklos kryptys realizuojant jam iškeltus uždavinius. Valdymo funkcijos parodo, iš kokių sudėtinių dalių susideda subjekto veikla, jos turinys. Be to, viešojo valdymo subjekto funkcijos atspindi jo veiklos specializaciją. Viešojo valdymo subjekto teisės ir pareigos parodo jo funkcijų apimtį ir įgyvendinimo ribas. Viešojo valdymo subjekto veiklos forma – tai uždavinių ir funkcijų įgyvendinimo būdas, valdymo veiklos išorinė išraiška. Vykdydamas pavestus uždavinius ir funkcijas, viešojo valdymo subjektas atlieka konkrečius veiksmus, kurie pagal tam tikrus požymius jungiami į grupes, kurias priimta vadinti valdymo formomis. Svarbiausios teisinės valdymo formos yra norminių ir individualių aktų priėmimas, šių aktų įgyvendinimas, įvairių klausimų svarstymas ir kt. Viešojo valdymo subjekto veiklos metodai – tai poveikio priemonės, kurios taikomos teisės subjektams, norint užtikrinti, kad valdymo sprendimai būtų vykdomi. Poveikio priemonės gali būti įvairios – įtikinimo, prievartos, administracinio ir ekonominio pobūdžio.
4.2. VIEŠOJO VALDYMO SUBJEKTŲ RŪŠYS
Kadangi viešojo valdymo subjektų yra labai daug ir jie labai įvairūs, juos galima surūšiuoti į tam tikras grupes. Tokio rūšiavimo kriterijai gali būti viešojo valdymo subjektų formavimo tvarka, jų teisinė prigimtis, kompetencijos pobūdis, klausimų sprendimo būdas, veiklos teritorija. Pagal formavimo tvarką viešojo valdymo subjektai gali būti sudaromi (pvz., Vyriausybė), steigiami (pvz., ministerijos, vyriausybės įstaigos ir įstaigos prie ministerijų), paskiriami (pvz., Vyriausybės atstovas apskrityje). Šių subjektų formavimo tvarka nustatyta įstatymuose. Pagal teisinę prigimtį viešojo valdymo subjektai gali būti valstybiniai ir nevalstybiniai (vietos savivaldos). Pagal kompetencijos pobūdį viešojo valdymo subjektai gali būti: 1) bendrosios kompetencijos subjektai, t. y. tie, kurie yra plačiausios, įvairiapusės kompetencijos, sprendžia įvairius socialinius kultūrinius, ūkinius, administracinius politinius klausimus (pvz., Vyriausybė). Šiems subjektams būdinga ir tai, kad jie tam tikroje teritorijoje jungia kitus valdymo subjektus, vadovauja jų veiklai bei ją koordinuoja; 2) šakinės kompetencijos subjektai, t. y. tokie, kurie vadovauja tam tikrai socialinės kultūrinės, ūkinės ar administracinės politinės veiklos šakai (pvz., ministerijos). Pagal klausimų sprendimo būdą viešojo valdymo subjektai skirstomi į kolegialius ir vienvaldžius. Kolegialūs subjektai (pvz., Vyriausybė) svarbiausius savo kompetencijos klausimus sprendžia posėdžiuose, o valdymo aktai priimami subjekto narių balsų dauguma. Vienvaldžių subjektų (pvz., ministerijų) kompetencijos klausimus sprendžia jų vadovai vienasmeniškai. Šiai grupei priklauso dauguma viešojo valdymo subjektų.
147
ADMINISTRACINĖ TEISĖ
Pagal veiklos teritoriją viešojo valdymo subjektai skirstomi į: 1) centrinius valstybinio valdymo subjektus, kurie valdymą vykdo visoje valstybės teritorijoje (Vyriausybė, ministerijos); 2) teritorinius valstybinio valdymo subjektus, kurie valdymą vykdo tam tikroje teritorijoje (apskrities viršininkas); 3) teritorinius nevalstybinio valdymo subjektus (savivaldybės). Pagal finansavimo šaltinius viešojo valdymo subjektai skirstomi į biudžetinius ir ūkiskaitinius.
4.3. VIEŠOJO VALDYMO SUBJEKTŲ SISTEMA
Viešojo valdymo sistema apima įvairių rūšių, skirtingos tikslinės paskirties, nevienodos teisinės padėties valstybinius ir nevalstybinius (savivaldybių ir kt.) viešojo valdymo subjektus. Tarp įvairių šių viešojo valdymo sistemos subjektų yra pastovūs ryšiai ir kiekvienas iš jų šioje sistemoje užima tam tikrą vietą. Viešojo valdymo subjektus vienija tai, kad jie įgyvendina tą pačią valstybės veiklos rūšį – vykdomąją valdžią. Visi jie tiesiogiai ar netiesiogiai atsakingi atstovaujamosios valdžios institucijoms, atitinkamiems aukštesniesiems valdymo subjektams. Valstybiniai viešojo valdymo subjektai susiję tarpusavyje organizaciniais pavaldumo arba koordinavimo ir kontrolės santykiais. Valstybiniai ir savivaldybių valdymo subjektai turi tam tikrą savarankiškumą ir atitinkamus valdingo pobūdžio įgaliojimus, kurie būtini jiems pavestiems uždaviniams ir funkcijoms įgyvendinti. Savivaldybių valdymo institucijos yra visiškai savarankiškos ir nepavaldžios valstybiniams viešojo valdymo subjektams. Vienas iš svarbiausių principų, kuriuo remiantis formuojami viešojo valdymo subjektai, yra teritorinis principas. Pagal šį principą formuojami centriniai ir administraciniai teritoriniai subjektai. Centriniai valstybiniai viešojo valdymo subjektai formuojami valdymo veiklai atlikti visoje valstybės teritorijoje. Administraciniai teritoriniai viešojo valdymo subjektai formuojami valstybės administraciniuose teritoriniuose vienetuose (apskrityse, savivaldybėse) ir jie savo veiklą vykdo atitinkamos teritorijos ribose. Minėtinas ir kitas labai svarbus – šakinis – viešojo valdymo subjektų steigimo principas. Pagal šį principą steigiami centriniai šakiniai valstybinio valdymo subjektai – ministerijos bei kitos įstaigos, kurios valdymą vykdo tam tikroje socialinės kultūrinės, ūkinės ar administracinės politinės veiklos šakoje. Taigi pagal Lietuvos Respublikos Konstituciją ir Viešojo administravimo įstatymą (8 str.) viešojo valdymo subjektų sistemą sudaro: 1) valstybinio valdymo (administravimo) subjektai -valstybės institucijos, įstaigos, tarnybos bei valstybės tarnautojai (pareigūnai); 2) savivaldybių valdymo subjektai – savivaldybės taryba, savivaldybės administracija, joms pavaldžios įstaigos, tarnybos; 3) kiti viešojo valdymo subjektai – viešosios įstaigos ir nevyriausybinės organizacijos, kurioms įstatymų nustatyta tvarka suteikti viešojo valdymo įgaliojimai.
4.4. LIETUVOS RESPUBLIKOS VYRIAUSYBĖ
Lietuvos Respublikos Vyriausybė – tai aukščiausioji kolegiali valstybės institucija, įgyvendinanti vykdomąją valdžią Lietuvoje. Vyriausybės sudarymo ir veiklos teisi
148
Viešojo valdymo subjektai
niai pagrindai, sudėtis, sudarymo tvarka, kompetencija yra apibrėžta Konstitucijos VII skirsnyje ir Vyriausybės įstatyme. Vyriausybė savo veiklą grindžia kolegialumo, demokratijos, teisėtumo ir viešumo principais. Ji solidariai atsako Seimui už bendrą savo veiklą. Ne rečiau kaip kartą per metus ji pateikia Seimui savo programos įgyvendinimo ataskaitą. Seimo reikalavimu jo statuto nustatyta tvarka Vyriausybė arba atskiri ministrai atsiskaito Seime už savo veiklą. Vyriausybę sudaro Ministras Pirmininkas ir ministrai (apie jų skyrimo tvarką bei Seimo ir Respublikos Prezidento kompetenciją jau minėta šio vadovėlio antrame skyriuje „Konstitucinė teisė”). Ministras Pirmininkas ir ministrai turi neliečiamumo garantijas. Jie negali būti patraukti baudžiamojon atsakomybėn ar suimti, taip pat negali būti suvaržyta jų laisvė be išankstinio Seimo sutikimo, o tarp Seimo sesijų – be išankstinio Respublikos Prezidento sutikimo. Ministras Pirmininkas ir ministrai negali eiti kitų renkamų ar skiriamų pareigų (Seimo nariai gali eiti Ministro Pirmininko ar ministro pareigas), negali dirbti verslo, komercijos ar kitose privačiose įstaigose ar įmonėse, taip pat gauti kitą atlyginimą, išskyrus jiems nustatytą darbo užmokestį pagal pareigas Vyriausybėje bei užmokestį už kūrybinę veiklą. Ministras Pirmininkas ir ministrai turi teisę Seimo statuto nustatyta tvarka dalyvauti Seimo, jo komitetų, komisijų bei frakcijų posėdžiuose ir pareikšti savo nuomonę svarstomais klausimais. Ministras Pirmininkas arba ministras, kuriam Seimo sesijoje yra pateiktas Seimo nario paklausimas dėl Vyriausybės, ministerijų ar kitų Vyriausybės įstaigų veiklos, privalo atsakyti žodžiu ar raštu Seimo statuto nustatyta tvarka. Ministras Pirmininkas ir ministrai Seimo sesijos metu Seimo statuto nustatyta tvarka atsako į Seimo narių klausimus. Konstitucijoje ir Vyriausybės įstatyme nustatyti pagrindiniai Vyriausybės įgaliojimai (taip pat žr. šio vadovėlio antro skyriaus „Konstitucinė teisė” 4.2 skirsnį). Taigi Lietuvos Respublikos Vyriausybė: 1) tvarko krašto reikalus, saugo Lietuvos Respublikos teritorijos neliečiamybę, garantuoja valstybės saugumą ir viešąją tvarką; 2) vykdo įstatymus ir Seimo nutarimus dėl įstatymų įgyvendinimo, taip pat Respublikos Prezidento dekretus; 3) koordinuoja ministerijų ir kitų Vyriausybės įstaigų veiklą, turi teisę panaikinti ministrų, Vyriausybės įstaigų ir įstaigų prie ministerijų teisės aktus; 4) rengia valstybės biudžeto projektą ir teikia jį Seimui; vykdo valstybės biudžetą, teikia Seimui biudžeto įvykdymo apyskaitą; 5) rengia ir teikia Seimui svarstyti įstatymų projektus; 6) užmezga diplomatinius santykius ir palaiko ryšius su užsienio valstybėmis ir tarptautinėmis organizacijomis; teikia Respublikos Prezidentui siūlymus dėl Lietuvos Respublikos diplomatinių atstovų užsienio valstybėse ir prie tarptautinių organizacijų skyrimo bei atšaukimo; 7) teikia Seimui siūlymus dėl ministerijų steigimo ir panaikinimo; 8) steigia, reorganizuoja, likviduoja Vyriausybės įstaigas bei įstaigas prie ministerijų ir paveda ministerijoms vykdyti visas ar dalį įstaigų prie ministerijų steigėjo funkcijų; Vyriausybė gali pavesti atitinkamai ministerijai vykdyti dalį Vyriausybės įstaigos steigėjo funkcijų; 9) tvirtina ministerijų, Vyriausybės įstaigų ir įstaigų prie ministerijų nuostatus (tvirtinti įstaigų prie ministerijų nuostatus Vyriausybė gali pavesti atitinkamam ministrui); 10) įstatymo nustatyta tvarka organizuoja valdymą aukštesniuose administraciniuose vienetuose; 11) skiria ir atleidžia iš pareigų apskričių viršininkus, jų pavaduotojus bei savivaldybių veiklos administracinę priežiūrą vykdančius Vyriausybės atstovus;
149
ADMINISTRACINE TEISE
12) įstatymo numatytais atvejais siūlo Seimui įvesti tiesioginį valdymą savivaldybės teritorijoje; 13) prireikus gali sudaryti nuolatines ir laikinas komisijas; 14) turi teisę kreiptis į Konstitucinį Teismą su prašymu ištirti, ar įstatymai ar Seimo priimti teisės aktai neprieštarauja Konstitucijai; 15) turi įstatymų leidybos iniciatyvos teisę; 16) vykdo kitas pareigas, kurias Vyriausybei paveda Konstitucija ir kiti įstatymai. Vyriausybei ir jos veiklai atstovauja Ministras Pirmininkas. Ministras Pirmininkas: 1) sudaro Vyriausybę ir teikia jos sudėtį tvirtinti Respublikos Prezidentui; 2) teikia siūlymus Respublikos Prezidentui dėl ministrų skyrimo ir atleidimo; 3) valstybės tarnybos įstatymo nustatyta tvarka skiria į pareigas ir atleidžia iš jų Ministro Pirmininko politinio (asmeninio) pasitikėjimo valstybės tarnautojus (vyriausiąjį patarėją, patarėjus, atstovą spaudai ir kt.), Vyriausybės kanclerį, Vyriausybės įstaigų departamentų, kontrolės ar apskaitos funkcijas vykdančių tarnybų, agentūrų, inspekcijų vadovus; 4) Konstitucijos numatytais atvejais teikia Respublikos Prezidentui siūlymus pavesti vienam iš ministrų pavaduoti Ministrą Pirmininką; 5) teikia Seimui svarstyti Vyriausybės programą; 6) teikia Vyriausybės siūlymus dėl apskričių viršininkų, jų pavaduotojų skyrimo ir atleidimo iš pareigų; 7) kviečia Vyriausybės posėdžius ir jiems vadovauja (pirmininkauja), tvirtina Vyriausybės posėdžio darbotvarkę; 8) suteikia įgaliojimus derėtis ir pasirašyti tarptautines sutartis; 9) sudaro Vyriausybines delegacijas oficialiems vizitams į kitas šalis, taip pat dalyvauti tarptautiniuose kongresuose, konferencijose, sesijose; 10) suteikia įgaliojimus Vyriausybei atstovauti Konstituciniame Teisme ir kituose teismuose; 11) sprendžia Vyriausybės veiklos organizavimo klausimus; 12) jeigu nepritaria ministro veiklos Vyriausybėje nuostatoms, turi teisę pateikti Respublikos Prezidentui siūlymą šį ministrą atleisti; 13) pasirašo Vyriausybės nutarimus; 14) leidžia potvarkius, priima operatyvius sprendimus – pavedimus, įforminamus rezoliucijomis; 15) vykdo kitas pareigas, kurias Ministrui Pirmininkui paveda Konstitucija, Vyriausybės įstatymas ir kiti įstatymai. Vyriausybės įstatymo IX skyriaus nuostatose ir Vyriausybės reglamente įtvirtinta Vyriausybinės veiklos organizavimo tvarka. Vyriausybė valstybės valdymo reikalus sprendžia posėdžiuose visų Vyriausybės narių balsų dauguma priimdama nutarimus. Juos pasirašo Ministras Pirmininkas ir atitinkamas ministras. Vyriausybės posėdžiuose gali dalyvauti asmenys, kuriems tokią teisę numato Konstitucija ir kiti įstatymai (pvz., Valstybės kontrolierius, generalinis prokuroras, Lietuvos banko pirmininkas). Į Vyriausybės posėdžius gali būti kviečiami ir kiti asmenys. Posėdžio darbotvarkę tvirtina Ministras Pirmininkas. Svarstomu klausimu Vyriausybės posėdyje išklausomas pranešimas, kurį paprastai padaro atitinkamas ministras, ir Vyriausybės narių nuomonė. Ministrui Pirmininkui leidus, savo nuomonę gali pareikšti ir kiti posėdyje dalyvaujantys asmenys. Vyriausybės posėdis teisėtas, jeigu jame dalyvauja dauguma Vyriausybės narių. Vyriausybės posėdžiai yra protokoluojami. Protokoluose nurodomi posėdyje dalyvaujantys asmenys, svarstomą klausimą pateikęs pranešėjas, nuomones šiuo klausimu pareiškę kalbėtojai ir priimtas sprendimas. Protokolą pasirašo Ministras Pirmininkas. Vyriausybės ir Ministro Pirmininko funkcijas vykdyti yra įsteigta Vyriausybės kanceliarija. Jai vadovauja Vyriausybės kancleris, kuris pavaldus Ministrui Pirmininkui. Vyriausybės kanceliarijos struktūrą ir nuostatus tvirtina Vyriausybė.
150
Viešojo valdymo subjektai 4.5. MINISTERIJOS
Ministerijos yra šakinės kompetencijos valdymo institucijos, vadovaujančios joms pavestoms valdymo sritims (šakoms). Ministerijas Vyriausybės siūlymu steigia ir panaikina Seimas priimdamas įstatymą. Ministerija vykdo įstatymų ir kitų teisės aktų jai pavestos srities valstybės valdymo funkcijas ir įgyvendina toje srityje valstybės politiką. Jos konkrečius uždavinius ir funkcijas bei kitus teisinės padėties klausimus reglamentuoja nuostatai, kuriuos tvirtina Vyriausybė. Ministerijai vadovauja ministras, kuris: 1) sprendžia ministerijos kompetencijai priklausančius klausimus ir yra tiesiogiai atsakingas už Vyriausybės programos įgyvendinimą bei tiesioginių uždavinių ir funkcijų įgyvendinimą; 2) užtikrina įstatymų, tarptautinių sutarčių, Respublikos Prezidento dekretų, Vyriausybės nutarimų, Ministro Pirmininko potvarkių ir kitų teisės aktų vykdymą; 3) teikia Vyriausybei jos darbo reglamento nustatyta tvarka įstatymų ir kitų teisės aktų projektus; 4) leidžia įsakymus ir jais patvirtintus kitus teisės aktus, tikrina, kaip jie vykdomi. Esant reikalui, keli ministrai gali leisti bendrus įsakymus arba įsakymu patvirtintus kitus teisės aktus; 5) Vyriausybei pavedus, tvirtina įstaigų prie ministerijų nuostatus; 6) tvirtina ministerijos administracijos struktūrą ir pareigybių sąrašą neviršydamas darbo užmokesčiui skirtų lėšų; 7) tvirtina ministerijos administracijos padalinių nuostatus, koordinuoja ir kontroliuoja ministerijos padalinių, taip pat įstaigų prie ministerijų veiklą; 8) Valstybės tarnybos įstatymo nustatyta tvarka skiria į pareigas ir atleidžia iš jų ministerijos valstybės tarnautojus, įstaigų prie ministerijos vadovus; 9) nustato ministerijos valstybės sekretoriaus, ministerijos sekretorių administravimo sritis; 10) vykdo kitas įstatymų ir Vyriausybės nutarimų jam suteiktas funkcijas. Ministrą gali laikinai pavaduoti tik Ministro Pirmininko paskirtas kitas Vyriausybės narys. Ministro politinio (asmeninio) pasitikėjimo valstybės tarnautojai -viceministras, ministro patarėjai, atstovas spaudai – padeda ministrui suformuoti politines nuostatas ir prioritetus, priimti sprendimus ir juos įgyvendinti. Viceministras: 1) kontroliuoja, ar ministerijos teisės aktai ir programų projektai atitinka ministro politines nuostatas jam pavestoje valdymo srityje; 2) koordinuoja ministro politinio (asmeninio) pasitikėjimo valstybės tarnautojų veiklą; 3) ministro pavedimu atstovauja ministrui pristatydamas bei aptardamas ministro politines nuostatas ir sprendimus visuomenei, Seimo komitetuose, Vyriausybės posėdžiuose; 4) dalyvauja derinant ministerijos rengiamų teisės aktų ir programų projektus su suinteresuotomis institucijomis bei atlieka kitas ministro jam pavestas funkcijas. Ministerija turi savo administraciją. Jai vadovauja ministerijos valstybės sekretorius. Jis koordinuoja ir kontroliuoja ministerijos administracijos padalinių veiklą, užtikrina, kad įgyvendinant strateginius veiklos planus optimaliai būtų valdomi ir panaudojami finansiniai, materialiniai, intelektiniai ir informaciniai ištekliai. Ministro pavedimu valstybės tarnybos įstatymo nustatyta tvarka skiria į pareigas ir atleidžia iš jų ministerijos administracijos valstybės tarnautojus. Ministerijos valstybės sekretorius pagal savo kompetenciją leidžia potvarkius, saugo ministerijos antspaudą ir atsako už jo naudojimą. Ministerijos valstybės sekretoriaus nereikia painioti su ministerijos sekretoriais, kurie yra karjeros valstybės tarnautojai, tiesiogiai pavaldūs ministerijos valstybės
151
ADMINISTRACINE TEISE
sekretoriui ir ministrui. Ministerijos sekretoriai vykdo funkcijas ministro nustatytose administravimo srityse. Ministerijoje yra sudaroma kolegija, kuri yra ministro patariamoji institucija. Kolegijos nariai yra ministras (kolegijos pirmininkas), viceministras, ministerijos valstybės sekretorius ir ministerijos sekretoriai. Į kolegijos sudėtį gali būti įtraukiami kiti ministerijos bei kitų institucijų atstovai. Kolegijos narių skaičių nustato ir kolegijos personalinę sudėtį bei darbo reglamentą tvirtina ministras. Jis taip pat teikia klausimus kolegijai svarstyti. Pagal Vyriausybės įstatymo 29 straipsnį Lietuvos Respublikoje yra šios ministerijos: 1) Aplinkos ministerija; 2) Finansų ministerija; 3) Krašto apsaugos ministerija; 4) Kultūros ministerija; 5) Socialinės apsaugos ir darbo ministerija; 6) Susisiekimo ministerija; 7) Sveikatos apsaugos ministerija; 8) Švietimo ir mokslo ministerija; 9) Teisingumo ministerija; 10) Ūkio ministerija; 11) Užsienio reikalų ministerija; 12) Vidaus reikalų ministerija; 13) Žemės ūkio ministerija.
4.6. VYRIAUSYBĖS ĮSTAIGOS IR ĮSTAIGOS PRIE MINISTERIJŲ
Vyriausybės įstatymo 33 straipsnyje pasakyta, kad ministerijų funkcijoms nepriskirtiems klausimams spręsti Vyriausybė gali steigti Vyriausybės įstaigas – departamentus, kontrolės ar apskaitos funkcijas vykdančias tarnybas, agentūras, inspekcijas ir kitas įstaigas. Vyriausybės įstaigos uždaviniai, funkcijos ir teisės nustatomos jos nuostatuose, kuriuos tvirtina Vyriausybė. Vyriausybės įstaigai vadovauja generalinis direktorius (direktorius, viršininkas), kuris pavaldus Ministrui Pirmininkui. Vyriausybės įstaigos vadovas sprendžia įstaigos kompetencijai priskirtus klausimus, taip pat vykdo kitas įstatymų ir Vyriausybės nutarimų nustatytas funkcijas. Vyriausybės įstaigos vadovas yra asmeniškai atsakingas, kad įstaiga spręstų jai pavestus uždavinius. Vyriausybės įstaigos vadovas leidžia įsakymus ir įsakymu patvirtintus kitus teisės aktus (nuostatus, taisykles), tikrina, kaip jie vykdomi. Vyriausybės įstaigos veikla organizuojama vadovaujantis Vyriausybės patvirtintais viešai paskelbtais strateginiais veiklos planais, parengtais vadovaujantis Vyriausybės programa, kuriai Seimas pritarė, ir suderintais su valstybės ilgalaikės raidos strategija. Šiuo metu veikia tokios Vyriausybės įstaigos: Europos teisės departamentas prie Lietuvos Respublikos Vyriausybės, Statistikos departamentas prie Lietuvos Respublikos Vyriausybės, Kūno kultūros ir sporto departamentas prie Lietuvos Respublikos Vyriausybės, Lietuvos archyvų departamentas prie Lietuvos Respublikos Vyriausybės, Tautinių mažumų ir išeivijos departamentas prie Lietuvos Respublikos Vyriausybės, Valstybinė atominės energetikos saugos inspekcija prie Lietuvos Respublikos Vyriausybės, Tabako ir alkoholio kontrolės tarnyba prie Lietuvos Respublikos Vyriausybės, Viešųjų pirkimų tarnyba prie Lietuvos Respublikos Vyriausybės, Valstybinė maisto ir veterinarijos tarnyba prie Lietuvos Respublikos Vyriausybės ir kt. Antrosios grandies įstaigos, kurias steigia Vyriausybė, – tai įstaigos prie ministerijų. Jų teisinė padėtis, palyginti su Vyriausybės įstaigų teisine padėtimi, yra kitokia. Įstaiga prie ministerijos priklauso tos ministerijos veiklos sričiai. Pagal Vyriausybės įstatymo 30 straipsnį prie ministerijų Vyriausybė gali steigti departa
152
Viešojo valdymo subjektai
mentus, taip pat kontrolės ar apskaitos funkcijas vykdančias tarnybas ir inspekcijas. Joms vadovauja direktorius arba viršininkas. Jis sprendžia šios įstaigos kompetencijai priskirtus klausimus, taip pat vykdo kitas įstatymų ir Vyriausybės įstatymų numatytas funkcijas. Įstaigos prie ministerijos veiklą reglamentuoja nuostatai, kuriuos tvirtina Vyriausybė. Įstaigų prie ministerijų vadovus skiria ir iš pareigų atleidžia ministras, nors yra ir išimčių (pvz., Policijos departamento prie VRM generalinį komisarą vidaus reikalų ministro siūlymu ir Vyriausybės teikimu į pareigas skiria Respublikos Prezidentas). Vyriausybė turi teisę panaikinti įstaigų prie ministerijų vadovų teisės aktus, jeigu jie prieštarauja Konstitucijai, įstatymams, Prezidento dekretams, Vyriausybės nutarimams, Ministro Pirmininko potvarkiams, tarptautinėms sutartims. Štai prie Vidaus reikalų ministerijos yra įsteigtos tokios įstaigos: Policijos departamentas, Valstybės sienos apsaugos tarnyba, Priešgaisrinės apsaugos ir gelbėjimo darbų departamentas, Finansinių nusikaltimų tyrimo departamentas. Prie kitų ministerijų yra įsteigta tokių įstaigų: Civilinės saugos departamentas prie Krašto apsaugos ministerijos, Muitinės departamentas prie Finansų ministerijos, Valstybinė mokesčių inspekcija prie Finansų ministerijos, Valstybinė ne maisto produktų inspekcija prie Ūkio ministerijos, Geologijos tarnyba prie Aplinkos ministerijos, Lietuvos darbo birža prie Socialinės apsaugos ir darbo ministerijos, Valstybinė darbo inspekcija prie Socialinės apsaugos ir darbo ministerijos, Nacionalinė vartotojų apsaugos taryba prie Teisingumo ministerijos ir kt.
4.7. APSKRITIES VALDYMAS
Apskritis yra aukštesnysis administracinis teritorinis vienetas. Lietuvos Respublikoje yra 10 apskričių: Vilniaus, Kauno, Klaipėdos, Šiaulių, Panevėžio, Alytaus, Tauragės, Marijampolės, Telšių ir Utenos. Apskrities valdymą per apskrities viršininką, ministerijas bei kitas Vyriausybės institucijas organizuoja Vyriausybė. Apskrities viršininką ir jo pavaduotoją (pavaduotojus) skiria ir atleidžia Vyriausybė Ministro Pirmininko teikimu. Taigi apskrities valdymas yra sudėtinė viešojo valdymo dalis. Apskrities valdymo organizavimą reguliuoja Apskrities valdymo įstatymas. Apskrities viršininkas, vykdydamas jam paskirtas funkcijas, bendradarbiauja su savivaldybių ir valstybės valdymo institucijomis, joms pavaldžiomis valdymo įstaigomis bei organizacijomis apskrityje. Apskrities viršininkas praneša atitinkamai Seimui ar Vyriausybei apie valstybės valdymo institucijų padalinių apskrityje veiklą ir sprendimus, jei jie neatitinka įstatymų, Vyriausybės nutarimų arba pažeidžia piliečių ir organizacijų teises. Jeigu ginčijamo sprendimo nepanaikina, nepakeičia jį priėmęs valstybės institucijos padalinys ar aukštesnioji valstybės valdymo institucija, galutinį sprendimą priima Vyriausybė. Pagal Apskrities valdymo įstatymą pagrindiniai apskrities viršininko uždaviniai yra: 1) įgyvendinti valstybės politiką regioninės plėtros, socialinio aprūpinimo, švietimo, kultūros, sveikatos priežiūros, teritorijų planavimo, paminklotvarkos, žemės naudojimo ir apsaugos, aplinkos apsaugos ir kitose srityse, vykdyti apskrityje valstybines ir tarpregionines programas; 2) koordinuoti apskrityje esančių ministerijų ir kitų Vyriausybės institucijų struktūrinių padalinių veiklą, taip pat derinti savivaldybių vyk
153
ADMINISTRACINE TEISE
domųjų institucijų veiklą įgyvendinant regionines programas; 3) numatyti prioritetines apskrities raidos kryptis ir rengti programas. Apskrities viršininkas jam priskirtoms funkcijoms įgyvendinti formuoja administraciją ir jai vadovauja. Apskrities viršininko administracija yra juridinis asmuo. Administracijos struktūrą ir jos nuostatus tvirtina apskrities viršininkas pagal vyriausybės patvirtintą tipinę struktūrą ir jos tipinius nuostatus. Administracijos struktūrą sudaro departamentai (socialinių reikalų ir švietimo, regioninės plėtros, teisės, žemės tvarkymo, kaimo reikalų ir kt.), skyriai, tarnybos (pvz., apskrities gydytojo), administracijos sekretorius. Apskrities viršininkas organizuoja administracijos darbą ir atsako už jos veiklą; teikia Vyriausybei apskrities lėšų sąmatos projektą; įstatymų ir kitų teisės aktų nurodyta tvarka skiria ir atleidžia iš pareigų jam pavaldžių įstaigų vadovus, administracijos departamentų, skyrių ir padalinių vadovus, sudaro su šiais darbuotojais darbo sutartis; tvirtina administracijos etatų sąrašą; šaukia regionų plėtros tarybos pirmąjį posėdį ir vadovauja tarybos darbui, kol taryba išsirinks pirmininką ir pasitvirtins savo nuostatus; vykdo kitus įstatymus, Vyriausybės ir Ministro Pirmininko jam suteiktus įgaliojimus. Kai apskrities viršininko nėra, jo pareigas eina apskrities viršininko pavaduotojas. Apskrities viršininkas leidžia įsakymus ir kitus teisės aktus ir tikrina, kaip jie vykdomi. Apskrities viršininko įsakymai ir kiti teisės aktai gali būti naikinami įstatymų nustatyta tvarka.
4.8. SAVIVALDYBĖS VALDYMO SUBJEKTAI
Pagal Lietuvos Respublikos vietos savivaldos įstatymą savivaldybės valdymo (administravimo) subjektai yra savivaldybės institucijos ir joms pavaldžios įstaigos bei kiti subjektai, turintys teisės aktų suteiktus įgaliojimus, atliekantys jiems pavestas valdymo funkcijas ir atsakingi už šių funkcijų įgyvendinimą. Savivaldybės institucijos yra atstovaujamoji institucija -savivaldybės taryba ir vykdomoji institucija – savivaldybės administracijos direktorius, turinčios vietos valdžios ir viešojo valdymo (administravimo) teises ir pareigas. Viešojo valdymo funkcijas įstatymų nustatyta tvarka atlieka savivaldybės taryba, savivaldybės administracijos direktorius, kiti savivaldybės įstaigų ir tarnybų vadovai, valstybės tarnautojai, kuriems teisės aktai ar savivaldybės tarybos sprendimai suteikia viešojo valdymo teises. Savivaldybės taryba susideda iš įstatymų nustatyta tvarka demokratiškai išrinktų savivaldybės bendruomenės atstovų. Savivaldybės taryba savo įgaliojimus įgyvendina kolegialiai posėdžiuose. Tarybos posėdis teisėtas, jeigu jame dalyvauja dauguma tarybos narių. Svarstomais klausimais taryba priima sprendimus ir kontroliuoja jų įgyvendinimą. Sprendimai priimami visų tarybos narių, dalyvaujančių posėdyje, balsų dauguma. Sprendimus pasirašo meras arba jo pavaduotojas, pirmininkavęs posėdžiui. Tarybos posėdžiai šaukiami prireikus, bet ne rečiau kaip kas trys mėnesiai. Posėdžiams pirmininkauja meras, o kai jo nėra – mero pavaduotojas. Tarybos posėdžiai protokoluojami. Protokolus pasirašo meras ar jo pavaduotojas, pirmininkavęs posėdžiui. Savivaldybės tarybos įgaliojimai yra labai platūs -jie apibrėžti Vietos savivaldos įstatymo 17 straipsnyje. Taigi savivaldybės taryba: tvirtina savo veiklos reglamentą; renka ir iš pareigų atleidžia merą, kuris yra tarybos pirmininkas (jo įgaliojimai
154
Viešojo valdymo subjektai
įtvirtinti įstatymo 20-21 str.); mero teikimu skiria mero pavaduotoją/pavaduotojus; priima sprendimą sudaryti savivaldybės tarybos kolegiją; sudaro savivaldybėje tarybos komitetus, komisijas ir kitas struktūras; renka Kontrolės komiteto pirmininką; priima sprendimą dėl savivaldybės kontrolieriaus priėmimo ir atleidimo; priima sprendimą dėl savivaldybės administracijos direktoriaus priėmimo į pareigas; tvirtina savivaldybės administracijos struktūrą; tvirtina savivaldybės biudžetą ir jo įvykdymo apyskaitą; tvirtina želdinių apsaugos, miesto ir kitų gyvenamųjų vietovių tvarkymo, sveikatos, sanitarijos ir higienos, aplinkos apsaugos, prekybos turgavietėse taisykles ir kt. Savivaldybės taryba savo įgaliojimų laikui iš savivaldybės tarybos narių gali sudaryti savivaldybės tarybos kolegiją ir pavesti jai vykdyti tam tikrus Vietos savivaldos įstatyme nustatytus savivaldybės tarybos įgaliojimus. Kolegijos nariai pagal pareigas yra meras ir mero pavaduotojas/pavaduotojai. Kitų kolegijos narių kandidatūras tarybai tvirtinti teikia meras. Kolegijos darbo tvarką, posėdžių organizavimo tvarką, kolegijos narių statusą nustato savivaldybės tarybos veiklos reglamentas. Savivaldybės kontrolierius – tai savivaldybės institucija, kuri vykdo savivaldybės kontrolės funkciją. Kontrolierius kontroliuoja, kaip vykdomas savivaldybės biudžetas ir naudojamas jos turtas. Jis atskaitingas savivaldybės tarybai ir jo veikla nesusijusi su savivaldybės tarybos įgaliojimų pabaiga. Savivaldybės administracija – tai savivaldybės įstaiga, į kurios sudėtį įeina struktūriniai padaliniai – departamentai, skyriai, tarnybos ir kt. ir struktūriniai teritoriniai padaliniai – seniūnijos. Savivaldybės administracijos struktūrą tvirtina ir jos direktorių į pareigas viešo konkurso būdu 5 metams skiria savivaldybės taryba. Savivaldybės administracijos direktorius yra pavaldus tarybai ir atsakingas merui. Savivaldybės administracija organizuoja ir kontroliuoja savivaldybės institucijų sprendimų įgyvendinimą arba pati juos įgyvendina; įgyvendina įstatymus ir Vyriausybės nutarimus, nereikalaujančius savivaldybės institucijų sprendimų; organizuoja savivaldybės biudžeto pajamų ir išlaidų bei kitų piniginių išteklių buhalterinės apskaitos tvarkymą, organizuoja ir kontroliuoja savivaldybės turto valdymą ir naudojimą; rengia savivaldybės institucijų sprendimų ir potvarkių projektus bei vykdo kitas įstatyme nurodytas funkcijas. Seniūnija yra savivaldybės administracijos struktūrinis teritorinis padalinys, veikiantis tam tikroje savivaldybės teritorijos dalyje. Seniūnijos aptarnaujamos teritorijos ribas ir seniūnijai perduodamas savivaldybės administracijos funkcijas savo sprendimu nustato savivaldybės taryba. Seniūnijai vadovauja seniūnas, kurį konkurso būdu skiria ir atleidžia savivaldybės administracijos direktorius vadovaudamasis Valstybės tarnybos įstatymu. Seniūnijos veiklą reglamentuoja vietos savivaldos įstatymas ir savivaldybės mero patvirtinti seniūnijos veiklos nuostatai. Seniūnijos ir seniūno funkcijos yra tokios: tvarko namų ūkio knygas kaimo vietovėje; savivaldybės administratoriui teikia duomenis, reikalingus mokyklinio amžiaus vaikų apskaitai; teikia savivaldybės administratoriui duomenis apie šaukiamojo amžiaus jaunuolius, gyvenančius seniūnijos teritorijoje; dalyvauja organizuojant civilinę saugą; organizuoja bendrojo naudojimo teritorijų, gatvių, šaligatvių valymą ir priežiūrą bei gatvių ir kitų viešųjų vietų apšvietimą; organizuoja želdinių ir kapinių priežiūrą; išduoda gyventojams pažymas apie šeimos sudėtį, gyvenamąją vietą; iš
155
ADMINISTRACINE TEISE
duoda leidimus laidoti; kaimo gyvenamųjų vietovių seniūnijose ir miestuose, kuriuose nėra civilinės metrikacijos įstaigų, registruoja mirties faktus; Notariato įstatymo nustatyta tvarka atlieka seniūnijai priskirtos teritorijos gyventojams notarinius veiksmus ir Civilinio kodekso nustatyta tvarka tvirtina gyventojų oficialiems testamentams prilyginamus testamentus; nagrinėja administracinių teisės pažeidimų bylas; vykdo kitas įstatymo numatytas funkcijas. Savivaldybės valdymo subjektai nėra pavaldūs valstybės institucijoms. Savivaldybės dėl jų teisių pažeidimo gali kreiptis į teismą. Savivaldybės tarybos sprendimai ir mero potvarkiai, neviršijantys šių institucijų kompetencijos, privalomi savivaldybės administracijai, visoms savivaldybės teritorijoje esančioms įstaigoms, įmonėms, organizacijoms ir gyventojams. Kaip savivaldybės laikosi Konstitucijos ir įstatymų, kaip vykdo Vyriausybės nutarimus, prižiūri Vyriausybės skiriami pareigūnai – Vyriausybės atstovai apskrityse.
5. VALSTYBĖS TARNYBA
Valstybės tarnyba yra vienas iš svarbiausių administracinės teisės institutų. Tai sudėtingas, kompleksinis teisės institutas, kurį sudaro ne tik administracinės teisės, bet ir kitų teisės sistemos rūšių (šakų) – konstitucinės, darbo, socialinės apsaugos, civilinės teisės – normų reguliuojami visuomeniniai santykiai. Valstybės tarnybos sąvoką, principus, valstybės tarnautojo statusą, atsakomybę, darbo užmokestį, socialines ir kitas garantijas, tarnybos valdymo teisinius pagrindus nustato Lietuvos Respublikos valstybės tarnybos įstatymas. Valstybės tarnybos sąvoka minėto įstatymo 2 straipsnyje apibūdinama gana plačiai. Glaustai valstybės tarnybą galima apibūdinti kaip visumą teisinių santykių, kurie atsiranda įgijus valstybės tarnautojo statusą valstybės ar savivaldybės institucijoje ar įstaigoje ryšium su viešojo valdymo veiklos vykdymu. Valstybės tarnyba grindžiama įstatymų viršenybės, lygiateisiškumo, lojalumo, politinio neutralumo, skaidrumo, atsakomybės už priimtus sprendimus, karjeros, taip pat valstybės tarnautojų veiklos etikos principais. Valstybės tarnybos įstatymas be išlygų taikomas valstybės tarnautojams, išskyrus statutinius valstybės tarnautojus, kurių tarnybą reglamentuoja įstatymo patvirtintas statutas (pvz., VRM vidaus tarnybos statutas) arba Diplomatinės tarnybos įstatymas, nustatantys specialias priėmimo į tarnybą, jos atlikimo, atsakomybės ir kitas su tarnybos ypatumais susijusias sąlygas. Statutiniams valstybės tarnautojams Valstybės tarnybos įstatymo nuostatos taikomos tiek, kiek jų statuso nereglamentuoja statutai ar Diplomatinės tarnybos įstatymas, išskyrus nustatytą darbo apmokėjimo tvarką. Valstybės tarnybos įstatymas netaikomas: 1) valstybės politikams (asmenims, įstatymų nustatyta tvarka išrinktiems ar paskirtiems į Respublikos Prezidento, Seimo Pirmininko, Seimo nario, Ministro Pirmininko, ministro, savivaldybės tarybos nario, savivaldybės mero, savivaldybės mero pavaduotojo pareigas); 2) Lietuvos Respublikos Konstitucinio Teismo, Lietuvos Aukščiausiojo Teismo, Lietuvos Vyriausiojo administracinio teismo ir kitų teismų teisėjams, prokurorams; 3) Lietuvos banko valdybos pirmininkui, jo pavaduotojams, valdybos nariams ir kitiems
156
Valstybės tarnyba
Lietuvos banko darbuotojams; 4) Seimo ar Respublikos Prezidento paskirtiems valstybės institucijų ir įstaigų vadovams, kitiems Seimo ar Respublikos Prezidento paskirtiems valstybės pareigūnams, išskyrus kai jie privalo atlyginti žalą, pažeidę Lietuvos Respublikos įstatymus (Valstybės tarnybos įstatymo 33 str. 3 d.). Valstybės saugumo departamento generaliniam direktoriui, jo pavaduotojams bei Specialiųjų tyrimų tarnybos direktoriui ir jo pavaduotojams taikomos Valstybės tarnybos įstatymo VI skirsnio nuostatos (Darbo užmokestis); 5) Seimo ar Respublikos Prezidento paskirtiems valstybinių (nuolatinių) komisijų ir tarybų pirmininkams, jų pavaduotojams ir nariams, taip pat pagal specialius įstatymus įsteigtų komisijų, tarybų, fondų valdybų pirmininkams ir nariams, išskyrus kai jie privalo atlyginti žalą, pažeidę Lietuvos Respublikos įstatymus (Valstybės tarnybos įstatymo 33 str. 3 d.); 6) profesinės karo tarnybos kariams; 7) valstybės ir savivaldybių įmonių darbuotojams; 8) viešųjų įstaigų darbuotojams; 9) darbuotojams, dirbantiems pagal darbo sutartis ir gaunantiems darbo užmokestį iš valstybės ir savivaldybių biudžetų ar valstybės pinigų fondų. Darbo santykius ir socialines garantijas reglamentuojantys įstatymai bei kiti teisės aktai valstybės tarnautojams taikomi tiek, kiek jų statuso ir socialinių garantijų nereglamentuoja Valstybės tarnybos įstatymas. Valstybės ir savivaldybių institucijose ir įstaigose yra nustatytos įvairios pareigybės, nuo kurių priklauso valstybės tarnautojo teisinė padėtis. Valstybės tarnautojai, be jiems būdingų bendrųjų požymių, skiriasi ne tik vykdomomis funkcijomis, bet ir užimama vieta valstybės ar savivaldybės institucijose, įstaigose bei vaidmeniu įgyvendinant valstybės tarnybos uždavinius. Pagal Valstybės tarnybos įstatymą (6 str.) valstybės tarnautojų pareigybės skirstomos į: 1) karjeros; 2) politinio (asmeninio) pasitikėjimo; 3) įstaigų vadovų; 4) pakaitinių. Karjeros valstybės tarnautojas priimamas į pareigas viešo konkurso būdu neterminuotam laikui ir turi galimybę įstatymo nustatyta tvarka įgyvendinti teisę į karjerą valstybės tarnyboje. Politinio (asmeninio) pasitikėjimo tarnautojas priimamas į pareigas jį priėmusio valstybės politiko ar kolegialios valstybės institucijos įgaliojimo laikui. Įstaigos vadovas yra valstybės tarnautojas, konkurso būdu ar politinio (asmeninio) pasitikėjimo pagrindu priimtas vadovauti valstybės ar savivaldybės institucijai ar įstaigai. Pakaitinis valstybės tarnautojas – tai tarnautojas, pakeičiantis (be konkurso) laikinai negalintį eiti pareigų karjeros valstybės tarnautoją. Valstybės tarnautojų pareigybės yra trijų lygių: 1) A lygio – pareigybės, kurioms būtinas aukštasis universitetinis arba jam prilygintas išsilavinimas; 2) B lygio -pareigybės, kurioms būtinas ne žemesnis kaip aukštasis neuniversitetinis arba jam prilygintas išsilavinimas; 3) C lygio – pareigybės, kurioms būtinas ne žemesnis kaip vidurinis išsilavinimas ir įgyta profesinė kvalifikacija. Valstybės tarnautojų pareigybės skirstomos į 20 kategorijų. Aukščiausia yra 20-oji kategorija, žemiausia – 1-oji kategorija. Valstybės tarnautojų pareigybės nustatomos Vyriausybės patvirtinta jų aprašymo ir vertinimo metodika. Pareigybės aprašyme nurodoma pareigybės lygis, kategorija, specialūs reikalavimai, keliami tam tikras pareigas einančiam valstybės tarnautojui, pareigybei priskirtos funkcijos. Nuo valstybės tarnautojo pareigybės kategorijos priklauso algos koeficientas ir darbo užmokestis.
157
ADMINISTRACINE TEISE
Valstybės tarnybos įstatyme nustatyti asmenų priėmimo į valstybės tarnybą bendrieji reikalavimai. Asmuo turi būti: 1) Lietuvos Respublikos pilietis; 2) mokėti lietuvių kalbą; 3) būti ne jaunesnis kaip 18 metų ir ne vyresnis kaip 62 metų ir 6 mėnesių (reikalavimas būti ne vyresniam kaip 62 metų ir 6 mėnesių netaikomas politinio (asmeninio) pasitikėjimo ir pakaitiniams valstybės tarnautojams). Į valstybės tarnautojo pareigas negali būti priimtas asmuo: 1) įstatymų nustatyta tvarka pripažintas kaltu dėl sunkaus nusikaltimo ar nusikaltimo valstybės tarnybai padarymo ir turintis neišnykusį ar nepanaikintą teistumą; 2) kurio teisę eiti valstybės tarnautojo pareigas yra atėmęs teismas; 3) kurio sutuoktinis, artimas giminaitis ar svainystės ryšiais susijęs asmuo eina valstybės tarnautojo pareigas valstybės ar savivaldybės institucijoje ar įstaigoje, jeigu jie pagal einamas pareigas būtų susiję tiesioginio pavaldumo santykiais; 4) kuris įstatymų nustatyta tvarka pripažintas neveiksniu; 5) kuris yra įstatymų nustatyta tvarka uždraustos organizacijos narys; 6) kitais įstatymų nustatytais atvejais. Valstybės tarnybos įstatyme įtvirtintos valstybės tarnautojų teisės ir pareigos. Yra bendros visiems tarnautojams teisės ir pareigos ir specialiosios teisės ir pareigos, atsižvelgiant į tai, kokioje institucijoje ar įstaigoje tarnautojas dirba. Valstybės tarnybos įstatyme nustatytos valstybės tarnautojų karjeros galimybės, tarnybinės veiklos vertinimo pagrindai, kokia veikla nesuderinama su valstybės tarnautojų pareigomis, jų kvalifikacijos kėlimo organizavimas, socialinės ir kitos garantijos. Labai svarbios Įstatymo nuostatos, numatančios valstybės tarnautojų skatinimą ir nustatančios atsakomybę. Už nepriekaištingą tarnybinių pareigų atlikimą valstybės tarnautojai gali būti skatinami: 1) padėka; 2) vardine dovana; 3) vienkartine pinigine išmoka. Už ypatingus nuopelnus valstybės tarnybai tarnautojai gali būti teikiami valstybės apdovanojimui gauti. Apie valstybės tarnautojo gautus paskatinimus ir valstybės apdovanojimus įrašoma į valstybės tarnautojo asmens bylą. Už tarnybinius nusižengimus valstybės tarnautojai traukiami tarnybinėn (drausminėn) atsakomybėn. Už valstybės ir savivaldybės institucijai ir įstaigai padarytą materialinę žalą valstybės tarnautojai traukiami materialinėn atsakomybėn. Už tarnybinius nusižengimus valstybės tarnautojui gali būti skiriama viena iš šių tarnybinių nuobaudų: 1) pastaba; 2) papeikimas; 3) griežtas papeikimas; 4) atleidimas iš pareigų. Tarnybinė nuobauda – atleidimas iš pareigų gali būti skiriama tik už šiurkščius nusižengimus, kurie numatyti Valstybės tarnybos įstatyme (29 str.). Tarnybinė nuobauda turi būti paskirta ne vėliau kaip per vieną mėnesį nuo tarnybinio nusižengimo paaiškėjimo dienos. Negalima skirti tarnybinės nuobaudos, jeigu praėjo 6 mėnesiai nuo nusižengimo padarymo dienos arba nuo tęstinio (trunkamojo) nusižengimo paaiškėjimo dienos. Į nustatytus terminus neįskaitomas laikas, kurį valstybės tarnautojas nėjo pareigų dėl ligos ar atostogų. Už vieną tarnybinį nusižengimą galima skirti tik vieną tarnybinę nuobaudą. Kai paaiškėja, kad tarnybinis nusižengimas turi baudžiamosios veikos ar administracinio teisės pažeidimo požymių, tarnybinio nuobaudų skyrimo procedūra sustabdoma ir tarnybinio patikrinimo medžiaga perduodama institucijai, kompetentingai tirti atitinkamas bylas. Jeigu atsisakoma iškelti baudžiamąją ar administracinę bylą arba asmuo atleidžiamas nuo baudžiamosios ar administracinės atsakomybės, tarnybinės nuobaudos skyrimo procedūra tęsiama ir tarnybinė nuobauda turi būti paskirta ne vėliau kaip per vieną mėnesį nuo šių sąlygų atsiradimo, jei po atitinkamą bylą tirti kompetentingos institucijos
158
Valdymo aktai
sprendimo priėmimo nepraėjo daugiau kaip vieneri metai. Jei daugiau kaip vienerių metų terminas praėjo, tarnybinės nuobaudos skyrimo procedūra nutraukiama. Tarnybines nuobaudas skiria valstybės tarnautoją į pareigas priėmęs asmuo. Tarnybinių nuobaudų skyrimo valstybės tarnautojams tvarką nustato Vyriausybė. Sprendimas dėl tarnybinės nuobaudos paskyrimo gali būti skundžiamas Administracinių bylų teisenos įstatymo nustatymo tvarka. Valstybės tarnautojas laikomas nebaustu tarnybine nuobauda, kai po nuobaudos paskyrimo datos praėjo vieneri metai. Tarnybinė nuobauda gali būti panaikinta anksčiau, negu sueis vieneri metai, ją paskyrusio asmens motyvuotu sprendimu arba jei valstybės tarnautojas gauna valstybės apdovanojimą. Valstybės tarnautojui, kuris pasirodė tarnyboje neblaivus, apsvaigęs nuo narkotinių ar toksinių medžiagų, tiesioginis jo vadovas tą dieną gali neleisti dirbti (nušalinti nuo pareigų) ir sustabdyti darbo užmokesčio mokėjimą. Valstybės tarnybos bendrąjį valdymą atlieka Vyriausybė ir vidaus reikalų ministras. Vyriausybė įgyvendina valstybės tarnybos politiką ir vykdo teisės aktuose nustatytas šios tarnybos bendrojo valdymo funkcijas. Vidaus reikalų ministras teikia Vyriausybei su valstybės tarnyba susijusių teisės aktų projektus, koordinuoja teisės aktų įgyvendinimo kontrolę, atlieka valstybės tarnybos bendrojo valdymo funkcijas.
6. VALDYMO AKTAI
6.1. VALDYMO AKTŲ SAMPRATA IR JURIDINĖ REIKŠMĖ
Teisinėje literatūroje teisės aktas (lot. actus – veiksmas, poelgis) suprantamas kaip juridinis dokumentas, kuriuo siekiama tam tikrų teisinių pasekmių. Viešojo valdymo institucijų (pareigūnų) valdymo aktai yra savarankiška teisinių aktų rūšis. Šiais aktais viešojo valdymo subjektai, siekdami tam tikrų juridinių rezultatų, teisiškai įtvirtina savo sprendimus. Valdymo aktais įtvirtinamos teisės normos, kurios reguliuoja visuomeninius santykius viešojo valdymo srityje, nustatomos tam tikros elgesio taisyklės, valdymo santykių šalių teisės, pareigos ir atsakomybė. Valdymo aktus leidžia viešojo valdymo subjektai, vykdydami įstatymus ir remdamiesi įstatymais, todėl šie aktai yra poįstatyminiai. Svarbiausia juridinė valdymo aktų reikšmė yra ta, kad jais sprendžiami viešojo valdymo uždaviniai. Valdymo aktais įtvirtinama viešojo valdymo subjektų kompetencija, jų veiklos formos, todėl jų priėmimas teisinėje literatūroje vadinamas pagrindine valdymo forma. Viešojo valdymo subjektai valdymo aktais vienašališkai išreiškia savo valią, kuri yra besąlygiškai privaloma subjektams, kuriems jie skirti. Valdymo aktas gali būti kaip juridinis faktas, t. y. kaip tam tikra aplinkybė, kurios pagrindu atsiranda, keičiasi ar pasibaigia teisiniai santykiai. Dažnai valdymo aktas gali būti kitų valdymo aktų priėmimo pagrindas. Pavyzdžiui, remiantis Vyriausybės nutarimu, ministerijos, Vyriausybės įstaigos ir kitos institucijos priima atitinkamus valdymo aktus. Valdymo aktas gali būti kaip tam tikras įrodymas teismui nagrinėjant civilinę, administracinę ar baudžiamąją bylą. Valdymo aktai turi juridinę reikšmę ir tais atvejais, kai jie gali būti civilinių teisinių sandorių sudarymo ir galiojimo sąlyga,
159
ADMINISTRACINĖ TEISĖ
taip pat pagrindas civiliniams teisiniams, darbo, finansų, žemės teisiniams santykiams atsirasti, pasikeisti ar pasibaigti. Valdymo aktai įgyvendinami remiantis įtikinimu ir valstybinės prievartos priemonėmis. Taigi apibendrinus galima teigti, kad: 1. Valdymo aktai yra pagrindinė teisinė viešojo valdymo subjektų veiklos forma. 2. Valdymo aktai – tai poįstatyminiai aktai, kurie priimami vykdant įstatymus ir remiantis įstatymais. 3. Valdymo aktai yra vienašališko pobūdžio teisiniai sprendimai, kurie privalomi atitinkamiems teisės subjektams. 4. Valdymo aktų pagrindu atsiranda, pasikeičia arba pasibaigia teisiniai santykiai. 5. Valdymo aktai įgyvendinami remiantis įtikinimu ir valstybės prievartos priemonėmis.
6.2. VALDYMO AKTŲ SKIRTUMAS NUO KITŲ TEISINIŲ AKTŲ
Valdymo aktas skiriasi nuo įstatymo. Įstatymas yra aukščiausia juridinė valstybės valdžios valios pasireiškimo forma ir visų kitų teisinių aktų šaltinis. Valdymo aktas galioja tik tada, kai yra išleistas pagal įstatymą ir vykdant jį. Nuo įstatymo priklauso valdymo akto turinys ir juridinė galia. Valdymo aktai skiriasi nuo teismo procesinių aktų. Teismas norminių aktų nepriima, elgesio taisyklių nenustato. Teismo aktai priimami remiantis civilinio, baudžiamojo ar administracinio proceso normomis bylose, kurių nagrinėjimas priklauso tik teismo kompetencijai. Teismas įstatymų nustatyta procesine tvarka byloje priima sprendimą, nuosprendį, nutarimą arba nutartį nesikišant jokioms kitoms institucijoms ar pareigūnams. Tuo tarpu viešojo valdymo subjektas priima norminius ir nenorminius aktus ne tik remdamasis įstatymais, bet ir vykdydamas tiesioginius aukštesniųjų institucijų, jų pareigūnų nurodymus. Valdymo aktas skiriasi nuo civilinės sutarties – juridinio akto civiliniuose teisiniuose santykiuose. Civilinė sutartis yra sandoris, t. y. dviejų ar daugiau subjektų savitarpio susitarimo ir veikimo rezultatas. Valdymo aktų teisiniai santykiai paprastai nustatomi, pakeičiami arba nutraukiami vienašališkai, be antrosios šio santykio šalies sutikimo. Valdymo aktai skiriasi ir nuo visuomeninių organizacijų aktų. Visuomeninių organizacijų aktai yra privalomi šioms organizacijoms ir jų nariams. Viešojo valdymo subjektų aktai gali būti privalomi visiems adresatams, t. y. valstybinėms ir savivaldybių institucijoms, įstaigoms, įmonėms, jų pareigūnams, visuomeninėms organizacijoms ir piliečiams. Galiausiai valdymo aktai skiriasi nuo asmens ir tarnybinių dokumentų (asmens paso, asmens liudijimo, diplomo, gimimo liudijimo, revizijos akto ir kt.). Minėti dokumentai teisės normų nenustato, nesukuria, nepakeičia ir nenutraukia administracinių teisinių santykių. Dokumentų paskirtis – paliudyti asmens tapatybę, patvirtinti tam tikrą faktą. Tam tikrais atvejais tarnybiniai dokumentai (pvz., revizijos aktas) gali būti pagrindas valdymo aktui priimti. Pavyzdžiui, remdamasis revizijos aktu organizacijos vadovas priima įsakymą, kuriame nustatoma darbo tvarka, turto apsaugos taisyklės ir t. t.
160
Valdymo aktai 6.3. VALDYMO AKTŲ KLASIFIKACIJA
Viešojo valdymo subjektų valdymo aktai labai įvairūs ir jie turi specifinių, tik jiems būdingų požymių. Todėl šie aktai gali būti klasifikuojami pagal įvairius kriterijus. Specialiojoje teisinėje literatūroje valdymo aktai dažniausiai klasifikuojami pagal: 1) juridines savybes; 2) galiojimą erdvėje; 3) juos priimančių subjektų kompetencijos pobūdį; 4) juos priimančius subjektus; 5) jų išorinės išraiškos formą. Pagal juridines savybes valdymo aktai skirstomi į norminius ir individualius (administracinius, teisės taikymo) aktus. Norminiame valdymo akte nustatomos elgesio taisyklės individualiais bruožais neapibūdintų subjektų grupei. Vienas iš svarbiausių norminio akto požymių yra tas, kad jame nenurodomas konkrečiai apibrėžtas administracinių teisinių santykių subjektas, o tik bendrais bruožais nusakomi tam tikros kategorijos subjektai (apskritai piliečiai, pareigūnai, organizacijos ir pan.). Individualus teisės aktas – tai vienkartinis teisės taikymo aktas, skirtas konkrečiam subjektui ar individualiais bruožais apibūdintų subjektų grupei. Individualus aktas priimamas tada, kai reikia taikyti tam tikras norminiame akte nustatytas teisės normas. Pavyzdžiui, remdamasis Švietimo ir mokslo ministerijos nuostatais (norminiu aktu) ministras priima įsakymą (individualų aktą) įsteigti N vidurinę mokyklą. Pagal galiojimą erdvėje valdymo aktai skirstomi į centrinių ir teritorinių valdymo subjektų aktus. Pavyzdžiui, Vyriausybės aktai galioja visoje šalies teritorijoje, o teritorinių valdymo subjektų aktai – tik tam tikrame administraciniame teritoriniame vienete (apskrityje). Pagal viešojo valdymo subjektų kompetencijos pobūdį valdymo aktai skirstomi į bendrosios ir šakinės kompetencijos institucijų aktus. Bendrosios kompetencijos subjektų aktams priklauso Vyriausybės aktai, kuriais reguliuojami visuomeniniai santykiai įvairiose ūkinės, socialinės kultūrinės ir administracinės politinės veiklos srityse. Šakinės kompetencijos institucijų aktai yra ministerijų aktai. Pagal aktus priimančius valdymo subjektus aktai skirstomi į: 1) Vyriausybės nutarimus ir Ministro Pirmininko potvarkius, jo pavedimus, įformintus rezoliucijomis; 2) ministrų įsakymus; 3) Vyriausybės įstaigų vadovų įsakymus; 4) apskričių viršininkų įsakymus; 5) centrinių valdymo institucijų įstaigų, esančių administraciniuose teritoriniuose vienetuose, vadovų įsakymus; 6) savivaldybių administracijos vadovų įsakymus; 7) įstaigų, įmonių, organizacijų vadovų įsakymus. Pagal išorinės išraiškos formą valdymo aktai gali būti rašytiniai ir žodiniai. Paprastai daugkartiniam taikymui skirti aktai yra rašytiniai. Jeigu įsakymu nustatyta, kad akto forma privalo būti rašytinė, tai to nesilaikant aktas neturės teisinės galios. Operatyvioje veikloje valdymo subjektai dažnai vartoja žodinę aktų formą. Pavyzdžiui, institucijų, įstaigų vadovai dažnai žodžiu duoda tam tikrus įsakymus, nurodymus, kurie yra privalomi.
161
ADMINISTRACINE TEISE
6.4. VALDYMO AKTŲ RENGIMO IR PRIĖMIMO TVARKA
Valdymo aktų rengimo ir priėmimo tvarką reglamentuoja įstatymai ir kiti teisės aktai. Lietuvos Respublikos įstatymų ir kitų teisės norminių aktų rengimo tvarkos įstatymas nustato valdymo aktų rengimo tvarką, stadijas, aktų formas, struktūrą, turinio ir kalbos reikalavimus. Valdymo aktų rengimo ir priėmimo tvarką taip pat reglamentuoja Vyriausybės darbo reglamentas, ministerijų, Vyriausybės įstaigų, savivaldybių tarybų darbo reglamentai ir Teisingumo ministro įsakymas dėl valdymo aktų rengimo rekomendacijų. Numatytos tokios valdymo akto rengimo stadijos: 1) valdymo akto (norminio) paskirties ir jo rengimo užduoties nustatymas; 2) valdymo akto projekto rengėjo nustatymas (parinkimas); 3) valdymo akto teksto parengimas; 4) valdymo akto derinimas ir aprobavimas. Pasiūlymus valdymo aktams rengti turi teisę teikti visi fiziniai ir juridiniai asmenys. Valdymo akto paskirtį ir jo rengimo užduotį turi teisę nustatyti valstybės ar savivaldybės institucijos pagal savo kompetenciją. Akto rengėjais gali būti institucijos paskirti ar konkurso tvarka parinkti asmenys ar jų grupė, taip pat asmuo arba asmenų iniciatyvinė grupė. Valdymo aktas turi būti parengtas pagal minėtame įstatyme (5-12 str.) įtvirtintus reikalavimus. Tai reiškia, kad valdymo aktas turi atitikti Konstituciją, įstatymus, Vyriausybės nutarimus, kitus teisės aktus. Įstatyme nustatyta, kad būtina laikytis rengiamo valdymo akto formos, t. y. akte turi būti: 1) institucijos, įgaliotos priimti aktą, pavadinimas; 2) akto rūšies pavadinimas (nutarimas, įsakymas, nuostatai); 3) konkretaus klausimo, kuriuo priimamas aktas, pavadinimas (pvz., Švietimo ir mokslo ministro įsakymas „Dėl bendrojo lavinimo mokyklų kompiuterizavimo”); 4) akto priėmimo data, numeris, vieta; 5) akto tekstas; 6) aktą pasirašančio asmens pareigos, vardas, pavardė; 7) akto teksto viršutinėje dešinėje pusėje turi būti parašyta „Projektas”. Būtina laikytis įstatyme nurodytos valdymo akto struktūros. Vyriausybės ir kitų valdymo institucijų teisės aktus sudaro punktai, jų pastraipos ir punktų papunkčiai. Punktai žymimi arabiškais skaitmenimis ir numeruojami iš eilės. Po skaitmens dedamas taškas. Punktų pastraipos nenumeruojamos. Punktų papunkčiai žymimi arabiškais skaitmenimis ir turi pradinį punkto ir savo eilės skaitmenį. Tarp punkto ir papunkčio skaitmens ir po papunkčio skaitmens dedami taškai. Atskirai numeruoti papunkčiai gali būti vadinami punktais. Viso teisės akto punktų numeracija yra ištisinė. Didesnės apimties valdymo aktas gali būti skirstomas į skyrius ir skirsnius. Skyriai ir skirsniai žymimi romėniškais skaitmenimis iš eilės. Įstatyme yra įtvirtinti tokie valdymo akto turinio reikalavimai: 1) akto turinys turi atitikti jo paskirtį; 2) akto turinys turi būti logiškas, glaustas ir aiškus; 3) akto tekste neturi būti nereikalingų arba netaisyklingų žodžių, žodžių junginių, pastabų ir dviprasmybių. Punkto tekstas nekartojamas kituose punktuose. Neleistini žodžių sutrumpinimai, išskyrus atvejus, kai pateikiama nuoroda į leidinį, kuriame šis aktas oficialiai paskelbtas. Pirmą kartą minimas pavadinimas negali būti trumpinamas. Valdymo aktai rašomi laikantis bendrinės lietuvių kalbos normų ir teisės terminijos. Tarptautiniai žodžiai vartojami tik tada, kai lietuvių kalboje nėra šių žodžių atitikmenų. Įstatyme nustatyti taip pat reikalavimai, kurių reikia laikytis keičiant ar papildant valdymo aktą. Keičiant valdymo aktą, jei punktas, jo dalis keičiamas iš esmės, turi būti išdėstoma pakitimo esmė ir pateikiama keičiamo punkto, jo dalies
162
Valdymo aktai
nauja redakcija. Jei valdymo aktas papildomas nauju punktu (jo dalimi), išdėstoma papildymo esmė ir pateikiama naujo punkto redakcija. Jei keičiamame akte keičiama ar buvo pakeista daugiau kaip pusė jo punktų, aktas paprastai pateikiamas naujos redakcijos arba parengiamas naujas teisės aktas. Kiti reikalavimai keičiant ar papildant valdymo aktus yra nustatyti nurodyto įstatymo 11 straipsnyje. Parengtas valdymo akto projektas derinamas, tvirtinamas ir pateikiamas institucijai, įgaliotai jį priimti. Derinamas ekonominius santykius reguliuojančio akto projektas turi būti vertinamas antikorupciniu požiūriu. Jeigu yra pagrindo manyti, kad priimtas aktas gali daryti įtaką kriminogeninei situacijai, jį priimti įgaliota institucija skiria kriminologinę ekspertizę. Valdymo aktų priėmimo tvarka nustatyta įstatymuose. Aktai priimami kolegialiai ir vienvaldiškai. Vyriausybės įstatymo 41 straipsnyje pasakyta, kad Vyriausybės nutarimai priimami posėdžiuose visų Vyriausybės narių balsų dauguma. Vyriausybės nutarimus per 3 dienas pasirašo Ministras Pirmininkas ir atitinkamos srities ministras. Ministras Pirmininkas, ministrai, Vyriausybės įstaigų ir kitų valdymo institucijų vadovai aktus priima vienvaldiškai ir juos pasirašo. Prireikus keli ministrai gali leisti bendrą įsakymą arba įsakymu patvirtintus kitus teisės aktus (instrukcijas, taisykles) ir priimtą įsakymą pasirašo.
6.5. VALDYMO AKTŲ PASKELBIMO IR ĮSIGALIOJIMO TVARKA
Valdymo aktų paskelbimo ir įsigaliojimo tvarka nustatyta 1993 m. balandžio 6 d. įstatyme „Dėl Lietuvos Respublikos įstatymų ir kitų teisės aktų skelbimo ir įsigaliojimo tvarkos”. Lietuvos Respublikos Vyriausybės nutarimai, kuriuose nustatomos, keičiamos ar pripažįstamos netekusiomis galios teisės normos, įsigalioja kitą dieną po to, kai jie, pasirašyti Ministro Pirmininko ir atitinkamo ministro, paskelbiami „Valstybės žiniose”, jeigu pačiuose nutarimuose nenustatyta vėlesnė jų įsigaliojimo data. Lietuvos Respublikos Vyriausybės nutarimai, kuriuose nėra nustatomos, keičiamos ar pripažįstamos netekusios galios teisės normos, taip pat Ministro Pirmininko potvarkiai įsigalioja jų pasirašymo dieną, jeigu pačiuose nutarimuose ir potvarkiuose nenustatyta vėlesnė jų įsigaliojimo data. Lietuvos banko, ministerijų, departamentų ir Vyriausybės įstaigų bei kitų valstybės valdymo institucijų teisės aktai, kuriuose nustatomos, keičiamos ar pripažįstamos netekusiomis galios teisės normos, įsigalioja kitą dieną po jų paskelbimo „Valstybės žiniose”, jeigu pačiuose teisės aktuose nenumatyta vėlesnė jų įsigaliojimo data. Aukštesniųjų administracinių vienetų valdymo institucijų, savivaldybių ir jų vykdomųjų institucijų priimti teisės aktai, kuriuose nustatomos, keičiamos ar pripažįstamos netekusiomis galios teisės normos, įsigalioja kitą dieną po jų paskelbimo vietinėje spaudoje, jeigu pačiuose teisės aktuose nenustatyta vėlesnė jų įsigaliojimo data. Teisės aktai, kuriuose yra valstybės ar tarnybinę paslaptį sudarančių žinių, „Valstybės žiniose”, interneto tinklalapiuose ir vietinėje spaudoje neskelbiami. Šie aktai turi būti išsiunčiami toms institucijoms, kurios įstatymų nustatyta tvarka gali disponuoti valstybės ar tarnybinę paslaptį sudarančiomis žiniomis.
163
7. ADMINISTRACINĖ ATSAKOMYBĖ
7.1. ADMINISTRACINĖS ATSAKOMYBĖS SAMPRATA
Administracinė atsakomybė yra savarankiška teisinės atsakomybės rūšis, todėl jai yra būdingi visi bendrieji teisinės atsakomybės bruožai. Administracinė atsakomybė – tai asmens pareiga atsakyti už padarytą administracinės teisės pažeidimą. Administracinės atsakomybės pagrindas – įstatymo nustatytas administracinis teisės pažeidimas. Administracinio teisės pažeidimo ir administracinės atsakomybės subjektas gali būti fizinis asmuo, kaltas padaręs pažeidimą. Traukiamam administracinėn atsakomybėn asmeniui taikoma administracinė nuobauda, kuria teisės pažeidėjui daromas tam tikras teisinis poveikis. Atlikdamas administracinę nuobaudą kaltas asmuo priverstas patirti jam nemalonių moralinio, turtinio, fizinio ir kitokio pobūdžio padarinių. Nubaustas asmuo netenka kai kurių materialinių vertybių, teisių, laisvės, darbo, patiria moralinio pobūdžio padarinių. Administracinėn atsakomybėn, skirtingai negu drausminėn (tarnybinėn), kalti asmenys traukiami ne tarnybinio, ne darbo santykių pavaldumo tvarka. Tai reiškia, kad administracinių teisės pažeidimų bylas nagrinėja ir kaltiesiems nuobaudas skiria ne įstaigų, įmonių, organizacijų, kuriose dirba pažeidėjas, vadovai, bet kitos atitinkamos valstybės valdymo institucijos, jų pareigūnai ar teismas (teismo teisėjas). Kalti padarę administracinį teisės pažeidimą asmenys traukiami administracinėn atsakomybėn Lietuvos Respublikos administracinių teisės pažeidimų kodekso normose nustatyta tvarka, kuri garantuoja objektyvų, visapusišką, teisingą bylos nagrinėjimą ir asmens teisių apsaugą. Administracinei atsakomybei būdinga tai, kad ji yra labai svarbi teisinė priemonė kovojant ne tik su administraciniais, bet ir kitais teisės pažeidimais. Ypač svarbus profilaktinis administracinės atsakomybės vaidmuo kovojant su piktybiniais viešosios tvarkos pažeidimais, vagystėmis, tarnybiniais piktnaudžiavimais, kelių eismo taisyklių, aplinkos apsaugos, priešgaisrinės apsaugos ir kitais pavojingais teisės pažeidimais. Svarbu ir tai, kad taikant administracinę atsakomybę siekiama ne tik kaltąjį asmenį nubausti, bet ir sudrausminti kitus, pasakyti, jog būtina laikytis įstatymų, užtikrinti, kad būtų vykdomi teisės reikalavimai. Reikia pabrėžti, kad administracinė atsakomybė už teisės pažeidimus nustatoma tik įstatymais.
7.2. ADMINISTRACINĖS ATSAKOMYBĖS PRINCIPAI
Teisinės atsakomybės tikslas bus pasiektas, ji bus veiksminga, kai ją taikantys subjektai laikysis pagrindinių jos principų. Teisėje atsakomybės principai suprantami kaip tam tikros vadovaujančios nuostatos, idėjos, kurios atspindi ir išreiškia pagrindinius teisės reikalavimus, jos esmę. 1. Atsakomybės teisėtumo principas. Tai reiškia, jog reikia tiksliai laikytis materialiųjų ir procesinių teisės normų ir jas vykdyti. Administracinių teisės pažeidimų kodekso 7 straipsnyje pasakyta: „Niekam negali būti taikoma poveikio priemonė už administracinį teisės pažeidimą kitaip, kaip įstatymų nustatytais pagrindais ir tvarka”.
164
Administracinė atsakomybė
2. Atsakomybės tik už priešingą teisei veikimą ar neveikimą ir tik esant kaltei principas. Tai reiškia, kad administracinėn atsakomybėn gali būti traukiamas asmuo, kuris padarė teisės draudžiamą veikimą (neveikimą) kaltai, t. y. suvokdamas pavojingus, žalingus visuomenei tokio savo elgesio padarinius, jų siekdamas arba sąmoningai leisdamas tokiems padariniams atsirasti. 3. Atsakomybės teisingumo ir tikslingumo principas. Jo esmė yra ta, kad administracinės atsakomybės priemonės (jų griežtumas) turi būti nustatytos ir parinktos teisingai ir tikslingai, atsižvelgiant į teisės pažeidimo pavojingumą, pažeidėjo kaltės laipsnį, atsakomybę švelninančias ir sunkinančias aplinkybes. Tuo atveju, kai taikyti administracinę atsakomybę netikslinga arba atsakomybės tikslas pasiektas anksčiau negu buvo numatyta taikant nuobaudą, atsakomybė gali būti netaikoma arba nuo jos asmuo gali būti atleistas anksčiau. ATPK 301 straipsnyje pasakyta: „Organas (pareigūnas), nagrinėjantis administracinių teisės pažeidimų bylas, atsižvelgdamas į aplinkybes, nurodytas šio kodekso 30 straipsnio 2 dalyje, taip pat į šio kodekso 31 straipsnyje nustatytas atsakomybę lengvinančias bei kitas įstatymų nenurodytas lengvinančias aplinkybes, vadovaudamasis teisingumo ir protingumo kriterijais, gali paskirti mažesnę nuobaudą nei sankcijoje numatyta minimali arba paskirti švelnesnę nuobaudą nei numatyta sankcijoje, arba visai neskirti administracinės nuobaudos”. Pagal ATPK 329 straipsnį, jei asmuo, kuriam atimta teisė vairuoti transporto priemonę arba teisė medžioti ar žvejoti, teisės atėmimo laikotarpiu nepadarė teisės pažeidimo, nuobaudą skyręs pareigūnas gali, praėjus ne mažiau kaip pusei paskirtojo laiko, tarpininkaujant visuomeninei organizacijai, darbdaviui, sutrumpinti nurodytos specialiosios teisės atėmimo terminą. 4. Atsakomybės neišvengiamumo principas. Jo esmė ta, kad kiekvienas administracinės teisės pažeidimas turi būti išaiškintas, kaltas asmuo patrauktas atsakomybėn, skirta nuobauda įvykdyta. Įspėjamoji administracinės nuobaudos reikšmė svarbi tuo, kad ji neišvengiama ir daro realų poveikį teisės pažeidėjui. Nebaudžiamumas, teisės pažeidimo nepasmerkimas paprastai skatina naujus pažeidimus, padeda formuotis antivisuomeniniams pažeidėjo polinkiams bei įpročiams. 5. Atsakomybės viešumo principas. Jis reikalauja, kad kalto asmens poelgis būtų pasmerktas viešai, atvirai, žinant visuomenei. Atsakomybės viešumas didina auklėjamąjį poveikį pažeidėjui ir kitiems asmenims, daro atsakomybės priemones veiksmingesnes. Viešumo principo reikalavimai gali būti įgyvendinami įvairiais būdais viešai, atvirai svarstant bylas, organizuojant išvažiuojamuosius bylų nagrinėjimo posėdžius įmonėse, įstaigose, gyvenvietėse, pranešant darbovietės, mokyklos administracijai ir kolektyvui apie jo nario bylos svarstymo rezultatus. 6. Atsakomybės operatyvumo principas. Tai reiškia, kad administracinių teisės pažeidimų bylos tiriamos, nagrinėjamos, nuobaudos pažeidėjui skiriamos ir vykdomos nedelsiant, per kuo trumpesnį laikotarpį nuo pažeidimo padarymo. Administracinių teisės pažeidimų bylos ypatingos tuo, kad ATPK normose nemažai numatyta atvejų, kai administracinės nuobaudos kaltiems asmenims gali būti skiriamos teisės pažeidimo padarymo vietoje (ATPK 262 str.). Atsakomybės operatyvumas didina jos efektyvumą bei veiksmingumą.
165
ADMINISTRACINE TEISE 7.3. ADMINISTRACINĖS ATSAKOMYBĖS PAGRINDAS ADMINISTRACINIS TEISĖS PAŽEIDIMAS
Administracinės atsakomybės pagrindas yra administracinis teisės pažeidimas. Tai tokia asmens veika (veikimas ar neveikimas), kuriai būdingi šie pagrindiniai požymiai: 1) veikos priešingumas teisei; 2) veikos pavojingumas (žalingumas) visuomenei; 3) veikos kaltumas; 4) atsakomybė (baudžiamumas) už veikos padarymą (ATPK 9 str.). Pirmasis požymis reiškia, kad tam tikras veikas (veiksmus, poelgius, neveikimą) draudžia įstatymas. Tokios draudžiamos veikos yra suformuluotos ATPK ypatingojoje dalyje (pvz., žinomai melagingas specialiųjų tarnybų iškvietimas (186 str.), priešgaisrinės saugos taisyklių pažeidimas (192 str.), kontrabanda (210 str.). Antrasis požymis reiškia, kad draudžiamos veikos yra dėl to, kad jos pavojingos visuomenei, sukelia negatyvius, žalingus padarinius (pažeidžiama viešoji tvarka, sužalojamas turtas, padaroma žala žmogaus sveikatai ir t. t.). Trečiasis požymis reiškia, kad priešingi teisei veiksmai (neveikimas)padaromi kaltai, t. y. tyčia ar dėl neatsargumo. Ketvirtasis požymis reiškia, kad kaltas padaręs administracinį teisės pažeidimą asmuo turi būti baudžiamas, turi atsakyti už padarytą veiką pagal įstatymą. ATPK Ypatingosios dalies normose nustatytos sankcijos, kurios taikomos teisės pažeidėjams už konkrečius padarytus pažeidimus. Administracinis teisės pažeidimas yra savarankiška teisės pažeidimo rūšis. Administracinių teisės pažeidimų sudėtys yra suformuluotos ATPK Ypatingosios dalies normose. Pagrindinis administracinio teisės pažeidimo atribojimo nuo baudžiamojo nusižengimo ir baudžiamojo nusikaltimo kriterijus yra veikos visuomenei pavojingumo laipsnis. Administracinis teisės pažeidimas yra mažiau pavojingas visuomenei negu baudžiamasis nusikaltimas ar baudžiamasis nusižengimas. Kokios teisei priešingos ir visuomenei pavojingos veikos laikomos administraciniais teisės pažeidimais ir kokios veikos priskiriamos baudžiamiesiems nusikaltimams ar baudžiamiesiems nusižengimams, sprendžia įstatymo leidėjas.
7.4. JURIDINĖ ADMINISTRACINIO TEISĖS PAŽEIDIMO SUDĖTIES ANALIZĖ
Be nurodytų bendrųjų požymių, administraciniam teisės pažeidimui, kaip ir bet kokios kitos rūšies teisės pažeidimui, yra būdingi tam tikri objektyvūs ir subjektyvūs požymiai, kurie ir įeina į juridinę šio teisės pažeidimo sudėtį. Taigi į administracinio teisės pažeidimo sudėtį įeina tokie keturi pagrindiniai elementai: 1) objektas; 2) objektyvioji pusė;3) subjektas;4) subjektyvioji pusė. Šie visi administracinio teisės pažeidimo sudėties elementai yra tarpusavyje labai susiję. Jei veikoje bent vieno iš šių elementų trūksta, tai nėra ir administracinio teisės pažeidimo sudėties. Taigi administracinio teisės pažeidimo sudėtis suprantama kaip tam tikrų juridinių veikos požymių, rodančių jos priešingumą teisei ir pavojingumą visuomenei, visuma. Juridinė administracinio teisės pažeidimo sudėties elementų analizė padeda išaiškinti jo socialinį pobūdį, turinį, individualius jo sudėtinių elementų požymius ir teisingai nustatyti juridinę veikos kvalifikaciją.
166
Administracinė atsakomybė
Administracinio teisės pažeidimo objektas gali būti: bendrasis, rūšinis ir tiesioginis. Bendrasis administracinio teisės pažeidimo objektas – tai tie visuomeniniai santykiai (materialios ir dvasinės vertybės, teisiniai gėriai), į kuriuos kėsinasi teisei priešinga veika ir tuos visuomeninius santykius gina administracinės teisės normos. Rūšinis administracinio teisės pažeidimo objektas – tai grupė teisės reguliuojamų giminingų visuomeninių santykių, į kuriuos kėsinasi teisei priešinga veika ir kuriuos gina administracinės teisės sankcijos. ATPK Ypatingojoje dalyje yra išvardyti tipiškiausi rūšiniai administracinių teisės pažeidimų objektai: valstybinė ir viešoji tvarka, nuosavybė, piliečių teisės ir laisvės, gamtos, istorijos ir kultūros paminklai, valdymo tvarka, teisingumas ir kt. Tiesioginis administracinio teisės pažeidimo objektas – tai tie teisės reguliuojami visuomeniniai santykiai, tos vertybės, gėriai, į kuriuos tiesiogiai kėsinasi veika. Pavyzdžiui, viešoji tvarka (rūšinis objektas) gali būti pažeista sutrikdžius žmonių poilsį, rimtį. Taigi šiuo atveju tiesioginis pažeidimo objektas – žmonių poilsis, rimtis. Administracinio teisės pažeidimo objektyvioji pusė-tai visuma požymių, kurie apibūdina išorinę veikos pasireiškimo formą (pažeidimo padarymo būdą). Objektyvioji administracinio teisės pažeidimo pusė paprastai yra formuluojama teisės normos dispozicijoje. Objektyviajai administracinio teisės pažeidimo pusei priskiriami visi tie išoriniai veikos pasireiškimo požymiai bei aplinkybės, kurie tą veiką ir apibūdina: 1) teisei priešingi veiksmai (neveikimas); 2) žalingi padariniai; 3) priežastinis ryšys tarp veiksmų (neveikimo) ir žalingų padarinių; 4) pažeidimo padarymo vieta, laikas, būdas, priemonės. Kitaip sakant, objektyvioji pusė rodo, kaip, kokiais konkrečiais veiksmais (neveikimu), būdais, priemonėmis, kokiomis aplinkybėmis padaromas administracinis teisės pažeidimas. Pavyzdžiui, ATPK 1883 straipsnyje pasakyta: „Neįleidimas Valstybės kontrolės pareigūnų ir jų įgaliotų asmenų tikrinti įmonių, įstaigų ir organizacijų, nepateikimas jiems dokumentų arba dokumentų nuslėpimas, klaidingas, ne visų arba ne laiku žinių suteikimas, taip pat valstybės kontrolės pareigūnų teisėtų reikalavimų ir sprendimų nevykdymas užtraukia baudą pareigūnams nuo penkių šimtų iki vieno tūkstančio litų”. Administracinio teisės pažeidimo subjektas gali būti fizinis asmuo, kuriam iki administracinio teisės pažeidimo padarymo sukako 16 metų ir yra pakaltinamas. Šešiolika metų – tai amžiaus riba, nuo kurios asmuo sugeba suvokti savo poelgius, vertinti jų sukeltus žalingus padarinius ir savo atsakomybę už įvykdytus teisei priešingus veiksmus. Administracinėn atsakomybėn netraukiamas asmuo, kuris, darydamas priešingus teisei veiksmus (veikimas ar neveikimas), buvo nepakaltinamas, t. y. negalėjo suprasti savo veiksmų esmės arba jų valdyti dėl chroniškos psichikos ligos, laikino psichinės veiklos sutrikimo, silpnaprotystės ar kitokios patologinės būsenos. Administracinėje teisėje, be paprasto subjekto, yra išskiriamas ir specialusis pažeidimo subjektas. Toks subjektas gali būti asmuo, kuriam, be bendrųjų administracinio teisės pažeidimo subjekto požymių, yra būdingi papildomi, specialūs, požymiai, numatyti teisės normose. Paprastai specialieji administracinių teisės pažeidimų subjektai yra pareigūnai (ATPK 14 str.), tėvai, prekybos įmonių darbuotojai, profesionalūs transporto priemonių vairuotojai ir kt. Reikia pasakyti, kad ne visi administracinių teisės pažeidimų subjektai yra traukiami administracinėn atsakomybėn. ATPK 15 straipsnyje pasakyta, kad tikrosios karo tarnybos kariai, taip pat policijos, vidaus reikalų tarnybų pareigūnai ir jiems
167
ADMINISTRACINE TEISE
prilyginti asmenys už administracinius teisės pažeidimus atsako pagal drausmės statutus, o Specialiųjų tyrimų tarnybos pareigūnai – pagal Specialiųjų tyrimų tarnybos statutą. Kiti asmenys, kuriems taikomi drausmės statutai arba specialūs drausmės nuostatai, tiesiogiai šiuose aktuose nurodytais atvejais už administracinius teisės pažeidimus traukiami administracinėn atsakomybėn. Užsieniečiai, padarę administracinį teisės pažeidimą, atsako pagal ATPK, jeigu kitaip nenumatyta kituose įstatymuose ir Lietuvos Respublikos tarptautinėse sutartyse (ATPK 16 str.). Administracinio teisės pažeidimo subjektyvioji pusė – tai asmens psichinis santykis su jo daroma teisei priešinga veika ir jos padariniais. Administracinio teisės pažeidimo subjektyviąją pusę sudaro šie elementai: kaltė, motyvas ir tikslas. Kaltė yra pagrindinis administracinio teisės pažeidimo subjektyviosios pusės elementas. Jeigu asmens veiksmuose nėra kaltės, negali būti ir teisės pažeidimo sudėties, kartu ir administracinės atsakomybės. Kaltė gali reikštis dviem formomis: tyčia ir neatsargumu. Administracinis teisės pažeidimas laikomas padarytu tyčia, jeigu jį padaręs asmuo suprato priešingą teisei savo veikimo arba neveikimo pobūdį, numatė žalingas jo pasekmes ir jų norėjo arba nors ir nenorėjo šių pasekmių, bet sąmoningai leido joms kilti (ATPK 10 str.). Administracinis teisės pažeidimas laikomas padarytu dėl neatsargumo, jeigu jį padaręs asmuo numatė, kad jo veikimas ar neveikimas gali sukelti žalingas pasekmes, bet lengvabūdiškai tikėjosi, kad jų bus išvengta, arba nenumatė, kad gali kilti tokios pasekmės, nors turėjo k galėjo jas numatyti (ATPK 11 str.). Administracinio teisės pažeidimo motyvas – tai paskatos, kurios stumia subjektą įvykdyti teisei priešingus veiksmus. Paskatos gali būti, pavyzdžiui, kerštas, pavydas, pyktis. Teisės pažeidimo tikslas yra tai, ko subjektas siekia darydamas teisei priešingą veiką (pvz., pasisavinti svetimą turtą, sužaloti ar sunaikinti svetimą turtą). Reikia pabrėžti, kad norint teisingai įvertinti subjektyvius ir objektyvius veikos požymius bei teisingai juridiškai kvalifikuoti veiką, būtina išsiaiškinti teisės pažeidimo padarymo tikslus bei motyvus. Kaip jau buvo minėta, veika, neturinti administracinio teisės pažeidimo požymių, negali būti pripažįstama administraciniu teisės pažeidimu. Tačiau galiojantys įstatymai numato tokius veiksmus, kurie išoriškai primena administracinio teisės pažeidimo požymius, bet iš esmės dėl tam tikrų aplinkybių nėra pavojingi visuomenei ir priešingi teisei ir todėl, juos padarius, administracinė atsakomybė netaikoma. Kitaip tariant, įstatymai numato aplinkybes, kurios pašalina veikos pavojingumą ir priešingumą teisei. Tokios aplinkybės yra būtinasis reikalingumas ir būtinoji gintis (ATPK 17 ir 18 str.).
7.5. ADMINISTRACINĖS NUOBAUDOS IR JŲ RŪŠYS
Administracinės nuobaudos yra savarankiška administracinio teisinio poveikio priemonių rūšis. Administracinių nuobaudų paskirtis bei tikslai nustatyti ATPK 20 straipsnyje. Jame rašoma: „Administracinė nuobauda yra atsakomybės priemo
168
Administracinė atsakomybė
nė, kuri skiriama administracinį teisės pažeidimą padariusiems asmenims nubausti bei siekiant auklėti, kad jie laikytųsi įstatymų, gerbtų bendro gyvenimo taisykles, taip pat kad pats teisės pažeidėjas, tiek ir kiti asmenys nepadarytų naujų teisės pažeidimų”. Administracinės nuobaudos, panašiai kaip ir baudžiamosios bausmės, yra represinio pobūdžio. Asmuo, kuriam skirta administracinė nuobauda, priverstas patirti tam tikrus neigiamo pobūdžio padarinius, t. y. jis netenka tam tikrų teisių, laisvės, materialinių vertybių arba patiria kitokius teisių apribojimus. Tačiau be represinio pobūdžio poveikio, administracinės nuobaudos turi ir kitokių tikslų. Jomis siekiama teisės pažeidėją ir kitus asmenis drausminti, auklėti ir įspėti, kad nepadarytų teisės pažeidimų. Administracinių nuobaudų efektyvumą lemia tai, kad kiekvienu atveju jos turi būti taikomos atsižvelgiant į konkretų teisės pažeidimą, jo padarymo aplinkybes. Nuobaudos griežtumas visada turi atitikti padaryto administracinio teisės pažeidimo pavojingumo visuomenei laipsnį, kalto asmens savybes, atsakomybę lengvinančias ir sunkinančias aplinkybes. Reikia pabrėžti, kad administracinių teisės pažeidimų sudėtį ir administracines nuobaudas nustato tik įstatymų leidėjas. ATPK 21 straipsnyje numatytos tokios administracinių nuobaudų rūšys: 1) įspėjimas; 2) bauda; 3) daikto, kuris buvo administracinio teisės pažeidimo padarymo įrankis arba tiesioginis objektas, ir pajamų, kurios buvo gautos padarius administracinį teisės pažeidimą, konfiskavimas; 4) suteiktos asmeniui specialiosios teisės (teisės vairuoti transporto priemones, teisės skraidyti orlaivio įgulos nariu, atlikti orlaivių techninę priežiūrą, dirbti skrydžių vadovu, teisės medžioti arba žvejoti) atėmimas; 5) administracinis areštas; 6) nušalinimas nuo darbo (pareigų). Pagal ATPK 22 straipsnį administracinės nuobaudos gali būti pagrindinės ir papildomosios. Daikto konfiskavimas, specialiosios teisės atėmimas ir nušalinimas nuo darbo (pareigų) gali būti skiriami kaip pagrindinės ir kaip papildomosios administracinės nuobaudos. ATPK 21 straipsnio 1 dalyje nurodytos kitos administracinės nuobaudos gali būti skiriamos tik kaip pagrindinės. Įspėjimas, kaip administracinė nuobauda, pareiškiamas raštu. Įstatymų numatytais atvejais įspėjimas gali būti įforminamas kitokiu nustatytu būdu. Bauda asmenims negali būti skiriama mažesnė kaip dešimt litų ir didesnė kaip vienas tūkstantis litų. Pareigūnams skiriama bauda negali būti mažesnė kaip dvidešimt litų ir didesnė kaip du tūkstančiai litų. Už atskirų rūšių administracinius teisės pažeidimus įstatyme gali būti nustatyta didesnė bauda, negu numatyta ATPK 24 straipsnyje. Už administracinius teisės pažeidimus nepilnamečiui gali būti skiriama ne daugiau kaip pusė baudos, numatytos ATPK. ATPK numatytais atvejais kaip alternatyvi nuobauda rajono (miesto) apylinkės teismo nutarimu ir pažeidėjo sutikimu bauda gali būti pakeista nemokamais viešaisiais darbais. Daikto ar pajamų, gautų padarius administracinį teisės pažeidimą, konfiskavimas yra priverstinis neatlygintinas šio daikto ir šių pajamų pavertimas valstybės nuosavybe. Konfiskuotas gali būti tik daiktas ir pajamos, kurie yra pažeidėjo nuosavybė, išskyrus daiktą ir pajamas, kurie buvo administracinio teisės pažeidimo padarymo įrankis arba tiesioginis objektas ir kurie buvo gauti padarius administracinį teisės pažeidimą, numatytą ATPK 1632, 173 straipsniuose, 2091 straipsnio 2 dalyje, 2092 straipsnio 1,2 ir 3 dalyse, 2093 straipsnyje, 2094 straipsnio 3 ir 4 daly
169
ADMINISTRACINE TEISE
se, 210 ir 21410 straipsniuose. Šaunamojo ginklo, kitų medžioklės įrankių ir šaudmenų konfiskavimas negali būti skiriamas asmenims, kuriems medžioklė yra pagrindinis pragyvenimo šaltinis. Specialiosios teisės atėmimas skiriamas iki penkerių metų už šiurkštų arba sistemingą naudojimosi šia teise tvarkos pažeidimą. Tokios teisės atėmimo laikas negali būti trumpesnis kaip vienas mėnuo, jeigu kitaip nenustato įstatymai. Atėmimas teisės vairuoti transporto priemones negali būti skiriamas asmenims, kurie naudojasi šiomis priemonėmis dėl invalidumo, išskyrus atvejus, kai jas vairuoja neblaivūs. Atėmimas teisės medžioti arba žvejoti negali būti skiriamas asmenims, kuriems medžioklė arba žvejyba yra pagrindinis pragyvenimo šaltinis. Administracinis areštas nustatomas ir skiriamas tik išimtiniais atvejais už atskirų rūšių (pavojingesnių) administracinius teisės pažeidimus iki 30 parų. Administracinį areštą skiria rajono (miesto) apylinkės teismas (teismo teisėjas). Administracinis areštas negali būti skiriamas nėščioms moterims, moterims, turinčioms vaikų iki dvylikos metų amžiaus, asmenims, kuriems nesukako aštuoniolika metų, pirmos ir antros grupės invalidams. Nušalinimas nuo darbo (pareigų) skiriamas už atskirų rūšių administracinius teisės pažeidimus, padarytus ryšium su darbuotojo tarnybinių pareigų atlikimu. Nušalinimą nuo darbo (pareigų) skiria rajono (miesto) apylinkės teismas (teisėjas). ATPK 35 straipsnyje nustatyti administracinių nuobaudų skyrimo terminai. Administracinė nuobauda gali būti skiriama ne vėliau kaip per šešis mėnesius nuo teisės pažeidimo padarymo dienos, o esant trunkamam teisės pažeidimui (kuris nepertraukiamai tęsiasi iki jo nutraukimo, pvz., vengimas karinės įskaitos), – per šešis mėnesius nuo jo paaiškėjimo dienos. Administracinė nuobauda už ATPK dvyliktajame skirsnyje nurodytus pažeidimus, taip pat už 85,1852,1933, 209, 2091, 2092, 2093, 2094, 210, 21411 straipsniuose numatytus pažeidimus gali būti skiriama ne vėliau kaip per šešis mėnesius nuo pažeidimo nustatymo dienos, jeigu nuo pažeidimo padarymo dienos iki jo nustatymo dienos nėra praėję daugiau kaip vieneri metai. Atsisakius kelti baudžiamąją bylą arba nutraukus baudžiamąją bylą, jei pažeidėjo veiksmuose yra administracinio teisės pažeidimo požymių, administracinė nuobauda gali būti skiriama ne vėliau kaip per du mėnesius nuo sprendimo atsisakyti kelti baudžiamąją bylą arba ją nutraukti priėmimo dienos. Jeigu teisės pažeidėjas neturi nuolatinės gyvenamosios vietos, ilgam išvykęs ar gyvena užsienyje, ilgai serga arba kai dėl pažeidimo tyrimo ar kitų priežasčių negalima spręsti jo administracinės atsakomybės klausimo, ATPK 35 straipsnio 1 ir 2 dalyse nurodyti terminai pratęsiami, bet ne ilgiau kaip vieneriems metams, šį terminą skaičiuojant nuo teisės pažeidimo padarymo ar paaiškėjimo dienos arba nuo sprendimo atsisakyti kelti baudžiamąją bylą ar ją nutraukti priėmimo dienos. Kai pirmosios instancijos teismo nutarimas panaikinamas apeliacine tvarka, ATPK 35 straipsnyje nurodyti terminai pradedami skaičiuoti iš naujo nuo apeliacinės instancijos sprendimo įsiteisėjimo dienos. Jeigu asmuo, kuriam buvo paskirta administracinė nuobauda, per metus nuo tos dienos, kai pasibaigė nuobaudos vykdymas, nepadarė naujo administracinės teisės pažeidimo, laikoma, kad jam nebuvo paskirta administracinė nuobauda.
170
Administracinė atsakomybė 7.6. ADMINISTRACINIŲ NUOBAUDŲ SKYRIMO, APSKUNDIMO IR VYKDYMO PROCESINĖ TVARKA
Vienas iš svarbiausių administracinių nuobaudų skyrimo ypatumų yra tas, kad administracinių teisės pažeidimų bylas nagrinėja ir nuobaudas kaltiems asmenims taiko labai daug valstybės institucijų ir pareigūnų. Tai paaiškinti galima tuo, kad administracinių teisės pažeidimų padaroma palyginti daug ir įvairiose valdymo srityse. Valdymo institucijos, vadovaudamos tam tikroms socialinės kultūrinės, ūkinės ir administracinės politinės veiklos sritims, turi užtikrinti, kad būtų laikomasi nustatytos valdymo tvarkos, ginamos ir saugomos fizinių ir juridinių asmenų teisės, sėkmingai įgyvendinami visi uždaviniai ir funkcijos. Sėkmingai kovoti su administraciniais teisės pažeidimais ir kvalifikuotai nagrinėti tokių pažeidimų bylas gali tos institucijos bei jų pareigūnai, kurie turi specialių žinių ir yra kompetentingi. Tokie subjektai paprastai yra įvairios valstybinės inspekcijos, administracinės komisijos prie savivaldybių tarybų, apylinkės teismai (teisėjai), policijos ir kiti pareigūnai. Taigi pagal ATPK 216 straipsnį administracinių teisės pažeidimų bylas nagrinėja ir nuobaudas pažeidėjams skiria: 1) administracinės komisijos prie savivaldybių tarybų; 2) policija; 3) valstybinės inspekcijos; 4) seniūnijų kaimo vietovėse seniūnai; 5) rajonų (miestų) apylinkės teismai (teisėjai); 6) kitos įstatymų tam įgaliotos institucijos bei pareigūnai – įvairių transporto rūšių, aplinkos apsaugos, muitinės, valstybinės sienos apsaugos, krašto apsaugos, valstybinės tabako ir alkoholio kontrolės tarnybos ir kitų institucijų pareigūnai. Administracinių teisės pažeidimų bylos tiriamos, nagrinėjamos, kaltiems asmenims nuobaudos skiriamos ir jos įgyvendinamos tam tikra administracinės teisės normose nustatyta tvarka, kuri vadinama administracinių teisės pažeidimų bylų teisena. Taigi administracinių teisės pažeidimų bylų teisena -tai įstatymų reglamentuojama procesinė administracinių pažeidimų bylų iškėlimo, jų tyrimo ir nagrinėjimo, nutarimų šiose bylose priėmimo, jų apskundimo ir vykdymo tvarka. Administracinių teisės pažeidimų bylų teisena yra grindžiama teisėtumo, visų piliečių lygybės įstatymui ir institucijai (pareigūnui), tiriančiai ir nagrinėjančiai administracinio teisės pažeidimo bylą, objektyvumo, nekaltumo prezumpcijos, teisingumo, teisės į gynybą, lietuvių kalbos, viešumo ir operatyvumo principais. Kad administracinių teisės pažeidimų bylų teisena pasiektų savo tikslą ir būtų sėkmingai įgyvendinti jos uždaviniai, ATPK normose numatytos tam tikros priemonės. ATPK 264 straipsnyje rašoma: „Lietuvos Respublikos įstatymų tiesiogiai numatytais atvejais, siekiant užkirsti kelią administraciniams teisės pažeidimams, surašyti protokolams, užtikrinti kad būtų laiku ir teisingai nagrinėjamos bylos ir vykdomi nutarimai administracinių teisės pažeidimų bylose, leidžiamas asmens administracinis sulaikymas, asmens apžiūra, daiktų patikrinimas, daiktų ir dokumentų paėmimas, priverstinis transporto priemonės nuvežimas, transporto priemonės važiuoklės užblokavimas specialiu įtaisu arba vairuotojo nušalinimas nuo transporto priemonės vairavimo ir patikrinimas neblaivumui (girtumui) ar apsvaigimui nuo narkotikų arba vaistų ar kitų svaigiųjų medžiagų nustatyti”. Be minėtų teisenos užtikrinimo priemonių, ATPK 263 straipsnyje numatytas ir teisės pažeidėjo pristatymas jo asmenybei nustatyti, teisės pažeidimo faktui išaiškinti ir pažeidimo protokolui surašyti. Pristatyti asmenį į policiją, seniūnijos kaimo vietovėje bei kitų atitinkamų įstaigų būstinę turi teisę ATPK 263 straipsnyje nurodyti pareigūnai. Pristatytas asmuo gali būti laikomas iki 3 valandų.
171
ADMINISTRACINE TEISE
Administracinio asmens sulaikymo atvejai, sąlygos, terminai, pareigūnai, turintys teisę taikyti šią poveikio priemonę, numatyti ATPK 265-267 straipsniuose. Administracinis asmens sulaikymas gali trukti ne ilgiau kaip 5 valandas. Pasienio ruožo režimą arba pasienio kontrolės punktų režimą pažeidęs asmuo gali būti sulaikytas iki 3 valandų protokolui surašyti, o prireikus nustatyti jo asmenybę ir išaiškinti teisės pažeidimo aplinkybes – iki 48 valandų. Už kai kuriuos administracinius teisės pažeidimus (pvz., susirinkimų ir kitų masinių renginių tvarkos pažeidimą, nedidelį chuliganizmą) asmuo gali būti sulaikytas, kol apylinkės teismo teisėjas ar policijos komisaras įstatymo nustatytais terminais išnagrinės bylą, bet ne ilgiau kaip 48 valandas. Administracinio sulaikymo laikas skaičiuojamas nuo pažeidėjo pristatymo protokolui surašyti momento, o girto asmens – nuo jo išblaivėjimo laiko. Asmens apžiūros ir daiktų patikrinimo, daiktų ir dokumentų paėmimo, priverstinio transporto priemonės nuvežimo ar jos važiuoklės blokavimo, asmens nušalinimo nuo transporto priemonių vairavimo atvejai ir sąlygos nustatyti ATPK 268, 269, 270 straipsniuose. Administracinių teisės pažeidimų bylų teisena yra sudėtingos struktūros. Ją sudaro tam tikros viena paskui kitą nuoseklumo tvarka einančios stadijos. Kiekviena stadija susideda iš specifinių vienarūšių procesinių veiksmų ir turi savo tiesioginę paskirtį. Pagal ATPK normas, reglamentuojančias administracinių teisės pažeidimų bylų teisenos procesinius veiksmus, galima skirti tokias savarankiškas šios-teisenos stadijas: 1) bylos iškėlimas; 2) bylos nagrinėjimas; 3) nutarimo priėmimas bei paskelbimas; 4) nutarimo apskundimas; 5) nutarimo vykdymas. Administracinių teisės pažeidimų bylų teisenoje dalyvauja daug asmenų, kurių procesinės teisės, pareigos ir apskritai jų teisinė padėtis įtvirtinta ATPK normose. Taigi pagal ATPK dvidešimtajame skirsnyje įtvirtintas nuostatas administracinių teisės pažeidimų bylų teisenoje dalyvauja: 1) atsakomybėn traukiamas asmuo; 2) nukentėjusysis; 3) atstovas pagal įstatymą; 4) įgaliotas atstovas; 5) liudytojai; 6) ekspertas; 7) vertėjas; 8) institucijos, kurios pareigūnas surašė administracinio teisės pažeidimo protokolą, atstovas. Institucija (pareigūnas), tirianti bei nagrinėjanti administracinio teisės pažeidimo bylą, turi užtikrinti nurodytų asmenų procesines teises, taip pat reikalauti, kad minėti asmenys laikytųsi nustatytų procesinių pareigų. Atsakomybėn traukiamo asmens kaltumas turi būti pagrįstas įrodymais. ATPK 256 straipsnis, pabrėždamas įrodymų reikšmę ir apibrėždamas įrodymų sąvoką, teigia: „Įrodymai administracinio teisės pažeidimo byloje yra bet kurie faktiniai duomenys, kuriais remdamiesi organai (pareigūnai) įstatymo nustatyta tvarka nustato, ar yra padarytas administracinis teisės pažeidimas, ar jo nėra, ar dėl jo padarymo tas asmuo kaltas, ir kitokias aplinkybes, turinčias reikšmės bylai teisingai išspręsti”. Bylos iškėlimas. Tai pirmoji administracinių teisės pažeidimų bylų teisenos stadija, nuo kurios tiesiogiai priklauso ir kitos šių bylų teisenos stadijos. Šios stadijos esmę sudaro tai, kad tam įgaliotos institucijos pareigūnai nustato administracinio teisės pažeidimo faktą, asmenį, nusikaltusį padarius pažeidimą, ir iškelia jam bylą. Bylos iškėlimas įforminamas protokolo surašymu (kai kuriais įstatymo numatytais atvejais byla gali būti iškeliama prokuroro, teismo, teisėjo nutarimu). Surašyti administracinio teisės pažeidimo protokolą turi teisę asmenys, nurodyti ATPK 259, 2591 straipsniuose (policijos, vidaus reikalų, valstybinių inspekcijų, muitinės ir kiti pareigūnai). Protokolas surašomas laikantis ATPK 260 straipsnyje nurodytų reika
172
Administracinė atsakomybė
lavimų ir ne vėliau kaip per tris dienas nuo jo surašymo pasiunčiamas institucijai (pareigūnui), įgaliotai nagrinėti bylą. Bylos nagrinėjimas. Kaip minėta, vienas svarbiausių administracinių teisės pažeidimų bylų teisenos ypatumų yra tas, kad šių pažeidimų bylas nagrinėja gana daug įvairių institucijų ir pareigūnų. Šie subjektai numatyti ATPK 216 straipsnyje (administracinės komisijos prie savivaldybių tarybų, policija, valstybinės inspekcijos, teismas ir kt.). Bylų nagrinėjimo vieta, procesinė tvarka, asmenų, dalyvaujančių šiame procese, teisės bei pareigos, kiti procesiniai veiksmai yra reglamentuojami ATPK 272-284 straipsniuose. Nutarimo priėmimas. Išnagrinėjusi administracinio teisės pažeidimo bylą, institucija (pareigūnas) priima nutarimą. Nutarimas yra svarbiausias procesinis aktas, kuriuo išsprendžiama, ar yra padarytas administracinis teisės pažeidimas ar jo nėra, ar traukiamas atsakomybėn asmuo yra kaltas, ar nekaltas, skiriama jam administracinė nuobauda ar neskiriama. Nutarimo turinys nurodytas ATPK 286 straipsnyje. Pagal ATPK 287 straipsnį byloje gali būti priimtas toks nutarimas: 1) skirti pažeidėjui administracinę nuobaudą; 2) nutraukti bylą (nutarimas nutraukti bylą gali būti priimamas esant vienai iš aplinkybių, numatytų ATPK 250 ir 251 straipsniuose); 3) motyvuotai perduoti bylos nagrinėjimą kitai įgaliotai nagrinėti bylą institucijai (pareigūnui). Nutarimas skelbiamas tuojau pat, baigus bylos nagrinėjimą. Nutarimo nuorašas per tris dienas įteikiamas (pasirašytinai) arba išsiunčiamas asmeniui, dėl kurio nutarimas priimtas, institucijai, kurios pareigūnas surašė teisės pažeidimo protokolą, taip pat nukentėjusiajam jo prašymu. Nutarimo apskundimas. Ši teisenos stadija – fakultatyvi, nes nutarimas nebūtinai turi būti skundžiamas. Tačiau tais atvejais, kai patrauktas administracinėn atsakomybėn asmuo, nukentėjusysis ar institucija, kurios pareigūnas surašė administracinio teisės pažeidimo protokolą, yra nepatenkinti priimtu nutarimu, jį gali apskųsti įstatymo nustatyta tvarka. ATPK 291 straipsnyje pasakyta, kad nutarimą byloje gali apskųsti asmuo, kurio atžvilgiu jis priimtas, institucija, kurios pareigūnas surašė teisės pažeidimo protokolą, taip pat nukentėjusysis. Nutarimas administracinio teisės pažeidimo byloje, priimtas ATPK 216 straipsnio 1, 2 ir 5 punktuose nurodytų institucijų (pareigūnų), gali būti apskųstas atitinkamam apygardos administraciniam teismui. Skundas gali būti paduotas per 10 dienų nuo nutarimo priėmimo dienos. Jeigu šis terminas praleistas dėl svarbių priežasčių, jį pareiškėjo prašymu gali atnaujinti apygardos administracinis teismas. Apygardos administracinis teismas bylą pagal gautą skundą išnagrinėja per 20 dienų pirmosios instancijos teismo proceso tvarka. Tai reiškia, kad byla nagrinėjama iš esmės aiškinantis visas jos aplinkybes. Išnagrinėjęs bylą pagal skundą, apygardos administracinis teismas priima vieną iš sprendimų, nurodytų Lietuvos Respublikos administracinių bylų teisenos įstatymo 124 straipsnyje. Apygardos administracinio teismo nutarimas per 10 dienų nuo jo paskelbimo gali būti skundžiamas apeliacine tvarka Lietuvos vyriausiajam administraciniam teismui, kurio sprendimas yra galutinis ir neskundžiamas. Apylinkės teismo (teisėjo) nutarimas administracinio teisės pažeidimo byloje per 10 dienų nuo jo paskelbimo gali būti skundžiamas apeliacine tvarka Lietuvos vyriausiajam administraciniam teismui, kurio sprendimas yra galutinis ir neskundžiamas. Padavus skundą nustatytu laiku, sustabdomas nutarimo skirti nuobaudą (išskyrus įspėjimą ir administracinį areštą) vykdymas iki skundo išnagrinėjimo.
173
ADMINISTRACINĖ TEISĖ
Nutarimo vykdymas. Nutarimas administracinio teisės pažeidimo byloje vykdomas jam įsiteisėjus (išskyrus kai skiriamas įspėjimas ir areštas). Nutarimų skirti nuobaudą vykdymo tvarką reglamentuoja ATPK nuostatos. Pagal ATPK 304 straipsnio 4 dalį nutarimą vykdyti nukreipia jį priėmusi institucija (pareigūnas). ATPK 308 straipsnyje nustatytas nutarimų skirti administracinę nuobaudą vykdymo senaties terminas. Negali būti vykdomas nutarimas skirti administracinę nuobaudą, jeigu jis nebuvo perduotas vykdyti per 3 mėnesius nuo priėmimo dienos. Jeigu nutarimo vykdymas sustabdytas pagal ATPK 295 straipsnį, senaties termino eiga sustabdoma iki skundo išnagrinėjimo. Atidėjus nutarimo vykdymą, senaties termino eiga sustabdoma iki termino atidėjimo pabaigos. Nutarimo dėl administracinio arešto vykdymo senatis – vieneri metai. ATPK 306 straipsnyje pasakyta, kad, esant aplinkybių, dėl kurių nedelsiant įvykdyti nutarimą skirti administracinį areštą ar baudą negalima, priėmusi nutarimą institucija (pareigūnas) pagal asmens pareiškimą gali atidėti nutarimo vykdymą iki vieno mėnesio. Atsižvelgdama į pažeidėjo turtinę padėtį, kitas reikšmingas aplinkybes, nagrinėjanti administracinio teisės pažeidimo bylą institucija (pareigūnas) gali paskirtos baudos mokėjimą išdėstyti per laikotarpį iki dvejų metų (ATKP 313 str.) ATPK 3381 straipsnyje numatyta, kad administracinio arešto vykdymas gali būti sąlyginai atidėtas. Tai reiškia, kad asmeniui, kuriam paskiriamas administracinis areštas arba kuriam bauda pakeičiama administraciniu areštu šio kodekso 314 straipsnyje nustatyta tvarka, apylinkės teismas (teisėjas) pažeidėjui sutinkant tuo pačiu nutarimu gali atidėti arešto vykdymą nuo 1 iki 12 mėnesių. Atidėdamas paskirto arešto vykdymą, teismas paskiria pažeidėjui atlikti nemokamų viešųjų darbų nuo 25 iki 400 valandų. Detali nutarimų dėl konkrečių administracinių nuobaudų vykdymo procesinė tvarka reglamentuojama ATPK dvidešimt penktojo-trisdešimt pirmojo skirsnių nuostatose. Lietuvos Respublikos administracinių bylų teisenos įstatymas numato atvejus, kuriais gali būti atnaujinta administracinių teisės pažeidimų bylų teisena (įsiteisėjusių teismo nutarimų peržiūrėjimas).
174
PAGRINDINĖS SĄVOKOS Administracinė teisė – savarankiška teisės šaka, kurios normos reguliuoja visuomeninius santykius viešojo valdymo srityje, t. y.: 1) valstybės vykdomosios valdžios įgyvendinimo procese; 2) visų valstybės ir savivaldybių institucijų vidaus valdymo veikloje, įmonių, įstaigų, teikiančių socialines, kultūros, švietimo paslaugas, administracijos veikloje; be to, 3) jos normos reguliuoja visuomeninius santykius apylinkės ir administraciniams teismams, valdymo institucijoms (pareigūnams) nagrinėjant administracinių teisės pažeidimų bylas bei skundus, ginant fizinių asmenų bei kitų teisės subjektų teises ir jų teisėtus interesus. Administracinės teisės normos – savarankiškos ir specifinės teisės normos, kurios taikant administracinį teisinį metodą reguliuoja visuomeninius santykius viešojo valdymo srityje. Administraciniai teisiniai santykiai – administracinės teisės normų sureguliuoti visuomeniniai santykiai. Valdymas – tai tam tikroje sistemoje pasireiškiantis tikslingas bei kryptingas poveikis, kurį valdymo subjektas daro valdymo objektui, siekdamas išlaikyti sistemą tam tikroje organizacinėje struktūroje, išspręsti reikiamus uždavinius, pasiekti užsibrėžtą tikslą. Viešasis administravimas – įstatymų ir kitų teisės aktų reguliuojama valstybės ir vietos savivaldos institucijų, kitų įstatymais įgaliotų subjektų vykdomoji veikla, skirta įstatymams, kitiems teisės aktams, vietos savivaldos institucijų sprendimams įgyvendinti, numatytoms viešosioms paslaugoms administruoti. Viešojo valdymo subjektai – tai valstybės ir savivaldybių institucijos, įstaigos, tarnybos, valstybės tarnautojai (pareigūnai) ir kiti įstatymuose numatyti subjektai, kurių tiesioginė paskirtis – įgyvendinti valstybės vykdomosios valdžios uždavinius arba atskiras jos funkcijas, vadovauti socialinės kultūrinės, ūkinės ir administracinės politinės veiklos sritims. Ministerija – šakinės kompetencijos valdymo institucija, vadovaujanti jai pavestai valdymo sričiai (šakai). Kolegija – ministro patariamoji institucija, sudaroma iš tos ministerijos valstybės tarnautojų. Vyriausybės įstaigos – tai ministerijų funkcijoms nepriskirtiems klausimams spręsti Vyriausybės įsteigtos institucijos – departamentai, kontrolės ar apskaitos funkcijas vykdančios tarnybos, agentūros, inspekcijos ir kitos įstaigos. Apskritis – aukštesnysis administracinis teritorinis vienetas, kurio valdymą per apskrities viršininką, ministerijas bei kitas Vyriausybės institucijas organizuoja Vyriausybė. Lietuvos Respublikoje yra 10 apskričių: Vilniaus, Kauno, Klaipėdos, Šiaulių, Panevėžio, Alytaus, Tauragės, Marijampolės, Telšių ir Utenos. Savivaldybės valdymo (administravimo) subjektai – tai savivaldybės institucijos ir joms pavaldžios įstaigos bei kiti subjektai, turintys teisės aktų suteiktus įgaliojimus, atliekantys jiems pavestas valdymo funkcijas ir atsakingi už šių funkcijų įgyvendinimą. Savivaldybės institucijos yra atstovaujamoji institucija (sa
175
ADMINISTRACINĖ TEISĖ
vivaldybės taryba) ir vykdomoji institucija (savivaldybės administracijos direktorius), kurios turi vietos valdžios ir viešojo valdymo (administravimo) teises ir pareigas. Savivaldybės taryba – savivaldos institucija, kuri susideda iš įstatymų nustatyta tvarka demokratiškai išrinktų savivaldybės bendruomenės atstovų (savivaldybės tarybos narių). Meras – savivaldybės tarybos pirmininkas. Savivaldybės kontrolierius – tai savivaldybės institucija, kuri kontroliuoja, kaip vykdomas savivaldybės biudžetas ir naudojamas jos turtas. Savivaldybės administracija – savivaldybės įstaiga, į kurios sudėtį įeina struktūriniai padaliniai – departamentai, skyriai, tarnybos ir kt. ir struktūriniai teritoriniai padaliniai – seniūnijos. Seniūnija – savivaldybės administracijos struktūrinis teritorinis padalinys, veikiantis tam tikroje savivaldybės teritorijos dalyje. Valstybės tarnyba – visuma teisinių santykių, kurie atsiranda įgijus valstybės tarnautojo statusą valstybės ar savivaldybės institucijoje ar įstaigoje ryšium su viešojo valdymo veiklos vykdymu. Valdymo aktai – tai poįstatyminiai aktai, kurie priimami vykdant įstatymus ir remiantis įstatymais. Administracinė atsakomybė – asmens pareiga atsakyti už padarytą administracinės teisės pažeidimą. Administracinis teisės pažeidimas – tai asmens veika (veikimas ar neveikimas), kuri yra priešinga teisei, pavojinga (žalinga) visuomenei ir kurią padarius atsiranda kaltumas ir administracinė atsakomybė (baudžiamumas). Administracinė nuobauda – atsakomybės priemonė, kuri skiriama siekiant nubausti administracinį teisės pažeidimą padariusį asmenį bei priversti jį laikytis įstatymų, gerbti bendro gyvenimo taisykles, taip pat siekiant auklėti jį, kad nepadarytų naujų teisės pažeidimų. Administracinių teisės pažeidimų bylų teisena – tai įstatymų reglamentuojama procesinė administracinių pažeidimų bylų iškėlimo, jų tyrimo ir nagrinėjimo, nutarimų šiose bylose priėmimo, jų apskundimo ir vykdymo tvarka.
KLAUSIMAI
1. Kas sudaro administracinės teisės reguliavimo dalyką? 2. Kuo skiriasi civilinis teisinis metodas nuo administracinio teisinio reguliavimo metodo? 3. Kokia yra administracinės teisės normos struktūra? 4. Kokie yra pagrindiniai viešojo valdymo bruožai?
176
Pagrindinės sąvokos, klausimai, užduotys
5. Kokiame teisės akte reglamentuojama Vyriausybės darbo tvarka? Kokia pagrindinė Vyriausybės darbo forma? 6. Kokios yra ministro funkcijos? 7. Kuo skiriasi ministerijos sekretoriaus ir ministerijos valstybės sekretoriaus funkcijos? 8. Kokios šiuo metu Lietuvoje yra ministerijos? 9. Kokios pagrindinės apskrities viršininko funkcijos? 10. Kada sudaroma savivaldybės tarybos kolegija? Kas tampa šios kolegijos nariais pagal pareigas? 11. Kokie yra bendrieji asmenų priėmimo į valstybės tarnybą reikalavimai? 12. Kuo valdymo aktas skiriasi nuo civilinės sutarties? 13. Kokia tvarka apskundžiamas nutarimas administracinio teisės pažeidimo byloje?
UŽDUOTYS
1. Pagal poveikio subjektams pobūdį administracinės teisės normos skirstomos į įpareigojančias, įgaliojančias, draudžiamąsias. Paanalizuokite Administracinių teisės pažeidimų kodeksą ir pateikite minėtų normų pavyzdžių. 2. Apskrities viršininkas priėmė sprendimą, kuriuo valstybinę žemę išnuomavo savo žmonai. Kokius viešojo valdymo principus pažeidė apskrities viršininkas? 3. Apibūdinkite Valstybinę mokesčių inspekciją prie Finansų ministerijos pagal viešojo valdymo subjektų rūšis. 4. Nubraižykite schemą, kurioje būtų pavaizduota ministerijos struktūra. 5. Lietuvos pilietis, dvidešimt vienerių metų A. J., sulaikytas neblaivus prie automobilio vairo. Išanalizuokite šį administracinės teisės pažeidimą pagal administracinio teisės pažeidimo sudėtį (objektas, objektyvioji pusė, subjektas, subjektyvioji pusė).
177

Parašykite komentarą

El. pašto adresas nebus skelbiamas. Būtini laukeliai pažymėti *